You are on page 1of 260

BULGARIAN NATIONAL OLYMPIAD IN MATHEMATICS

Third round 1995

no common point with the x-axis. Prove that there exist points A and B on the parabola such that the segment AB is parallel to the x-axis and 6 AOB = 90 (O is the coordinate origin) if and only if p2 < q 1 4 . Find the values of p and q for which the points A and B are de ned in an unique way. Solution. Since the parabola has no common point with the x-axis, then the roots of the equation x2 ; 2px + q = 0 are not real and hence p2 < q . Let the points A(x1 y0 ) and B (x2 y0) (Figure 1) be with the required properties. Then x1 and x2 are the roots of the equation x2 ; 2px + q ; y0 = 0 and y0 > q ; p2, because the vertex of the parabola has coordinates 2 2 2 2 2 2 (p q ; p2 ). On the other hand OA2 = x2 1 + y0 , OB = x2 + y0 , AB = (x1 ; x2 ) and it follows 2 2 ; y + q = 0. from the Pythagorean theorem that y0 + x1x2 = 0. But x1 x2 = q ; y0 and thus y0 0 Consequently the existence of the points A and B is equivalent to the assertion that the equation f (y) = y2 ; y + q = 0 has a solution y0 > q ; p2. (A and B are de ned in an unique way if this is the only solution.) A necessary condition is that the discriminant of the equation is not 1 . The last condition is su cient because f (q ; p2) = (q ; p2 ) + p2 > 0 and negative, i.e. q 4 1 > 1 q ; p2 . The corresponding solution y is unique i q = 1 . 0 2 4 4 Figure 1. Figure 2.

Problem 1. Let p and q be positive numbers such that the parabola y = x2 ; 2px + q has

Problem 2. Let A1A2A3A4A5A6A7, B1B2B3 B4B5B6B7, C1C2C3C4C5C6C7 be regular heptagons with areas SA , SB and SC , respectively. Let A1 A2 = B1 B3 = C1C4. Prove that 1 < SB + SC < 2 ; p2: 2 SA Solution. Let A1 A2 = a, A1 A3 = b, A1 A4 = c (Figure 2). By the Ptolomeus theorem for the quadrangle A1 A3 A4 A5 it follows that ab + ac = bc, i.e. a + a = 1. Since 4A1A2 A3 = 4B1 B2 B3 , b c 2 2 2 2 B a a 1 B2 then = and hence B B = . Analogously C C = a . Therefore SB + SC = a + a .

1 2 1 2 B12B3 2 b b c SA b2 c2 a a 1 a a 1 a a Then + > ( + )2 = (equality is not possible because 6= ). On the other hand b c 2 b c 2 b c a2 + a2 = a + a 2 ; 2a2 = 1 ; 2a2 : (1) b2 c2 b c bc bc

sin 7 2 1 2 < By the sine theorem we get a = = . Since cos bc sin2 sin 4 7 4 cos 27 (1 + cos 27 ) 7 7 p 2 1 p = p2 ; 1. From here and from (1) we get the right hand p > cos 4 = 22 , then a bc
2

4 22 (1 + 22 ) side inequality of the problem. Problem 3. Let n > 1 be an integer. Find the number of the permutations (a1 a2 : : : an) of the numbers 1 2 : : : n with the following property: there exists only one index i 2 f1 2 : : : n ; 1g such that ai > ai+1 . Solution. Denote by pn the number of the permutations with the given properties. Obviously, p1 = 0 and p2 = 1. Let n 2. The number of the permutations with an = n is equal to pn;1. Consider all the permutations (a1 a2 : : : an ) with ai = n, where 1 i n ; 1 is xed. Their ! ;1 number is n i ; 1 . Consequently

pn = pn;1 +
From here

n ;1 X i=1

n ; 1 = p + 2n;1 ; 1: n;1 i;1

pn = (2n;1 ; 1) + (2n;2 ; 1) + + (2 ; 1) = 2n ; n ; 1: Problem 4. Let n 2 and 0 xi 1 for i = 1 2 : : : n. Prove the inequality (x + x + + x ) ; (x x + x x + + x x + x x ) n :


1 2

1 2

2 3

n;1 n

When is there an equality? Solution. Denote by S (x1 x2 : : : xn ) the left hand side of the inequality. This function is linear with respect to each of the variables xi . Particularly, S (x1 x2 : : : xn) max (S (0 x2 : : : xn) S (1 x2 : : : xn)) : 2

From here it follows by induction that it is enough to prove the inequality when all xi are equal to 0 or 1. On the other hand for arbitrary xi we have i.e. S (x1 x2 : : : xn ) n , when xi 2 0 1]. In the case when xi are equal to 0 or 1, the left 2 hand side of the last inequality is an integer. Consequently S (x1 x2 : : : xn) n 2 . It follows from ( ) that when n is even, the equality is satis ed i (x1 x2 : : : xn) = (0 1 0 1 : : : 0 1) when n is odd, the equality is satis ed i (x1 x2 : : : xn ) = (x 0 1 0 1 : : : 0 1), where x 2 0 1] is arbitrary. Problem 5. The points A1, B1, C1 lie on the sides BC , CA, AB of the triangle ABC respectively and the lines AA1 , BB1 , CC1 have a common point M . Prove that if the point M is center of gravity of 4A1B1 C1, then M is the center of gravity of 4ABC . Solution. Let M be the center of gravity of 4A1B1 C1. Let A2 be a point on MA! such that B1 A1 C1A2 is a parallelogram. The points B2 and C2 are constructed analogously. Since A1C1kA1B1kC1B2, then the points A2, C1, B2 are colinear and C1 is the midpoint of A2B2. The same is true for the points A2 , B1 , C2 and C2 , A1 , B2 . We shall prove that A2 = A, B2 = B and C2 = C , which will solve the problem. Assume that A2 6= A and let A be between A2 and M . Then C2 is between C and M , B is between B2 and M and consequently A2 is between A and M , which is a contradiction. 2 + y2 Problem 6. Find all pairs of positive integers (x y) for which xx ; y is an integer which is a divisor of 1995. Solution. It is enough to nd all pairs (x y ) for which x > y and x2 + y 2 = k(x ; y ), where k divides 1995 = 3:5:7:19. We shall use the following well-known fact: if p is a prime number of the type 4q + 3 and if it divides x2 + y 2 , then p divides x and y . (For p = 3 7 19 this can be proved directly.) If k is divisible by 3 then x and y are divisible by 3 too. Simplifying by 2 9 we get an equality x2 1 + x1 = k1(x1 ; y1 ), where k1 divides 5:7:19. Considering 7 and 19 in an analogous way we get an equality a2 + b2 = 5(a ; b) (it is not possible to get an equality a2 + b2 = (a ; b)), where a > b. From here (2a ; 5)2 + (2b ; 5)2 = 50, i.e. a = 3, b = 1 or a = 2, b = 1. The above considerations imply that the pairs we are looking for are of the type (3c c), (2c c), (c 3c), (c 2c), where c = 1 3 7 19 3:7 3:19 7:19 3:7:19. 2S (x1 x2 : : : xn) = n ; (1 ; x1 )(1 ; x2) ; (1 ; x2)(1 ; x3 ) ; ; ;(1 ; xn)(1 ; x1) ; x1x2 ; x2x3 ; ; xn x1 ()

BULGARIAN NATIONAL OLYMPIAD IN MATHEMATICS


Fourth round 1995

by n for every integer a. Solution. Let n be with the required property. Then p2 (p prime) does not divide n since p2 does not divide p25 ; p. Hence n is a product of pairwise di erent prime numbers. On the other hand 225 ; 2 = 2:32:5:7:13:17:241. But n is not divisible by 17 and 241 because 325 ;3 (mod 17) and 325 32 (mod 241). The Fermat theorem implies that a25 a (mod p) when p = 2 3 5 7 13. Thus n should be equal to the divisor of 2 3 5 7 13, di erent from 1. Therefore the number we are looking for is 25 ; 1 = 31. Problem 2. A triangle ABC with semiperimeter p is given. Points E and F lie on the line AB and CE = CF = p. Prove that the excircle k1 of 4ABC to the side AB touches the circumcircle k of 4EFC . Solution. Let P and Q be the tangent points of k1 with the lines CA and CB, respectively. Since CP = CQ = p, then the points E , P , Q and F lie on the circle with center C and radius p. We denote by i the inversion de ned by this circle. Since i(P ) = P , i(Q) = Q, then i(k1) = k1. On the other hand i(E ) = E and i(F ) = F . Hence i(k) is the line AB . But k1 touches AB and thus k touches k1. Problem 3. Two players A and B take stones one after the other from a heap with n 2 stones. A begins the game and takes at least 1 stone but no more then n ; 1 stones. Each player on his turn must take at least 1 stone but no more than the other player has taken before him. The player who takes the last stone is the winner. Find who of the players has a winning strategy. Solution. Consider the pair (m l), where m is the number of the stones in the heap and l is the maximal number of stones that could be taken by the player on turn. We must nd for which n the position (n n ; 1) is winning (i.e. A wins) and for which n it is losing (B wins). We shall apply the following assertion several times: If (m l) is a losing position and l1 < l, then (m l1) is losing too. Now we shall prove that (n n ; 1) is a losing position i n is a power of 2. Su ciency : Let n = 2k , k 1. If k = 1 then B wins on his rst move. Assume that (2k 2k ; 1) is a losing position and let consider the position (2k+1 2k+1 ; 1). If A takes at least 2k stones on his rst move, then B wins at ones. Let A take l stones, where 1 l < 2k . By the inductive assumption B could play in such a way that he could win the game (2k l) since l 2k ; 1 the last move will be the move of B. After this move we get the position (2k m) with m l, which is losing for A, according to the inductive assumption. Necessity: It is enough to prove that if n is not a power of 2, then (n n ; 1) is a winning k position. Let n = 2 + r, where 1 r 2k ; 1. On his rst move A takes r stones and B is faced to the position (2k r), which is losing for B. 1

Problem 1. Find the number of all integers n > 1, for which the number a25 ; a is divisible

Problem 4. The points C1, A1 and B1 lie on the sides AB, BC and CA of the equilateral triangle ABC respectively in such a way that the inradii of the triangles C1AB1 , B1 CA1, A1 BC1 and A1 B1 C1 are equal. Prove that A1 , B1 and C1 are the midpoints of the corresponding sides. Solution. We shall prove that BA1 = CB1 = AC1 (Figure Figure 1. 1). Assume the contrary and let BA1 CB1 > AC1 . Let be the rotation at 1200 which center coincides with the in center of the incircle of 4ABC . This rotation transforms the incircles of the triangles C1BA1 , A1CB1 and B1 AC1 to the incircles of the triangles A1CB1 , B1 AC1 and C1BA1 , respectively. Let A2 = (A1), B2 = (B1) and C2 = (C1). It follows that BB2 < BC1 and BC2 < BA1 . But the incircles of the triangles BC1 A1 and BC2B2 have equal radii (because (4AC1B1 ) = 4BC2B2 ), which is a contradiction. Let r be the radius of the incircles of the triangles C1 AB1 , B1CA1, A1 BC1p and A1 B1 C1. From the triangle B1 AC1 we have 1 ; B C 3 , and from 4A B C which is equilateral we 1 1 : r= 1 1 1 2 3 p have r = B1 C1 : 63 . From here B1 C1 = 1 2 and consequently A1, B1 , C1 are midpoints of the corresponding sides. Problem 5. Let A = f1 2 : : : m + ng, where m and n are positive integers and let the function f : A ! A be de ned by the equations:
f (i) = i + 1 for i = 1 2 : : : m ; 1 m + 1 : : : m + n ; 1 f (m) = 1 and f (m + n) = m + 1: a) Prove that if m and n are odd then there exists a function g : A ! A such that g (g (a)) = f (a) for all a 2 A. b) Prove that if m is even then m = n i there exists a function g : A ! A such that g (g (a)) = f (a) for all a 2 A. Solution. a) Let m = 2p +1, n = 2q +1 and g(i) = p + i +1 for i = 1 2 : : : p g(i) = q + i +1 for i = m + 1 m + 2 : : : m + q g (2p + 1) = p + 1 g (p + 1) = 1 g (m + 2q + 1) = m + q + 1 g (m + q + 1) = m + 1. It is easy to check that g (g (a)) = f (a) for all a 2 A. b) Let m = n and g (i) = m + i for i = 1 2 : : : m g (m + i) = i + 1 for i = 1 2 : : : m ; 1 g(2m) = 1. For the converse let M = f1 2 : : : mg. It follows by the de nition of f that the elements of M remain in M after applying the powers of f with respect to superposition. Moreover, these powers scoop out the whole M . The same is true for the set A n M . The function f is bijective in T A and if there exists g verifying the condition, then g is bijective too. We shall prove that g (M ) M = . It follows from the contrary that there exists i 2 M such that g (i) 2 M . Consider the sequence i g (i) g 2(i) : : : and the subsequence i f (i) f 2(i) : : :. It is easy to see that g(M ) = M . We deduce that there exists a permutation a1 a2 : : : am of elements of M , such that g (ai) = ai+1 for i = 1 2 : : : m ; 1 g (am ) = a1 and f (a2i;1) = a2i+1 for i = 1 2 : : : s ; 1 f (a2s;1 ) = a1 , whereT m = 2s. The last contradicts to the properties of f which were mentioned already. Thus g (M ) M = . Analogously g (A n M ) = A n M , if g (i) 2 A n M for i 2 A n M . At last let us observe that when starting from an element of M and applying g we go to A n M , but when applying g for a second time we go back to M . The same is true for the set A n M .
2

From here and from the bijectivity of g it follows that M and A n M have one and the same number of elements, i.e. n = m. n yn Problem 6. Let x and y be di erent real numbers such that xx ; ; y is an integer for some

n ; yn four consecutive positive integers n. Prove that xx ; y is integer for all positive integers n. n n ; y . Then t + b:t + c:t = 0 for b = ;(x + y), c = xy, where Solution. Let tn = xx ; n+2 n+1 n y t0 = 0, t1 = 1. We shall show that b c 2 Z . Let tn 2 Zfor n = m m + 1 m + 2 m + 3. Since cn = (xy)n = t2 ; t :t 2 Z when n = m m + 1, then cm cm+1 2 Z . Therefore c is rational n n +2 n+1 m +1 and from c 2 Zit follows that c 2 Z . On the other hand

i.e. b is rational. From the recurrence equation it follows by induction that tn could be represented in the following way tn = fn;1 (b), where fn;1 (X ) is a monic polynomial with integer coe cients and degfn;1 = n ; 1. Since b is a root of the equation fm (X ) = tm+1 , then b 2 Z . Now from the recurrence equation it follows that tn 2 Zfor all n.

tm+1 tm+2 b = tm tm+3 ; m c

BULGARIAN NATIONAL OLYMPIAD IN MATHEMATICS


Third round 1996

7x2 + y 2 = 2 : Solution. If n = 3 we have x3 = y3 = 1. Suppose that for an integer n 3 there are odd positive integers x , y , such that 7x2 + y 2 = 2 . We shall prove that for each pair j7x ; y j and X = jx ; y j Y = 7x + y y X=x + 2 Y= 2 2 2 we have 7X 2 + Y 2 = 2 +1 . Indeed, 2 2 7 x 2 y + 7x 2 y = 2 7x2 + y 2 = 2 2 = 2 +1 : Since x and y are odd, i.e. x = 2k + 1 and y = 2l + 1 (k, l are integers), then x + y = k + l + 1 and jx ; y j = jk ; lj, which shows that one of the numbers x + y 2 2 2 j x ; y j and is odd. Thus, for n + 1 there are odd natural numbers x +1 and y +1 with the 2 required property. Problem 2. The circles k1 and k2 with centers O1 and O2 respectively are externally tangent at the point C , while the circle k with center O is externally tangent to k1 and k2 . Let ` be the common tangent of k1 and k2 at the point C and let AB be the diameter of k, which is perpendicular to `, the points A and O1 lie in one and the same semiplane with respect to the line `. Prove that the lines AO2, BO1 and ` have a common point. Figure 1. Figure 2.
n n n n n n n n n n n n n n n n n n n n n n n n n n n n n n n n n n n n n n n

x and y , such that


n n

Problem 1. Prove that for all positive integers n 3 there exist an odd positive integers

Solution. Denote by r, r1 and r2 the radii of k, k1 and k2, by M and N the tangent points of k with k1 and k2, respectively and by P the common point of ` and AB (Figure 1). It follows from O1 O2 ? ` and AB ? ` that 4BON 4CO2N . Then CNO2 = ONB , BO = r . Analogously, and consequently the points C , N and B are colinear. Also, BN = CN CO r
6 6

AM = r . A, M and C are colinear and MC r1 The lines AN , BM and ` have a common point H , which is the altitude center of 4ABC . By Ceva's theorem we have: AP BN CM = AP r r1 = 1, from where: PB NC MA PB r2 r r1 = r2 : (1) PB AP Let now D1 and D2 be the common points of the line ` with the lines BO1 and AO2, CD1 = r1 . Analogously, respectively. Obviously, 4O1CD1 4BPD1 (Figure 2), from where D PB 1P CD2 = r2 and according to (1) we have CD1 = CD2 , which shows that D D . Thus, the 1 2 D2P AP D1 P D2P lines AO2, BO1 and ` have a common point. Problem 3. a) Find the maximal value of the function y = 4x3 ; 3x in the interval ;1 1]. b)Let a, b and c be real numbers and M be the maximal value of the function y = 4x3 + ax2 + bx + c in the interval ;1 1]. Prove that M 1. For which a, b, c is the equality
reached? Figure 3. 12 x ; 1 2
Solution.

a) Using that 4x3 ; 3x = 12x2 ; 3 = 1 , we nd that the function x+ 2


0

y = 4x3 ; 3x
has a local maximums when x = 1 2 . Then its maximal value in the interval ;1 1] is the biggest among the numbers y (;1), y (1), y ; 1 and y 1 (Figure 3). -1 -1/2 0 1/2 1 2 2 1 1 But y (;1) = y ; 2 = y 2 = y (1) = 1, thus the maximal value is equal to 1. b) Let f (x) = 4x3 + ax2 + bx + c. Assume that there exist numbers a, b, c, for which the maximal value M of the function y = jf (x)j in ;1 1] is less than 1, i.e. M < 1. Then ;1 < f (x) < 1 for all x 2 ;1 1]. ; Consider the function g (x) = f (x) ; 4x3 ; 3x = ax2 + (b + 3)x + c. We have g (;1) > 0, 1 > 0 and g (1) < 0. Consequently g (x) changes its sign at least 3 times, g ;1 < 0, g 2 2 which means that the quadratic equation ax2 + (b + 3)x + c = 0 has at least 3 di erent roots. This is possible only if a = b + 3 = c = 0, i.e. if f (x) = 4x3 ; 3x. According to a) the maximal value of y = 4x3 ; 3x is 1. The equality M = 1 is reached only when a = 0, b = ;3 and c = 0. Problem 4. The real numbers a1 a2 : : : a (n 3) form an arithmetic progression. There exists a permutation a 1 a 2 : : : a n of the same numbers, which is a geometric progression.
r r r r n i i i

Find the numbers a1 a2 : : : a , if they are pairwise di erent and the biggest among them is equal to 1996. Solution. Let a1 < a2 < < a = 1996 and q be the quotient of the geometric progression a 1 a 2 : : : a n . We have q 6= 0 and q 6= 1. The numbers a n a n;1 : : : a 1 form also a geometric 1 . Thus, we can assume that jq j > 1, i.e. q > 1 or q < ;1. Then progression which quotient is q ja 1 j < ja 2 j < < ja n j, from where a 6= 0 for all i. More exactly, either all numbers are positive (q > 1) and then a 1 < a 2 < < a n , which together with a1 < a2 < < a shows that a k = a , i.e. the numbers a1 a2 : : : a form an arithmetic as well as a geometric progression, or the numbers a 1 a 2 : : : a n change their signs alternatively (q < ;1) and then the positive ones form an increasing geometric progression with quotient q 2 , and the order is the same as in the arithmetic progression. (The numbers a1 a2 : : : a could not be all negative, because a = 1996 > 0.) Assume now that 3 among the numbers a1 a2 : : : a are positive. Then 0 < a 2 < a 1 < a and they form a geometric as well as an arithmetic progression. Therefore 2a 1 = a 2 + a and a2 1 = a 2 a . From here a 2 = a 1 = a , which is a contradiction. Thus at most two among the numbers are positive. Analogously, at most two among the numbers are negative. Consequently, n 4. Let n = 4. Then a1 < a2 < 0 < a3 < a4 and 2a2 = a1 + a3 , 2a3 = a2 + a4 . But q < ;1 and the geometric progression is either a3 a2 a4 a1 or a2 a3 a1 a4. Let it be a3 a2 a4 a1. Then a2 = a3q, a4 = a3q2 and a1 = a3 q 3 . Thus, 2a3q = a3q3 + a3 and 2a3 = a3q + a3q2 . From here q = 1, which contradicts to q < ;1. So n = 3. There are two possibilities: I. a1 < a2 < 0 < a3 = 1996. Then the geometric progression is a2, a3 = a2q, a1 = a2q2. It follows from 2a2 = a1 + a3 that 2a2 = a2q 2 + a2 q , i.e. q 2 + q ; 2 = 0. Thus, q = ;2, ;2a2 = 1996, a2 = ;998 and the numbers are (;3992 ;998 1996). II. a1 < 0 < a2 < a3 = 1996. Now the geometric progression is a2, a1 = a2q, a3 = a2q2. From 2a2 = a1 + a3 we obtain 2a2 = a2 q + a2 q 2, i.e. again q = ;2. Therefore, a3 = 4a2 = 1996 and a2 = 499. The numbers are (;998 499 1996). Problem 5. A convex quadrilateral ABCD, for which ABC + BCD < 180 , is given. The common point of the lines AB and CD is E . Prove that ABC = ADC if and only if AC 2 = CD CE ; AB AE: Solution. Let ABC = , ADC = , BAC = ' and CAD = (Figure 4). The point A is between E and B and the point D is between E and C . Also, AEC = + ' + ; 180 . Applying the sine theorem to the trianFigure 4. gles ACD, ACE , ABC and again to 4ACE , we obtain: CD = sin AC sin sin ' CE = ; AC sin ( + ' + ) AB = sin ( + ') AC sin AE = ; sin ( + ) : AC sin ( + ' + )
n n i i i i i i i i i i i i i n i k n i i i n n n n; n; n n; n; n n; n; n n; n; n
6 6 6 6 6 6 6 6 6

parallel to its sides. In how many ways could two of the squares be canceled and the remaining part be covered with dominoes 2 1? Solution. Denote by F (m n) the number we are looking for. Since every domino covers exactly two squares, then F (m n) = 0 if m and n are odd. Let at least one of the numbers m and n be even. We color the Figure 5. squares in two colors | white and black in such a way that every two neighbor squares (with common side) are of di erent colors (Figure 5). The number S0 of the white squares is equal to the number S1 of the black ones and S0 = S1 = mn 2 . Each domino covers one white and one black square. If two white or two black squares are canceled, then it is impossible to cover by dominoes the remaining part of the rectangle. Now we shall show that if one white and one black squares are canceled then the remaining part can be covered by dominoes. Since mn is even, then mn = 2t, where t 2. We make induction with respect to t. The case t = 2 is obvious. Let t0 > 2 and the proposition is true for all 2 t t0. Let mn = 2 (t0 + 1). Denote by T1 the rectangle consisted of the rst two rows of the considered rectangle, and by T2 | the rectangle consisted of the remaining rows. If the two canceled squares are in T1 or in T2, then we can cover each of the rectangles by dominoes and consequently we can cover the given rectangle. Let one of the canceled squares be in T1 , and the other | in T2. We place a domino in such a way that it covers one square from T1 and one square from T2. It is also possible that the canceled square from T1 and the covered square from T1 are of di erent colors. Thus, the canceled square from T2 and the covered square from T2 are of di erent colors. According to the inductive assumption the remaining part of T1 and the remaining part of T2 can be covered. Thus, the rectangle can be covered too. 2 Finally one white and one black squares can be chosen in S0 S1 = mn ways. Therefore, 2 2 n2 m in this case F (m n) = 4 . 4

CD CE ; AB AE ; 1 = 0 AC AC AC AC sin ( + ') sin ( + ) sin ; sin sin ' sin ; sin sin sin (' + + ) = 0 (cos ( + ' ; ; ) ; cos ( + ' + + )) sin ; (cos (' ; ) ; cos (' + )) sin ; (cos ( ; ) ; cos ( + )) sin (' + + ) = 0 sin ( + ' + ) ; sin (2 + ' + ; ) + sin (2 + ; ; ') ; sin ( + ; ') = 0 sin ( ; ) cos ( + ' + ) + sin ( ; ) cos ( + ; ') = 0 sin ( ; ) sin ( + )sin ' = 0 But sin ' 6= 0 and sin ( + ) = 6 0. Consequently, sin ( ; ) = 0 and = . Problem 6. A rectangle m n (m > 1, n > 1) is divided into mn squares 1 1 with lines,

From here the equation AC 2 = CD CE ; AB AE is equivalent to the equations:

BULGARIAN NATIONAL OLYMPIAD IN MATHEMATICS


Fourth round 1996
p p)(5q ; 2q ) Problem 1. Find all primary numbers p and q, for which (5 ; 2 pq is an integer. Solution. Let p be prime number and pj (5p ; 2p ). It follows by the Fermat theorem that p 5 ; 2p 3 (mod p). Consequently p = 3. p p q q Let now p and q be such prime numbers that (5 ; 2 )(5 ; 2 ) is an integer. If pj (5p ; 2p ),

then p = 3. Since 53 ; 23 = 3 3 13, then either q j (5q ; 2q ), i.e. q = 3, or q = 13. Therefore the pairs (3,3), (3,13), (13,3) satisfy the problem condition. It remains the case when p 6= 3 q 6= 3. Now pj (5q ; 2q ) and q j (5p ; 2p ). We can assume that p > q . It is clear that (p q ; 1) = 1 and consequently, there are positive integers a and b, for which ap ; b(q ; 1) = 1 (Bezou theorem). Since (q 5) = (q 2) = 1, it follows by the Fermat theorem that 5q;1 2q;1 (mod q ). From 5p 2p( (mod q ) we deduce that 5ap 2ap (mod q ) and therefore, 5b(q;1)+1 2b(q;1)+1 (mod q ). But 5b(q;1)+1 5 (mod q ) and 2b(q;1)+1 2 (mod q ). Thus, q = 3, which is a contradiction. Finally, (p q ) = (3 3) (3 13) (13 3). Problem 2. Find the side length of the smallest equilateral triangle in which three disks with radii 2, 3 and 4 without common inner points can be placed. Solution. Let in a equilateral 4ABC two disks with radii 3 and 4 without common inner points be placed. It is clear that a line ` exists, which separates them, i.e. the disks are in di erent semiplanes with respect to ` (Figure 1). Figure 1. Figure 2.

pq

This line divides the triangle into a triangle and a quadrilateral or into two triangles. In both cases the disks can be replaced in the gure in a way that each of them is tangent to two of the sides of 4ABC . It is clear that the new disks have no common inner point. Let the disks be inscribed in 6 A and 6 B of 4ABC , respectively. We translate the side BC parallelly to itself 1

towards the point A, till the disk which is inscribed in 6 B touches the disk which is inscribed in 6 A (Figure 2). Thus, we get an equilateral 4A1B1 C1 with a smaller sides, in which two disks with radii 3 and 4 and without common inner points are placed. Let A1 B1 = x, I be the incenter of 4A1B1 C1, while O1 Figure 3. and O2 be the centers of the two disks. Then A1I = B1 I = x p A1O1 = 6 B1O2 = 8. Since the disk with radius 4 is 3 inside thep4A1B1 C1, then O2 2 IB1 . Thus, B1 O2 B1 I , x ;6 O I = p x; i.e. x 8 3. On the other hand O1I = p 2 3 3 8 O1O2 = 7 and by the cosine theorem for 4O1O2 I we nd x ; 8 2+ p x ;6 p x ; 8 = 49. x ; 6 2+ p that p 3p 3 3 3 p 3, and from here x = 11 3. Consequently, AB But x 8 p 11 3. On the other hand in the equilateral 4 ABC with p side length 11 3 three disks with radii 2, 3 and 4 (without common inner points) can be placed inscribing circles with these radii in the angles of the triangle (Figure 3). Note that the disks wit radii 3 and 4 are tangent pto each other. It follows from the above considerations that the solution of the problem is 11 3. Problem 3. The quadratic functions f (x) and g(x) are with real coe cients and have the following property: if the number g (x) is integer for a positive x, then the number f (x) is integer too. Prove that there are such integers m and n, that f (x) = mg (x) + n for all real x. q q2 Solution. Let g (x) = px2 + qx + r. We can assume that p > 0. Since g (x) = p(x + )2 + r ; 2p 4p q after the variable change of x by x + 2p we reduce the problem for the following quadratic 2 functions f (x) = ax2 + bx + c and k be such s s sk > s and s !g (x) = px + s p > 0. Let ! an integer that k ; s > q . Since g k ; s = k is integer, then f k ; s = a(k ; s) + b k ; s + c p 2p p p p p is an integer too. Consequently, the number s s ! ! k + 1 ; s k ; s b :p 1 p f ; f =p (1) +a p p p k+1;s+ k;s p is an integer for all k which are su ciently big. It follows from here that a is an integer. Indeed, p suppose that a is not an integer. If b > 0, we chose k in a way that

and if b < 0, we chose k in a way that

b :p 1 p a +1; a < p p k+1;s+ k;1 p p a ; a: b :p 1 p > p p k+1;s+ k;1 p p

In both cases there is a contradiction with the fact that (1) is an integer. Now, it follows that b :p 1 p p p k+1;s+ k;s 2

for all k, which are su ciently s big. ! This is possible only when b = 0. a k ; s Let = m. Then, f p p = m(k ; s) + c is an integer (when k is su ciently big), i.e. c ; ms is an integer. Let n = c ; ms. Now it is clear that f (x) = mg(x) + n for all x. Problem 4. The sequence fan g1 n=1 is de ned by

n n a1 = 1 an+1 = a n +a

n 1:

n ; 1 = (n ; 1)2 + p n2 (n ; 2) < pn + 2 an+1 = f (an ) < f p n;2 (n ; 1)n n ; 2 when n p 4. (The last inequality is equivalent to 2n2(n ; 3) + 4n ; 1 > 0.) Therefore, pn a 4, i.e. ba2 n < n + 1 when n n c = n. Problem 5. The quadrilateral ABCD is inscribed in a circle. The lines AB and CD meet each other in the point E , while the diagonals AC and BD | in the point F . The circumcircles of the triangles AFD and BFC have a second common point, which is denoted by H . Prove that 6 EHF = 90 . Solution. Let O be the circumcenter of ABCD. We shall prove that O is the second common point of the circumcircles of 4AHB and 4CHD. (Since AB and CD are not parallel, then O= 6 H .) After that we shall prove that the points E , H and O are colinear and 6 OHF = 90 . We shall consider the possible positions of H . Let G be the common point of AD and BC (these lines are not parallel because the circumcircles of 4AFD and 4BFC are not tangent). It is clear that H is in the interior of 6 AGB . 1) H is in 4CGD. Then, 6 CHD = 6 CHF + 6 DHF = 180 ; 6 CBF + 180 ; 6 DAF = d > 180 (from CD d < AB d and AB d + CD d < 360 ), which is impossible. 360 ; CD d= 2) H is in 4CFD. Then, 6 CHD = 360 ; 6 CHF ; 6 DHF = 6 CBF + 6 DAF = CD d = 6 AOB . 6 COD and 6 AHB = 6 AHF + 6 BHF = 6 ADF + 6 BCF = AB 3) H is in 4ABF . Analogously, 6 CHD = 6 COD and 6 AHB = 6 AOB . 4) H is on AB . Again 6 CHD = 6 COD and 180 = 6 AHB = 6 AOB . Consequently, O is the midpoint of AB . 5) H is not in 4ABG. Then 6 CHD = 6 COD and 6 AHB = 360 ; 6 AOB .
3

Prove that ba2 4 (it is denoted by bxc the integer part of the number x). n c = n when n x n (a ; b)(ab ; n2 ) , it follows that the Solution. Let f (x) = + . Since f (a) ; f (b) = n x abn function f (x) is decreasing in the interval (0 n). p Firstly, by induction we shall prove that n an p n when n 3. We have n ; 1n p p 3 a1 = 1 a2 = 2 and a3 = 2, i.e. 3 a3 p2 . Let n an p for an integer n 3. n;1 p + 1 and a = f (a ) f p n n > pn + 1 p Then, an+1 = f (an ) f ( n) = np = n+1 n n n;1 n;1 and thus the induction nishes. pn, it remains to prove, that a < pn + 1. We have a = f (a ) Since an n n+1 n n n n ; 1 f p =p when n 3. Consequently, an p when n 4. Then, n;1 n;1 n;2

Note that in the cases 2), 3) and 5) the points O and H are in one and the same semiplane with respect to the line AB and with respect to the line CD. Consequently, the points A B H and O are concyclic. We have the same for the points C D H and O. Now we shall prove that the line EH passes through the point O. Let this line meet the circumcircles of 4AHB and 4CHD at the points O1 and O2 , respectively. Since these points lie on the ray EH ! , it follows from the equalities EH:EO1 = EA:EB = EC:ED = EH:EO2 that O1 O2 O. The points O and H are in one and the same semiplane with respect to CD and we can assume that the quadrilateral COHD is convex. Let H be in 4CFD. Then, 6 OHF = 6 FHC ; 1 :6 6 6 OHC = 180 ; 6 FBC ; 6 ODC = 180 ; 2 COD ; (90 ; 1 2 : COD) = 90 . The other possibility for H is to be inside 6 AGB and outside the non-convex quadrilateral AFBG. Hence 6 OHF = 6 OHC + 6 FHC = 6 ODC + 6 FBC = 90 . Problem 6. A square table of size 7 7 with the four corner squares deleted is given. a) What is the smallest number of squares which need to be colored black Figure 4. so that a 5-square entirely uncolored Greek cross (Figure 4) cannot be found on the table? b) Prove that it is possible to writte integers in each square in a way that the sum of the integers in each Greek cross is negative while the sum of all integers in the square table is positive. Solution. Denote the square in row i and column j by (i j ). Note that a cross is uniquely determined by its central cell. The cross with central cell (i j ) is denoted by Cij , 2 i j 6. The number of all crosses is 25. a) The squares (1 i), (i 1), (7 i), (i 7), i = 2 : : : 6, are included in exactly one cross the squares (2,2), (2,6), (6,2), (6,6) are included in exactly 3 crosses the squares (2 i), (i 2), (6 i), (i 6), i = 3 4 5 | in exactly 4 crosses and nally, the squares (i j ) 3 i j 5 are included in exactly 5 crosses. Since C22 contains a colored square, then at least one of (1,2), (2,1), (2,2), (2,3), (3,2) should be colored. Similarly, at least one of (2,7), (1,6), (2,6), (2,5), (3,6), at least one of (7,2), (6,1), (6,2), (5,2), (6,3) and at least one of (6,7), (7,6), (6,6), (6,5), (5,6) are colored. Any of them is contained in at most four crosses (the rst two in each quintuple in 1 cross, the third one in 3 crosses and the remaining two | in 4 crosses). Denote by x the number of the colored squares among (i j ) 3 i j 5. The number of crosses containing a colored square is not greater than 4:4 + 5x, whence 16 + 5x 25: (1) Thus, x 2 and the number of the colored cells is at least 6. Suppose that the number of the colored squares is 6. Then x = 2. Moreover, acording (1) there exists at most one Greek cross containing more than one colored square. If two squares with a common side or a common vertex are colored, then it is easy to check that there are two crosses with at least two colored squares each. Therefore, the squares (4,4), (3,3), (3,5), (5,3), (5,5) should be uncolored. With no loss of generality, let the two colored squares be (4,3) and (4,5). But now the crosses C34 and C54 do not contain colored squares. To prove that the minimal number of colored squares is 7, color for example the squares (2,5), (3,2), (3,3), (4,6), (5,4), (6,2), (6,5). b) Let us write ;5 in each of the colored squares from a) and 1 in the remaining squares. Since every Greek cross contains a colored square, the sum of the numbers in its squares does not exceed 1+1+1+1;5 = ;1 < 0. The sum of all numbers in the table is 7:(;5)+(45;7):1 = 3 > 0 and we are done. 4

XLV National Mathematics Olympiad: 3rd round, April 1997


Problem 1. Find all natural numbers a, b and c such that the
roots of the equation

are natural numbers.

x2 ; 2ax + b = 0 x2 ; 2bx + c = 0 x2 ; 2cx + a = 0

rst, second and third equation respectively, and let all of them be natural numbers. Assume that x 2 for all i = 1 2 : : : 6. Then 2a = x1 + x2 x1x2 = b, 2b = x3 + x4 x3x4 = c and 2c = x5 + x6 x5x6 = a. Thus 2(a + b + c) a + b + c, which is impossible, since a b c are natural numbers.
i

Solution: Let fx1 x2g, fx3 x4g and fx5 x6g be the roots of the

Therefore at least one of the numbers x equals 1. Without loss of generality suppose x1 = 1, so 1 ; 2a + b = 0. If x 2 for i = 3 4 5 6, then
i i

2(b + c) = (x3 + x4) + (x5 + x6) x3x4 + x5x6 = c + a whence 2(2a ; 1+ c) c + a ) c 2 ; 3a, which is impossible when a b c are natural numbers. So at least one of x3 x4 x5 x6 equals 1. Let x3 = 1. Now 1 ; 2b + c = 0. Assuming that x5 2 and x6 2, we get 2c = x5 + x6 a, 1 . Thus 7b 5, a contradiction. so 2(2b ; 1) b + 2 Therefore at least one of the numbers x5 x6 is 1 and it follows that 1 ; 2c + a = 0. Further 0 = (1 ; 2a + b) + (1 ; 2b + c) + (1 ; 2c + a) = 3 ; (a + b + c) and since a b c are natural numbers, it follows that a = b = c = 1. Direct veri cation shows that a = b = c = 1 satisfy the conditions of the problem.

Problem 2. Given a convex quadrilateral ABCD which can be


inscribed in a circle. Let F be the intersecting point of diagonals AC and BD and E be the intersecting point of the lines AD and BC . If M and N are the midpoints of AB and CD, prove that MN = 1 AB ; CD : EF 2 CD AB 2

;! ! Solution: Let 6 AEB = , EC = c, ED = d, ; i =1 c EC and 1 ;! AB = ; ! j =d ED. Since ABCD is an inscribed quadrilateral, CD AE = BE = k. Therefore ;! ;! ; ! ; ! EA = kc j and EB = kd i . Since CE DE F 2 AC and F 2 BD there exist x and y such that ;! ! ! EF = x;! EA + (1 ; x);! EC = xkc; j + (1 ; x)c; i
and

Therefore and thus


2

;! ! ! EF = y;! EB + (1 ; y);! ED = ykd; i + (1 ; y)d; j: ! ! Comparing the coe cients of ; i and ; j in these equalities gives kd ; c . xkc = (1 ; y)d and ykd = (1 ; x)c. This implies x = (k 2 ; 1)c ;! ; ! ; ! EF = k2 k ( kd ; c ) j + ( kc ; d ) i ;1
k

EF = k2 ; 1

!2

(kd ; c)2 + (kc ; d)2 + 2(kd ; c)(kc ; d) cos

and it follows that 2 2 MN 2 = 1 4 (d ; kc) + (c ; kd) + 2(d ; kc)(c ; kd) cos : 3

On the other hand 1 ;! ;! ;! ;! ;! ; ;! = 1 ;! AD + BC = MN 2 2 ED ; EA + EC ; EB ; ! ; ! = 1 2 (d ; kc) j + (c ; kd) i

! 2 2;1 2 1 k 1 k; 1 MN Therefore EF 2 = 4 k =4 k MN = 1 AB ; CD EF 2 CD AB

and so

Problem 3. Prove that the equation


x2 + y2 + z2 + 3(x + y + z) + 5 = 0 has no solution in rational numbers.

Solution: It is easy to see that the equation is equivalent to


(2x + 3)2 + (2y + 3)2 + (2z + 3)2 = 7: It has a solution in rational numbers if and only if there exist integer numbers a, b, c and a natural number m such that () a2 + b2 + c2 = 7m2: Suppose such numbers exist and give m its smallest possible value. There are two cases: (I) m = 2n is an even number. Now a2 + b2 + c2 is divisible by 4. This implies that all numbers a, b, c are even ones, so a = 2a1, 2 2 2 b = 2b1, c = c1. It follows now that a2 1 + b1 + c1 = 7n , which contradicts the way m have been chosen. (II) m = 2n + 1 is an odd number. Now m2 1 (mod 8) and therefore a2 + b2 + c2 7 (mod 8), which is impossible when a, b, c are integer. 4

Problem 4. Find all continuous functions f (x) de ned in the set


of real numbers and such that for all real x. 1 f (x) = f x2 + 4

Solution: Let f (x) be a function satisfying the conditions of the


problem. Obviously f (x) is an even function. Let x0 0. There are two cases: (I) 0 x0 1 2 . Consider the sequence (1)

1 for all n. Moreover 2 1= x ;1 2 0 x +1 ; x = x2 ; x + 4 2 which implies that (1) is a monotone increasing function. Since it is bounded, it follows that it is a convergent function. Let lim !1 x = . 1. Now 2 ; + 1 = 0, so = 4 2 On the other hand, since f (x) is a continuous function, 1 : lim f ( x ) = f !1 2 5
n n n n n n n n n n

x0 x1 : : : x : : : 1. de ned by the equalities x +1 = x2 + 4 It is easy to see by induction that 0 x


n n n

But

1 for all n. Thus f (x0) = f (x1) = which means that f (x0) = f 2 1. for all x0 2 0 2 (II) x0 > 1 2 . Consider the following sequence:

1 = f (x ) f (x +1) = f x2 + 4
n n n

s de ned by x +1 = x ; 1 4. As in the previous case, we show that (2) is a convergent function 1 1 and lim !1 x = 2 . Further lim !1 f (x ) = f 2 and since
n n n n n n

(2)

x0 x1 : : : x : : :
n

1 . for all n we get that f (x0) = f 2 Therefore f (x) is a constant function in the interval 0 +1), and since it is even, it is a constant for all x. Conversely, any constant function satis es the conditions of the problem.

1 = f (x ) f (x +1 ) = f x2 +1 + 4
n n n

Problem 5. Two squares K1 and K2 with centres M and N and


6

sides of length 1 are placed in the plane in a way that MN = 4, two of the sides of K1 are parallel to MN and one of the diagonals of

K2 lies on the line MN . Find the locus of midpoints of segments XY where X is an interior point for K1 and Y is an interior point for K2.

Solution: The locus is the interior of a regular hexagon centred at


1. the midpoint of the segment MN and with a side length of 2

To prove this, proceed as follows. Fix the point Y in the interior of K2. When X varies in the interior of K1 the locus of the midpoints of XY is a square K10 which 1. is homothetic to K1 by homothecy with centre Y and coe cient 2 1 and its centre is the midpoint Obviously the side of this square is 2 of MY . When Y varies in the interior of K2 then the locus of the midpoints of MY is a square K20 which is homothetic to K2 by 1 . The side of this square homothecy with centre M and coe cient 2 equals 1 and its centre Q is the midpoint of MN . Finally, when 2 0 , the squares vary in the the centres of K10 vary in the interior of K2 interior of a regular hexagon centred at Q and with a side length of 1 2. 7

Problem 6. Find the number of non-empty sets of S = f1 2 : : : ng


such that there are no two consecutive numbers in one and the same set.
n

f1 = 1, f2 = 2, f3 = 4. Divide the subsets of S having no two consecutive numbers into two groups|those that do not contain the element n and those that do. Obviously the number of subsets in the rst group is f ;1. Let T be a set of the second group. Therefore either T = fng or T = fa1 : : : a ;1 ng, where k > 1. It is clear that a ;1 6= n ; 1, so fa1 : : :a ;1g S ;2 , whence the number of sets in the second group is f ;2 + 1. Therefore f = f ;1 + f ;2 + 1:
n n k k k n n n n n

Solution: Denote the required number by f . It is easy to see that


n

After substituting u = f + 1 we get


n n

u1 = 2 u2 = 3
n n

u = u ;1 + u ;2 :
n n n

Therefore the sequence fu g1=1 coincides with the Fibonacci sequence from its third number onwards. Thus we obtain 0 p ! +2 p ! +21 5 1 + 1 ; 1 ; 2 5 A ; 1: f =p @ 2 5
n n n

XLV National Mathematics Olympiad: 4th round, May 1997


Problem 1. Consider the polynomial ! ! ! n n n
2

k Pn (x) = 2 + 5 x + 8 x + + 3k n +2 x where n 2 is a natural number and k = n 3 2 .


; ;

(a) Prove that Pn+3 (x) = 3Pn+2 (x) 3Pn+1 (x) + (x + 1)Pn (x) (b) Find all integer numbers a such that Pn (a3) is divisible by n;1 ] 3 2 for all n 2.

Solution: (a) Compare the coe cients in front of xm, 0 m


; ;

n + 1 . It su ces to show that 3 ! ! ! ! ! n+3 =3 n+2 n + 1 n n 3m + 2 3m + 2 3 3m + 2 + 3m + 2 + 3m 1 :


1

1 = a + a we get that Using the identity a + b b b 1 ! !! ! !! n+3 n+2 n + 2 n + 1 2 + 3m + 2 ! 3m + 2 !! 3m + 2! 3m + 2 +1 n n + 3n = m!+ 2 3m + 2 3 m 1 ! ! ! n n + 2 n + 1 n = = 3m + 1 2 3m + 1 + 3m + 1 3 m 1 ! !! n + 2 n + 1 = + 3m + 1 ! 3m + 1 !! ! n+1 n n 3! m + 1 ! 3 m + 1 ! 3m 1 = 1 n n = n3+ m 3m 3m 1 = = 0:
; ; ; ; ; ; ; ; ; ; ; ; ; ; ; ; ; ;

(b) Suppose a satis es the condition of (b). Then P5(a3) = 10 + a3 is divisible by 9 and so a 1 (mod 3). On the contrary, 1 (mod 3). Now a3 + 1 0 (mod 9). Since P2(a3) = let a 1 P3(a3) = 3 and P4(a3) = 6 it follows by induction from (a) that n;1 Pn (a3) is divisible by 3 2 ] for any n. Therefore the required values of a are all integer numbers congruent to 2 modulo 3.
; ;

Problem 2. Let M be the centroid of ABC . Prove the inequality


4

sin 6 CAM + sin 6 CBM 2

2 3

(a) if the circumscribed circle of AMC is tangent to the line AB (b) for any ABC .
4 4

Let G be the midpoint of AB . (a) It follows from the conditions of the problem that c 2 = GA2 = GM GC = 1 m2 = 1 (2a2 + 2b2 c2) 2 3 c 12 and therefore a2 + b2 = 2c2. Using the median formula we get ma = 3 b and m = 3 a. Further b 2 2
; p p p

Solution: We use the standard notation for the elements of ABC .


4

1 + 1 = (a2 + b2) sin : A = sin 6 CAM + sin 6 CBM = S bm 3ab a amb 2 b2 , so a2 + b2 = From the Cosine Law a2 + b2 2ab cos = c2 = a + 2 2 2 4ab cos . Therefore A = sin 2 . 3 3 (b) There are two circles through C and M tangent to the line AB . Denote the contact points by A1 and B1 and let A1 GA!, B1 GB !. Since G is the midpoint of A1B1 and CM : MG = 2 : 1, M must be the centroid of A1B1C . Furthermore it is clear that 6 CAM 6 CA1M and 6 CBM 6 CB1M . Suppose 6 CA1M 90 and 6 CB1M 90 . Now sin 6 CAM + sin 6 CBM sin 6 CA1M + sin 6 CB1M 2 . 3
; p p 2 2 4 p

It remains to consider the case 6 CA1M > 90 , 6 CB1M 90 (the above angles could not both be obtuse). It follows from CA1M 2 that CM 2 > CA2 1 + A1M , so 1 (2b2 + 2a2 c2) > b2 + 1 (2b2 + 2c2 a2) 1 1 1 1 1 9 1 9 1 2 2 (a1 b1 c1 are sides of A1B1C ). We know from (a) that a2 1 + b1 = 2c1 2 2 and the above inequality becomes a1 > 7b1. Again from (a) we obtain v ! u 2 + b2 2 u b sin b a 1 1 1 t 1 1 = 1 sin 6 CB1M = 4a1b1 : a1 3 a1 3 b2 1 Substituting a 2 = x we get that 1 s 1 1 1 1= 1 sin 6 CB1M = 14x x2 1 < 2 49 7 4 3 4 3 since x < 1 7 . Therefore 1< 2 : sin 6 CAM + sin 6 CBM < 1 + sin 6 CB1M < 1 + 7 3
4 ; ; 4 p p ; p p ; ; p ; ; p

22:5 , = 112:5 , = 45 or = 112:5 , = 22:5 , = 45 .

Note: The inequality holds only for ABC with angles =


4

Problem 3. Let n and m be natural numbers such that m + i =

ai b2 i for i = 1 2 : : : n, where ai and bi are natural numbers and ai is not divisible by a square of a prime number. Find all n for which there exists an m such that a1 + a2 + + an = 12.
4

a perfect square, at most three of the numbers ai equal 1 (prove it!). It follows now from a1 + a2 + + an = 12 that n 7. We show now that the numbers ai are pairwise distinct. Assume 2 the contrary and let m + i = ab2 i and m + j = abj for some 1 i < 2 j n. Therefore 6 n 1 (m + j ) (m + i) = a(b2 j bi ). It is easy to see that the former is true only if (bi bj a) = (1 2 2) or (2 3 1) and in either case a1 + a2 + + an > 12. All possible values of ai are 1, 2, 3, 5, 6, 7, 10 and 11. There are three possibilities for n: n = 2 and a1 a2 = 1 11 2 10 5 7 n = 3 and a1 a2 a3 = 1 5 6 2 3 7 n = 4 and a1 a2 a3 a4 = 1 2 3 6 . Suppose n = 4 and a1 a2 a3 a4 = 1 2 3 6 . Now (6b1b2b3b4)2 = (m + 1)(m + 2)(m + 3)(m + 4) = (m2 + 5m + 5)2 1, which is impossible. Therefore n = 2 or n = 3. If n = 3 and (a1 a2 a3) = (1 5 6), then m = 3 has the required property, and if n = 2 and (a1 a2) = (11 1), then m = 98 has the required property. (It is not di cult to see that the remaining cases are not feasible.)
; ; ; f g f g f f g f g f g f g f g g f g f g f g ;

Solution: It is clear that n 12. Since ai = 1 if and only if m + i is

Problem 4. Let a, b and c be positive numbers such that abc = 1.


Prove the inequality 1 + 1 + 1 1+a+b 1+b+c 1+c+a 1 + 1 + 1 : 2+a 2+b 2+c

(abc = 1). It follows from Cauchy's Inequality that x 3 and y 3. Since both sides of the given inequality are symmetric functions of 5

1 + 1 + 1 = ab + bc + ca Solution: Let x = a + b + c and y = a b c

a, b and c, we transform the expression as a function of x, y and abc = 1. After simple calculations we get 3 + 4x + y + x2 12 + 4x + y 2x + y + x2 + xy 9 + 4x + 2y which is equivalent to 3x2y + xy2 + 6xy 5x2 y2 24x 3y 27 0: Write the last inequality in the form 5 x2y 5x2) + ( xy2 y2) + ( xy2 3y) + ( 4 x2y 12x) + (3 3 3 3 2 + ( xy 0: 3 3x) + (3xy 9x) + (3xy 27)
; ; ; ; ; ; ; ; ; ; ; ;

When x 3, y 3, all terms in the left hand side are non-negative and the inequality is true. Equality holds when x = 3, y = 3, which implies a = b = c = 1.

Problem 5. Given a ABC with bisectors BM and CN (M


4 2 4

AC , N AB ). The ray MN ! intersects the circumcircle of ABC at point D. Prove that 1 = 1 + 1 : BD AD CD


4

Solution: Let A1, B1 and C1 be the orthogonal projections of D


4

on the lines BC , CA and AB , respectively. It follows from DAB1 and the Sine Law that DB1 = DA sin 6 DAB1 = DA sin 6 DAC = DA DC (R is the circumradius of ABC ). Analogously DA = 1 2R 6

DB DC and DC = DA DB . Our equality is now equivalent to 1 2R 2R AD CD = BD CD + AD BD and so it su ces to prove that (1) DB1 = DA1 + DC1 Denote by m the distance from M to AB and BC and by n the DM = x (x > 1). Further, distance from N to AC and BC . Let MN DB1 = x, DC1 = x 1 and DA1 m = x. Therefore DB = nx, 1 n m n m DC1 = m(x 1) and DA1 = nx m(x 1) = DB1 DC1 and (1) holds.
; ; ; ; ; ; ;

n-tuples (a1 a2 : : : an) and (b1 b2 : : : bn) formed from distinct elements of X are called `disjoint' if distinct indices i and j exist such that ai = bj . Find the maximal number of n-tuples any two of which are `disjoint'.
ordered n-tuples such that any two of them are `disjoint'. Also let S (X ) be a set of such n-tuples for which S (X ) = A(n + 1). It is clear that for any X the following holds: (a1 a2 : : : an) S (X ) a1 = A(n): Thus A(n + 1) (n + 1)A(n). Therefore A(n + 1) (n + 1)n : : : A(3): 1)! . Direct veri cation shows that A(3) = 3, so A(n + 1) (n + 2 7
j j 2 jf 2 j gj

Problem 6. Let X be a set of n + 1 elements, n 2. Ordered

Solution: For n 2 denote by A(n + 1) the maximum number of

1)! by constructing a set of We prove now that A(n + 1) = (n + 2 (n + 1)! ordered n-tuples, any two of which are `disjoint'. We may 2 assume that X = 1 2 : : : n + 1 . Consider a set E of all even permutations of 1 2 : : : n + 1. (A permutation (a1 a2 : : : an+1) is called even if the number of pairs (i j ) such that i < j and ai > aj is an even number.) The set
f g f

(a1 a2 : : : an) (a1 a2 : : : an an+1) E


j 2

1)! ordered n-tuples, any two of which are `disjoint'. has (n + 2

XLVII National Mathematics Olympiad: 3rd round, 25{26 April 1998


Problem 1. Find the least positive integer number n (n
3) with the following property: for any colouring of n di erent points A1 A2 : : : An on a line and such that A1A2 = A2A3 = : : : = An;1An in two colours, there are three points Ai Aj A2j;i(1 i < 2j ; i n) which have the same colour.

Solution: Assume the two colours are white and black. Consider

8 points coloured as follows: A1 A2 A5 A6 (white), A3 A4 A7 A8 (black). Obviously no three points Ai Aj A2j;i (1 i < 2j ; i n) have the same colour and therefore n 9. If we can show that n = 9 has the required property, we will be done. Suppose there are 9 points coloured black or white and no three points Ai Aj A2j;i (1 i < 2j ; i n) have the same colour. First assume that for i = 3, i = 4 or i = 5 points Ai and Ai+2 have the same colour (say white). Then the points Ai;2 Ai+1 Ai+4 1

should be black (note that i ; 2 1 and i + 4 9), which is a contradiction. Suppose now that for i = 3 4 5 the points Ai and Ai+2 have di erent colours. Without loss of generality assume A5 is a white point. Then A3 and A7 are black. Because of the symmetry we may suppose that A4 is white and A6 is black. Consiquently A8 is white, A2 is black (2 + 8 = 2 5) and A9 is white (7 + 9 = 2 8). Therefore A1 should be both white (1 + 3 = 2 2) and black (1 + 9 = 2 5), which is again a contradiction. Consequently the assumption is not true and so n = 9.

Problem 2. Let ABCD be a quadrilateral such that AD = CD

and 6 DAB = 6 ABC < 90 . The line passing through D and the midpoint of the segment BC intersects the line AB at the point E . Prove that 6 BEC = 6 DAC .

meet at point N and AN and EC meet at point P . It follows from Menelaus' Theorem aplied to 4DMN and 4DEN that DP NC ME = PN CM ED and DA NB ME = AN BM ED. Combining the above equalities with AN = BN , BE = CE and AD = CD, we get DP NC = DC PN . Therefore CP is bisector of 6 DCN . 6 6 Consequently 6 ACP = 6 ACD + 6 DCP = 1 2 ( NDC + DCN ) = 6 NAB so 6 DCP = 6 CAB and we obtain 6 BEC = 6 ABC ; 6 BCE = 6 BAD ; 6 DCP = 6 DAC , Q. E. D. 2

Solution: Let M be the midpoint of BC and let AD and BC

is left out.

Note: The assertion is also true if the condition 6 ABC < 90

Problem 3. Let R+ be the set of all positive real numbers. Prove that there is no function f : R+ ! R+ such that
(f (x))2 f (x + y)(f (x) + y) for arbitrary positive real numbers x and y.

Solution: Suppose there exists a function satisfying the conditions


of the problem. Write the initial equality in the form f (x)y : f (x) ; f (x + y) f ( x) + y

1 for x > 0. Obviously First we prove that f (x) ; f (x + 1) 2 f is a (strictly) monotone non-increasing function. Fix x > 0 and choose a natural number n, such that n F (x + 1) 1. When k = 0 1 : : : n ; 1, we obtain that k)1 k ) ; f (x + k + 1 ) f (x + n 1: n f (x + n k 1 n f (x + n ) + n 2n 1 Adding the above inequalities gives f (x) ; f (x + 1) 2 . Let the natural number m be such that m 2f (x). Therefore m;1 f (x) ; f (x + m) = (f (x + i) ; f (x + i + 1)) m 2 f (x) i=0 and so f (x + m) 0. But this contradicts the fact that f is strictly positive.

Problem 4. Let f (x) = x3 ; 3x + 1. Find the number of di erent


real solutions of the equation f (f (x)) = 0.

the equation f (x) = 0 has three distinct roots x1 x2 x3 such that x1 < ;1 < x2 < 1 < x3 < 3. Therefore f (x) = x1 has only one real root (which is less than ;1) and f (x) = x2 and f (x) = x3 have three distinct real roots each (one in each of the intervals (;1 ;1) (;1 1) and (1 1)). Since the roots of f (f (x)) = 0 are exactly the roots of these three equations, we conclude that it has seven distinct real roots. with radius R. The inradii of the triangles ABC , ABD, AEC and AED are denoted by rABC , rABD , rAEC and rAED . Prove that b.) If rABC = rAED and rABD = rAEC , then 4ABC = 4AED. a.) cos 6 CAB + cos 6 ABC + cos 6 BCA = 1 + rABC R

Solution: Since f 0(x) = 3(x ; 1)(x + 1) it follows that f is strictly monotone non-decreasing in the intervals (;1 ;1] and 1 1) and strictly monotone non-increasing in the interval ;1 1]. Moreover lim f (x) = 1 f (;1) = 3 f (1) = ;1 f (3) = 19 > 0 and so x! 1

Problem 5. The convex pentagon ABCDE is inscribed in a circle

Solution: a.) Using the standard notation for the elements of 4ABC , we obtain that r = S = 4S 2 = 4(p ; a)(p ; b)(p ; c) =
R pR pabc abc
4

1 (a(b2 + c2 ; a2) + b(c2 + a2 ; b2) + c(a2 + b2 ; c2) ; 2abc) = = 2abc = cos 6 CAB + cos 6 ABC + cos 6 BCA ; 1: b.) Applying continuously the equality from a.) and using the fact that ABCDE is inscribed in a circle convex pentagon, it is easy to see that

rABC + rAEC + rEDC = rAED + rABD + rBCD :


The condition of the problem and the above equality imply rBCD = rEDC . Since 4BCD and 4EDC have a side in common and equal circumradiuses we get that 4BCD = 4EDC (prove it using a.)). In particular BC = ED and using rABC = rAED again, we get 4ABC = 4AED.

Problem 6. Show that the equation x2y2 = z2(z2 ; x2 ; y2)


has no solution in positive integer numbers.

Solution: Suppose x y z is a solution for which xy z (which is a

natural number|why?) has its minimum value. Write x y z in the form x = dx1 y = dy1 z = dz1, where d = 2 2 2 2 2 (x y z). Our equation is equivalent to x2 1y1 = z1 (z1 ; x1 ; y1 ). Let u = (x1 z1) v = (y1 z1) x1 = ut y1 = vw. Since z1 divides x1y1, we get z1 = uv. A substitution in the last equality gives 5

(u2 + w2)(v2 + t2) = 2u2v2. Further, since (x1 y1 z1) = 1, it follows that (u w) = 1 (v t) = 1. Therefore

u2 + w2 = v2 v2 + t2 = 2u2
or

u2 + w2 = 2v2 v2 + t2 = u2:

Without loss of generality we may assume that the rst pair of equalities hold. It is easily seen that v and u are odd integer numbers. It follows now from u2 + w2 = v2 that u = m2 ; n2 w = 2mn v = m2 + n2, where m and n are coprime natural numbers (of distinct parity). Substitution in v2 + t2 = 2u2 shows that t2 + (2mn)2 = (m2 ; n2)2 and so t = p2 ; q2 mn = pq m2 ; n2 = p2 + q2 for some natural numbers p and q. Therefore

p2 q2 = m2(m2 ; p2 ; q2)
which shows that p, q, m is a solution of the original equation. It rexy , which contra2 2 mains to be seen that pq = n < d ( p ; q )2 mn = m z dicts the way we have chosen x y z.

XLVII National Mathematics Olympiad: 4th round, 16{17 May 1998


number k for which there exist k sequences of 0's and 1's of length 2n + 2 with the following property: any sequence of 0's and 1's of length 2n + 2 coincides in at least n + 2 positions with some of these k sequences. the respective sequences be ai1 ai2 : : : ai2n+2 for i = 1 : : : k. Since k 3 there is a sequence b1 b2 : : : b2n+2 such that (b2l+1 b2l+2) 6= (ai2l+1 ai2l+2) for l = 0 1 : : : n and i = 1 : : : k. This is a contradiction. For k = 4 it is easily seen that the sequences 000 : : : 0 011 : : : 1, 100 : : : 0 111 : : : 1 have the required property.

Problem 1. Let n be a natural number. Find the least natural

Solution: We shall prove that k = 4. Assume that k 3 and let

Problem 2. The polynomials Pn (x y) n = 1 2 : : : are de ned by P1(x y) = 1 Pn+1 (x y) = (x+y;1)(y+1)Pn(x y+2)+(y;y2)Pn (x y):
1

Prove that Pn (x y) = Pn (y x) for all x y and n.

Solution: We know that P1 (x y) = 1 and P2(x y) = xy + x + y ; 1. Assume that Pn;1 (x y) and Pn (x y) (n 2) are symmetric
polynomials. Then
P

n+1 (x

y)

= = =

(x + (x + (x +

y y

; 1)( ; 1)( ; 1)(


y

y y

+ 1)Pn (x

+ 2) + (y
x)

+ 1)Pn (y + 2 + 1) (y + x
x

+ (y

; ;

y y

2 2

)Pn (x )Pn (y

y) x) x

(x + + (x

; 2) + ( ) ( + ; 1)( + 1)( + + 1)( + 1) n;1 ( + 2 2 2 +( ; )( ; ) n;1 ( ) 2 + ( + ; 1)( + 1)( ; ) n;1 ( + 2 ) 2 +( ; )( + ; 1)( + 1) n;1 ( + 2 )
+ (y
x x P x

; 1)( + 1) ; 2) n;1 (
x P y x y x x x P

+ 1)(x + 1)Pn;1 (y + 2 2 x )Pn;1 (y + 2 x)

+ 2)

n;1 (y
P

+ 2)

y x

+ 2)

x y

y :

and by induction it follows that all the polynomials are symmetric.

Problem 3. On the sides of a non-obtuse triangle ABC a square,

a regular n-gon and a regular m-gon (n m > 5) are constructed externally, so that their centres are vertices of a regular triangle. Prove that m = n = 6 and nd the angles of ABC .

Solution: Let the square, the n-gon and the m-gon be constructed

on the sides AB , BC and CA, respectively. Denote their centres by O1, O2 and O3 denote by A1, B1 and C1 the centres of the equilateral triangles constructed externally on BC , CA and AB . 2

The lines O1C1 O2A1 and O3B1 intersect at the circumcentre O of 4ABC . Since 4A1A2A3 is equilateral, it follows straightforwardly that 4O1O2O3 is equilateral if and only if C1A1jjO1O2 A1B1jjO2O3 and B1C1jjO1O3. This is equivalent to OC1 = OA1 = OB1 = k: C1O1 A1O2 B1O3 On the other hand, OC1 = cot C + tan 30 C 1 O1 cot 45 ; tan 30 OA1 = cot A + tan 30 A1O2 cot 180 n ; tan 30 OB1 = cot B + tan 30 : B1O3 cot 180 m ; tan 30 180 = y. The above identities imply that Set cot 180 = x and cot n m + 1 cot C = k ; kp + 1: + 1 cot B = ky ; kp cot A = kx ; kp 3 3 3 From the identity cot A cot B + cot B cot C + cot C cot A = 1 we get p 2( x + y + 1 ; 3) p p k = p 3xy + ( 3 ; 2)(x + y) p + 3;2 y ; xy + 3 ; 2 3 p : cot C = p x +p 3xy + ( 3 ; 2)(x + y) + 3 ; 2 p p 3,i. e. xy p 3(x + y) ; 3. Since m n 6 it follows that x y The inequality cot C p0 implies p x + y ; xy + 3 ; 2 3 0. p and therefore x + y + 3 ; 2 3 p xy 3(x + y) ; 3 ,i. e. x + y 2 3. This shows that x = y = 3, i. e., m = n = 6. Hence cot C = 0 cot A = cot B = 1, so 6 C = 90 6 A = 6 B = 45 . 3

Problem 4. Let a1 a2 : : : an be real numbers, not all of them


zero. Prove that the equation

1 + a1x + 1 + a2x + : : : + 1 + an x = n
n

has at most one nonzero real root.


X ai Solution: The given equation is equivalent to x p1 + ai x = n. i=1 i Since the function p1 a 6 0) is strictly decreasing, it follows + a x (ai =

that this equation has at most one nonzero root.

Problem 5. Let m and n be natural numbers such that A = ((m +


3)n + 1)=3m is integer. Prove that A is an odd integer number.

well-known that in this case m1 = 6t1 +1. Since m = 2 (6t1 +1) has the form 3t +2 and 1 2 we see that = 1. Then m = 12t1 +2 n and from (m + 3) + 1 = 6km it follows that 4j5n + 1, which is impossible.

Solution: Assume that A is an even integer number, i. e., (m + 3)n + 1 = 6km. Then m is an even integer number. Moreover mj3n +1, which shows that m = 3t +2 and n is odd. Let m = 2 m1, where 1 and m1 is odd. Then 2 j3n + 1 and therefore 2 n+1 n 2 2 . Since m1j3 + 1, it follows that m1ja + 3, where a = 3 . It is

Problem 6. The sides and the diagonals of a regular n-gon X are


coloured in k colours so that: 4

(i) for each colour a and any two vertices A and B of X , the segment AB is coloured in colour a or there is a vertex C such that AC and BC are coloured in colour a (ii) the sides of any triangle with vertices among the vertices of X are coloured in at most two colours. Prove that k 2. ent colours a, b, c. We shall construct an in nite subset of vertices of X , which will imply a contradiction. Let Z 2 X and A1 is a vertex such that the colour of A1Z is a. From (i) it follows that there is a vertex B1, such that the colour of B1Z and B1A1 is b. Analogously there is a vertex C1 such that the colour of C1Z and C1B1 is c. Considering the triangles C1A1Z and C1A1B1 we see (using condition (ii)) that the colour of C1A1 is c. Let A2 2 X be such that the colour of A2C1 and A2Z is a. It is easily seen that A2 6= A1 and the colour A2A1 and A2B1 is a. Now we shall proceed by induction. Let the vertices A2 B2 C2 : : : Ak;1 Bk;1 Ck;1 be such that the colour of AiAj AiBj AiCj is a, the colour of BiAj BiBj BiCj is b and the colour CiAj CiBj CiCj is c (2 j < i < k). Take a vertex Ak such that the colour of Ak Ck;1 and Ak Z is a. Then considering the triangles ZAk Bj and Ck;1 AkBj (j < k), ZAkCj and Bj+1 Ak Cj (j + 1 < k), Ak Aj Bj and Ak Aj Bj we see that the colour of Ak Bj Ak Cj and AkCj is a. The vertices Bk and Ck are constructed in a similar way. 5

Solution: Assume that the colouring involves at least three di er-

XLVIII National Mathematics Olympiad: 3rd round, 17{18 April 1999


Problem 1. Find all triples (x y z) of natural numbers such that y is a prime number, y and 3 do not divide z, and x3 ; y3 = z2.
Nikolay Nikolov

Solution: Since (x ; y)((x ; y)2 + 3xy)) = x3 ; y3 = z2, it follows from the conditions of the problem that x ; y and (x ; y)2 +3xy are relatively prime. Therefore x ; y = u2 and x2 + xy + y2 = v2. Thus 3y2 = (2v ; 2x ; y)(2v + 2x + y) and since y is a prime number,
there are three cases to consider: 1. 2v ; 2x ; y = y 2v + 2x + y = 3y. Now x = 0, which is impossible. 2. 2v ; 2x ; y = 1 2v + 2x + y = 3y2. Now 3y2 ; 1 = 2(2x + y) = 2(2u2 + 3y) and it follows that 3 divides u2 + 1, which is impossible. 1

3. 2v ; 2x ; y = 3 2v + 2x + y = y2. Now y2 ; 3 = 2(2x + y) = 2(2u2 +3y) and it follows that (y ; 3)2 ; (2u)2 = 12. Therefore y = 7 u = 1 x = 8 z = 13. Direct veri cation shows that (8 7 13) is a solution of the problem.

Problem 2. A convex quadrilateral of area S is inscribed in a

circle whose centre is a point interior to the quadrilateral. Prove that the area of the quadrilateral whose vertices are the projections of the point of intersection of the diagonals on the sides does not exceed S Christo Lesov 2. centre O and radius R. Denote by E the point of intersection of AC and BD. Denote further by M , N , P , Q and F the projections of E on AB , BC , CD, DA and MN , respectively. We know that MN = BE sin 6 ABC = BE2RAC . Also,

Solution: Let ABCD be a quadrilateral inscribed in a circle with

sin 6 AEB sin 6 CBE: EF = EM sin 6 EMN = AE BE AB Since BE sin 6 CBE = CE sin 6 BCE = CE AB and AE CE = 2 R 2 OE 2 sin 6 AEB . Therefore R2 ; OE 2 , it follows that EF = R ; 2R 6 AEB (R2 ; OE 2 ) : S4MEN = MN2 EF = AC BE sin 8 R2 2

Similar equalities hold for S4NEP S4PEQ and S4QEM . By combining the above we obtain 6 AEB (R2 ; OE 2 ) SABCD SMNPQ = AC BD sin 4 R2 2 Q. E. D.

Problem 3. In a competition 8 judges marked the contestants by

yes or no. It is known that for any two contestants, two judges gave both a yes two judges gave the rst one a yes and the second one a no two judges gave the rst one a no and the second one a yes, and nally, two judges gave both a no. What is the greatest possible number of contestants? Emil Kolev

table with 8 rows and n columns such that the cell in the ith row and j th column contains 0 (1) if the ith judge gave the j th contestant a no (a yes). The conditions of the problem now imply that the table formed by any two columns contains among its rows each of the pairs 00, 01, 10 and 11 twice. We shall prove that 8 columns having this property do not exist. Assume the opposite. It is easily seen that if in any column all 0s are replaced by 1s and vice versa, the above property is retained. Therefore without loss of generality suppose that the rst row consists of 0s. Denote the number of 0s in the ith row by ai. It is clear that the total number of 0s is 8 8 4 = 32. Further, the number of occurrences of 00 is 2 = 56. 2 ! On the other hand the same number is 3
8 X

Solution: Denote the number of contestants by n. Consider a

i=1

ai . Since a = 8, it 1 2

ai = 24. It is easy to prove now that i=2 2 ! 8 X ai Therefore 56 = 58, which is false.
follows that
i=2 i=1

8 X

8 X

ai

30. 0 0 1 1 0 1 1 0 0 0 1 1 1 0 0 1

The diagram on the right shows that it is possible to have exactly 7 contestants:

0 0 0 0 1 1 1 1

0 1 1 0 0 0 1 1

0 1 1 0 1 1 0 0

0 1 0 1 0 1 0 1

0 1 0 1 1 0 1 0

Problem 4. Find all pairs (x y) of integer numbers such that x3 =


y3 + 2y2 + 1.
Nikolay Nikolov and Emil Kolev

Solution: It is obvious that x > y. On the other hand x < y + 1 () (y + 1)3 > y3 + 2y2 + 1 () y(y + 3) > 0. Therefore if y > 0 or y < ;3 the problem has no solution. Direct veri cation yields all pairs (x y) which satisfy the equality x3 = y3 + 2y2 + 1, namely (;2 ;3), (1 ;2) and (1 0). Problem 5. Let B1 and C1 be points on the sides AC and AB of 4ABC . The straight lines BB1 and CC1 intersect at point D. Prove that the quadrilateral AB1DC1 is circumscribed if and only if the incircles of 4ABD and 4ACD are tangent.
Rumen Kozarev and Nikolay Nikolov

Solution: Note that the incircles of 4ABD and 4ACD are tangent if and only if AB + AD ; BD = AC + AD ; CD, so AB + CD =
Suppose AB1DC1 is circumscribed and the incircle touches AB1, B1D, DC1 , C1A in the points M N P Q, respectively. Therefore AB + CD = AQ + BQ + CP ; DP = AM + BN + CM ; DN = AC + BD. Conversely, let the incircles of 4ABD and 4ACD be tangent. Denote the point of intersection of the tangent through C (di erent from CA) with the incircle of 4ABB1 by D0 . It follows from the above that BD0 ; CD0 = AB ; AC = BD ; CD, so DD0 = jCD ; CD0j. Therefore D0 D, which completes the proof.

AC + BD.

Problem 6. Each interior point of an equilateral triangle of p side 1


3. lies in one of six circles of the same radius r. Prove that r 10 Nikolay Nikolov and Emil Kolev

Solution: Divide each side of the triangle into ve equal parts

and draw lines parallel to the sides through these points. Thus the 1 . The total triangle is divided into 25 equilateral triangles of side 5 number of vertices is 21 > 6 3. Therefore there exist 4 points which are interior to one and the same circle. It is easy to see now that p 3 , which solves the problem. r 10

XLVIII National Mathematics Olympiad: 4th round, 18{19 May 1999


Problem 1. The faces of an orthogonal parallelepiped whose dimensions are natural numbers are painted green. The parallelepiped is partitioned into unit cubes by planes parallel to its faces. Find the dimensions of the parallelepiped if the number of cubes having no green face is one third of the total number of cubes. Sava Grozdev

Solution: Let x y z be the dimensions of the parallelepiped. It follows from the conditions of the problem that x 3 and (x ; x ; 2 y ; 2 z ; 2 , when x 7, . Since 2)(y ; 2)(z ; 2) = xyz 3 x y z ( x ; 2)( y ; 2)( z ; 2) 5 1 we obtain that ( )3 > . Therefore x 6
xyz 7 and thus x = 3, x = 4, x = 5 or x = 6.
1 3 1. If x = 3, then (y ; 2)(z ; 2) = yz, which is impossible.

2. If x = 4, then 2(y ; 2)(z ; 2) = 4yz 3 , so (y ; 6)(z ; 6) = 24. In this case the only solutions are (4 7 30), (4 8 18), (4 9 14) and (4 10 12). 3. If x = 5, then 3(y ; 2)(z ; 2) = 5yz 3 , so (2y ; 9)(2z ; 9) = 45. Therefore the solutions are (5 5 27), (5 6 12) and (5 7 9). 4. If x = 6, then 4(y ; 2)(z ; 2) = 2yz, so (y ; 4)(z ; 4) = 8. Thus there is an unique solution (6 6 8). Answer: The problem has 8 solutions|(4 7 30), (4 8 18), (4 9 14), (4 10 12), (5 5 27), (5 6 12), (5 7 9) and (6 6 8).

Problem 2. Let fang1 n=1 be a sequence of integer numbers such


that (n ; 1)an+1 = (n + 1)an ; 2(n ; 1) for any n 1. If 2000 divides a1999, nd the smallest n 2 such that 2000 divides an. Oleg Mushkarov, Nikolai Nikolov

n 2. Therefore the sequence is uniquely determined by its second term. Furthermore the sequence an = (n ; 1)(cn + 2) (where c = a2 ; 1 is an arbitrary real number) satis es the equality from the 2 conditions of the problem. We conclude now that all sequences which satisfy this equality are of the kind given above. Since all terms are integer numbers and 2000 divides a1999, it is easy to see that c is integer and c = 1000m + 2. Therefore 2000 divides an if and only if
2

n + 1 a ; 2 when Solution: It is obvious that a1 = 0 and an+1 = n ;1 n

1000 divides (n ; 1)(n +1). Thus n = 2k +1 and k(k +1) is divisible by 250 = 53 2. Since k and k + 1 are relatively prime, we get that the smallest n 2 equals 2 124 + 1 = 249. and one of its sides is of length n, where n is a square-free natural number. Prove that the ratio of the circumradius and the inradius is an irrational number. Oleg Mushkarov, Nikolai Nikolov

Problem 3. The vertices of p a triangle have integer coordinates

Solution: Suppose that R r = q, where q is a rational number. Without loss of generality assume that one of the ends of the side p of length n is at the origin of the coordinate system. Let the remaining two vertices have coordinates (x y) and (z t), where px y z and t are integers. The sides of our triangle have lengths a = A b= p p 2 2 2 2 B and c = C , where n = A = x + y B = z + t and C = (x ; z)2 +(y ; t)2. It follows from the conditions of the problem that
abc p = abc(a + b + c) q=R = r 4S S 8S 2 where S is the rational (prove it!), it p area of pthe triangle. p p Since S is 2 follows that p ABC ( p A + B + C) = p8S q is a rational number. Therefore Ap BC + B AC = 8S 2q ; C AB and after squaring we obtain that AB is a rational number. Thus AB is a perfect square. By analogy both BC and CA are perfect squares. Let AB = E 2, BC = F 2 and CA = G2, where E , F and G are integer. Write A B 2 2 and C in the following form: A = a1a2 2 B = b1b2 and C = c1 c2, 2 where a1 b1 and c1 are square-free integers. So a1b1(a2b2) = m2 and therefore a1b1 is a perfect square, whence a1 = b1. By analogy
3

2 2 a1 = c1. Thus A = ma2 1 B = mb1 C = mc1 , where m is square-free. It follows from ma2 1 = n that m = n a1 = 1 and we obtain x2 + y 2 = n z2 + t2 = nb2 1 (x ; z)2 + (y ; t)2 = nc2 1 Since both b1 and c1 are integer, it follows from the Triangle Inequality that 1+b1 > c1 and 1+c1 > b1, whence b1 = c1. It is easy to determine now that x2 + y2 = 2(xz + yt) and consequently 2(xz + yt) = n. Let 2(xt ; yz) = k. Then n2 + k2 = 4(x2 + y2)(z2 + t2) = 4n2b2 1, 2 2 2 2 so k = n (4b1 ; 1). Therefore 4b1 ; 1 is a perfect square, which is impossible.

Problem 4. Find the number of all natural numbers n 4 n

1023, such that their binary representations do not contain three consecutive equal digits. Emil Kolev

and ones of length n which begin with 1 and do not contain three consecutive equal digits. Also, for any a b 2 f0 1g denote by xn ab the number of sequences of zeroes and ones of length n which begin with 1 and do not contain three consecutive equal digits, such that the last two terms are respectively a and b. It is easy to see that for n 5 n;1 n ;1 n;1 n n;1 n;1 n n;1 xn x01 = xn 00 = x10 00 + x10 x10 = x11 + x01 x11 = x01 : ;1 n;1 Adding up the above equalities, we obtain an = an;1 + xn 10 + x01 = an;1 + an;2 . Since a3 = 3 and a4 = 5, it follows that a5 = 8, a6 = 13, a7 = 21, a8 = 34, a9 = 55 and a10 = 89. 4

Solution: Denote by an n 3, the number of sequences of zeroes

Since the required number is equal to a3 + a4 + : : : + a10, it follows from the above that the answer is 228.

6 A1B1C1, 6 BCA = 6 B1C1A1, 6 CAB = 6 C1A1B1. Prove that the orthocentres of 4ABC and 4A1B1C1 are equaly remote from the circumcentre of 4ABC . Nikolai Nikolov

Problem 5. The vertices A, B and C of an acute triangle ABC lie on the sides B1C1 C1A1 and A1B1 of 4A1B1C1 and 6 ABC =

6 B1HA + 6 C1HA = 6 B1CA + 6 C1BA = 26 B1A1C1 and likewise 6 C1HA1 = 26 C1B1A1 and 6 A1HB1 = 26 A1C1B1, so H is the circumcentre of 4A1B1C1. Let us draw straight lines passing through the vertices of 4ABC and parallel to the corresponding sides and denote their points of intersection by A0, B0 and C0. Since 6 A0B0C0 = 6 A1B1C1 6 B0C0A0 = 6 B1C1A1 and 6 C0A0B0 = 6 C1B1A1, it follows from the above that the segments A0H B0H and C0H are of equal length and are diameters of k1 k2 and k3. It is clear now that there exists a composition of a rotation and a homothecy, both centred at H , such that the image of 4A1B1C1 is 4A0B0C0. Therefore the image of orthocentre H1 of 4A1B1C1 is the orthocentre H0 of 4A0B0C0. Thus 6 HH1 H0 = 6 HA1A0 = 90 , and to solve the problem we have to show that the circumcentre O of 4ABC is the midpoint of HH0. Indeed, the image of 4ABC by a homothecy centred at the cen5

Solution: Denote by H the orthocentre of 4ABC . Since 6 CHB = 180 ; 6 CAB = 180 ; 6 C1A1B1, we have that A1 lies on the circumcircle k1 of 4BHC . Similarly, B1 and C1 lie on circumcircles k2 and k3 of 4CHA and 4AHB . Therefore 6 B1HC1 =

troid coe cient ;! ;2 is 4A0B0C;! 0 . Therefore ;! of 4ABC ;! and with ;! ;! MH0 = ;2 MH and since MH = ;2 MO, we obtain OH0 = ; OH .

Problem 6. Prove that the equation


x3 + y3 + z3 + t3 = 1999
has in nitely many integer solutions.
Grigor Grigorov

k and l are integer. After simple calculations we obtain that our equation is equivalent to l(l +1) = 20k2, whence (2l +1)2 ; 80k2 = 1. The latter is Pell's equality. Since l = 4 k = 1 is a p solution, all solutions are of the form ( l k ), where 2 l + 1 + k n n n n 80 = (9 + p n 80) n = 1 2 : : :. Therefore the original equation has in nitely many integer solutions.

Solution: Since 103 + 103 + (;1)3 + 03 = 1999, we are looking for solutions of the form x = 10 ; k y = 10 + k z = ;1 ; l t = l, where

Union of Bulgarian Mathematicians Sava Grozdev Emil Kolev

BULGARIAN MATHEMATICAL COMPETITIONS 2000

So a, 2000

Winter Mathematical Competition Russe, 4-6 February 2000


where n is an integer. a) Factorize the expression ;3n3 ; 4n2 + n + 2: b) Find all n for which the inequality holds true for any positive number x: Solution: a) ;3n3 ; 4n2 + n + 2 = (n + 1)2(2 ; 3n): b) Since 0:x < 0 is not true for any x it follows that n 6= ;1: If n > ;1 then the inequality is equivalent to (n;1)x < (n+1)(2;3n): If n = 1 then 0:x < ;2 which is not true for any x: Let n 6= 1: 1)(2 ; 3n) < 0 and the inequality is not If n ; 1 > 0 then (n + n ;1 true for any positive x: If n ; 1 < 0 then the inequality becomes 1)(2 ; 3n) and n = 0 is the only solution. If n < ;1 then x > (n + n ;1 (n + 1)(2 ; 3n) > 0 and there exists x > 0 which is not a solution. n;1 Therefore the inequality has no solution when n < ;1:

Problem 8.1 Given the inequality (n2 ; 1)x < ;3n3 ; 4n2 + n + 2

Problem 8.2 In an isosceles 4ABC (AC = BC ) the points A1 B1

and C1 are midpoints of BC AC and AB respectively. Points A2 and B2 are symmetric points of A1 and B1 with respect to AB: Let M be the intersecting point of CA2 and A1C1 and let N be the intersecting point of CB2 and B1C1: The intersecting point of AN and BM is denoted by P: Prove that AP = BP: 2

Solution: Since CC1 k A1A2 and CC1 = A1A2 we have that

CC1A2A1 is a parallelogram. Thus, A1M = C1M: But A1B1C1B is also a parallelogram and therefore the intersecting point of BM and AC is B1: Hence P lies on the median BB1: Analogously P lies on the median AA1: In the isosceles 4ABC the medians AA1 and BB1 2 AA = 2 BB = BP: are of the same length. Therefore AP = 3 1 3 1 Problem 8.3 Find all pairs of prime numbers p and q such that p2 + 3pq + q2 is: a) a perfect square b) a power of 5. Solution: a) Let p2 + 3pq + q2 = r2 where p and q are prime numbers. If p 6= 3 q 6= 3 then p2 + 3pq + q2 2(mod 3) and r2 2(mod 3) a contradiction. Without loss of generality p = 3 and we get that q2 + 9q + 9 = r2 and 4q2 + 36q + 36 = (2r)2: Therefore (2q ; 2r +9)(2q +2r +9) = 45: We may assume that r > 0 and so 2q +2r +9 = 15 or 2q +2r +9 = 45: In the rst case q + r = 3 which is impossible and in the second case solving the system q+r = 18 2q ; 2r + 9 = 1
we nd q = 7: Because of the symmetry the only solutions are p = 3 q = 7 and p = 7 q = 3: b) Let p2 + 3pq + q2 = 5n where n is a natural number. Since p 2 q 2 we have p2 + 3pq + q2 20 and so n 2: It follows now that 25=(p2 + 3pq + q2) and 5=(p2 + 3pq + q2) = (p ; q)2 + 5pq: Thus, 5=(p ; q)2 and 25=(p ; q)2: Therefore 25=5pq showing that p = 5 or q = 5: But if p = 5 then q = 5 (and vice versa). We obtain p2 + 3pq + q2 = 125 = 53: The only solution of the problem is p = q = 5: 3

a parameter. a) Solve the equation. b) If a is the square of a prime number prove that the equation has a solution which is a compose integer. Solution: After squaring and simple calculations we obtain (26a ; 1)x = (26a ; 1)(26a + 1): 1 then every x 6= 2 is a solution. If a 6= 1 then the a) If a = 26 26 only solution is x = 26a + 1: b) Let a = p2 where p is a prime number. If p = 3 then x = 235 is not a prime. If p 6= 3 then p = 3k 1 and x = 26(3k 1)2 + 1 = 3A + 27 which is divisible by 3: diameter BD: Let M be the symmetric point of A with respect to BD and let N be the intersecting point of the straight lines AM and BD: The line passing through N which is parallel to AC intersects CD and BC in P and Q respectively. Prove that the points P C Q and M are vertices of a rectangle. Solution: It follows from the condition of the problem that _ M lies on the circumcircle of ABCD: Since < ) MAC =< ) MBC = MC 2 and < ) MNQ =< ) MAC i.e. < ) MNQ =< ) MBC , we get that the points M N B and Q lie on a circle. Since < ) MNB = 90 we conclude that < ) BQM = 90 : Also, since 4BDC is a right angle triangle we _ have MC that MQ k PC: From the other hand < ) MDC =< ) MAC = 2 and therefore < ) MDC =< ) MNQ so the points N P M and D lie on a circle. Thus, < ) MPD =< ) MND = 90 and MP k CQ: Therefore P C Q and M are vertices of a rectangle. 4

1 = 1 Problem 9a.1 Given the equation jx ; 2j jx ; 52aj where a is

Problem 9a.2 The quadrilateral ABCD is inscribed in a circle with

Problem 9a.3 See Problem 8.3. Problem 9b.1 Given the system:
1 +x = a;1 x+y x x+y = a;2 where a is a real parameter. a) Solve the system if a = 0: b) Find all values of a such that the system has an unique solution. c) If a 2 (2 3) and (x y) is a solution of the system nd all values of a such that the expression x + y takes its minimal value. y x 1 are roots of the equation Solution: It follows easily that x and x + y t2 ; (a ; 1)t + a ; 2 = 0: There are two cases to be considered: x = 1 x = 1 (1) when a 6= 2 1 = a ; 2 () 3 ; a y = a;2 x+y and x = a;2 x = a;2 (2) 1 = 1 () y = 3 ; a: x+y a) When a = 0 we have (x y) = (1 ; 3 2 ) or (x y) = (;2 3): 5

b) The sytem has an unique solution (0 1) when a = 2 and (1 0) when a = 3: y are positive. c) If (x y) is a solution and a 2 (2 3) then x and y x xy x y Further y + x 2 and equality occures when y = x : It follows a ; 2 : Using the equality a ; 2 = 3 ; a from (1) and (2) that x = y 3;a 3;a a;2 5 we nd the only value of a = 2 : intersects AB at point L: The feet of the perpendiculars from L to AC and BC are denoted by M and N respectively. Let P be the intersecting point of AN and BM: Prove that CP ? AB: Solution: Let l be the line through C which is parallel to AB: Let F and E be respectively the intersecting points of AN and BM with l: The intersecting point of CP and AB is denoted by D: We obtain AD = PD = BD so AD = CF : From the other hand AM = CF PC CE BD CE CM AB and BN = AB : But CM = CN and we get AM = CF : CE CN CF BN CE AD AM Therefore BD = BN which implies

Problem 9b.2 Given an acute 4ABC: The bisector of < ) ACB

AM = BN AD BD Further if CH ? AB (H 2 AB ) then 4ALM 4AHC and so AL = AM : In the same manner BL = BN : But CL is a bisecAC AH BC BH AL BL = so AM = BN : The last equation tor and therefore AC BC AH BH combined with (1) gives D H which implies CP ? AB: Problem 9b.3 Prove that the digit of the hundreds of 21999 +22000 +
(1) 6

22001 is even. Solution: Write the number 21999 +22000 +22001 in the form 21999(1+ 2 + 4) = 7:29:21990 = 7:29:210:21980 = 7:29:210:(220)99: Since 29 = 512 210 = 1024 and 220 = (210)2 we have that the last two digits of 220 coincide with the last two digits of 242 so the last two digits of 220 are 76: Moreover the last two digits of 76:76 are also 76: Therefore the last two digits of the given number are the last two digits of the product 7:12:24:76 which are 1 and 6: Since 21999 + 22000 + 22001 is divisible by 8 and it ends by 16 the digit of the hundreds is even. sin 3x form an in nite arithmetic progression. Solution: Since sin 2x = 2 sin x cos x and sin 3x = sin x(4 cos2 x;1) we may write the equation in the form sin x (2 cos2 x ; (2 ; a) cos x ; a) = 0: Thus sin x = 0 cos x = 1 or cos x = ; a 2 : The nonnegative solutions of the equations sin x = 0 and cos x = 1 are x = k and x = 2k k = 0 1 2 : : : respectively. a Let jaj > 2: In this case the equation cos x = ; 2 has no solution and therefore the nonnegative solutions of the initial equation are 0 2 : : : which form an arithmetic progression. Let now jaj 2 and let x0 be the only solution of the equation cos x = ; a 2 in the interval 0 ]: In this case the nonnegative solutions of the last equation are x = x0 + 2k and x = 2 ; x0 + 2k : It is clear now that the nonnegative solutions form an arithmetic progression only when x0 = 0 x0 = 2 and x0 = so giving a = ;2 a = 0 and a = 2: The values of a are a = 0 and jaj > 2:

Problem 10.1 Find all values of the real parameter a such that the nonnegative solutions of the equation (2a ; 1) sin x +(2 ; a) sin 2x =

Problem 10.2 Let O I and H be respectively the circumcenter, incenter and orthocenter for an acute nonequilateral 4ABC: Prove
7

that if the circumcircle of 4OIH passes through one of the vertices of 4ABC then it passes through another vertex of 4ABC: Solution: Assume that O I H and C lie on a circle. It is well known that CI is the bisector of < ) HCO: Thus < ) IHO =< ) ICO =< ) ICH =< ) HOI and it follows from 4IHO that IH = IO = t: We shall prove that if < ) BAC 6= 60 then O I H and A lie on a circle. Denote by M and N the projection points of I respectively on AO and AH: Let O1 and O2 be such that IO1 = IO2 = t and O1 lies between A and M and M lies between O1 and O2: Analogously let H1 and H2 be such that IH1 = IH2 = t and H1 lies between A and N and N lies between H1 and H2: If O O1 H H1 or O O2 H H2 then 4AIO 4AIH and therefore AO = AH: But AH = 2AO cos < ) BAC and so < ) BAC = 60 : If O O1 H H2 or O O2 H H1 then it follows from < ) IO1O2 =< ) IO2O1 =< ) IH1H2 =< ) IH2H1 that AOIH is inscribed. Suppose now that A and B do not lie on the circumcircle of 4OIH: In this case < ) BAC =< ) ABC = 60 and therefore 4ABC is equilateral which is a contradiction. the modulus of the di erence of the sum of the elements from the i;th row and the sum of the elements from the j ;th column. Prove that every element of the table equals either to the sum or to the di erence of two other elements of the table. Solution: Let p1 p2 p3 and q1 q2 q3 be the sum of the elements in the rst, second and third row and in the rst, second and third column respectively. It is clear that p1 + p2 + p3 = q1 + q2 + q3: Therefore the element in the rst row and the rst column equals to jp1 ; q1j: From the other hand jp1 ; q1j = jp2 + p3 ; q2 ; q3j which implies jp1 ; q1j = 1jp2 ; q2j + 2jp3 ; q3j where 1 2 2 +1 ;1: Since 8

Problem 10.3 In each of the cells of a 3 3 table is written a real number. The element in the i;th row and j ;th column equals to

By analogy every element of the table is the sum or the di erence of two other elements. Problem 11.1. Prove that for every positive number a the se3 2 quence fxng1 n=1 such that x1 = 1 x2 = a xn+2 = xn+1 xn n 1 is convergent and nd its limit. Solution: It follows by induction that the terms of the sequence 1 n fxng1 n=1 can be expressed as xn = a where f n gn=1 is a siquence + n n 1: Thus, de ned by 1 = 0 2 = 1 n+2 = 2 n+1 n+2 ; 3 n 1 1 ( ; n+1 = ; ( n+1 ; n ) and therefore n+2 ; n+1 = ; 3 n 3k 2 1 for k = 1 : Adding the equalities k+2 ; k+1 = ; 1) = ; 3 3 0 0 1 : : : n after simple calculations we obtain n+2 ; 0 = ; 1 + 3 n+1 1 n 1 ; ;1 1 1 3 (1 ; ; 1 n+1 ): Since 3 ;3 + : : : + ;3 = = 4 3 1+ 1 3 1 n = 0 it follows that lim = 3 : This shows lim ; 0 = 0 and n !1 n!1 n 4 3 3 4: that the sequence fxng1 x = a n n=1 is convergent and nlim !1
q

jp1 ; q1j 0 it is clear that 1 = 2 = ;1 is impossible. Therefore jp1 ; q1j is either the sum or the di erence of jp2 ; q2j and jp3 ; q3j:

Problem 11.2 Given a convex quadrilateral ABCD where M is

the intersecting point of its diagonals. It is known that DB = 3DM AM = MC: a) Express BC and CD by the sides of 4ABD: b) Prove that if 2 < ) ADB ; < ) ABD = 180 then < ) DBC = 2< ) BDC: Solution: a) Denote AB = c BC = p CD = q DA = b and 9

DB = a: It follows from the condition of the problem that DM = 1 a MB = 2 a: The formula for the median in a triangle applied for 3 3 4 ABC and 4ACD gives 16 a2 = 2p2 + 2c2 ; 4AM 2 4 a2 = 2q2 + 2b2 ; 4AM 2 (1) 9 9 Let < ) AMB = : From the Law of Cosines for 4AMB and 4AMD we obtain a2 ; 4 AM:a cos b2 = AM 2 + a2 + 2 AM:a cos c2 = AM 2 + 49 3 9 3 2 a2 and a substitution in (1) gives Thus 2b2 + c2 = 3AM 2 + 3 2 b2 ; 3c2 q2 = 3b2 + 6c2 ; 2a2 (2) p2 = 4a + 12 9 9
b) It follows from the condition of the problem that < ) ADB > 90 and so c > a: Let D1 be a point on AB such that D1B = DB: < ) ABD = 180 + < ) ABD = Further < ) AD1B =< ) ABD + 180 ; 2 2 < ) ADB: Therefore 4AD1D 4ADB and thus b2 = c(c ; a): A 2a and q2 = 9c2 ; 3ac ; 2a2 : substitution in (2) implies p = 3c ; 3 9 Hence q2 = p(p ; a): Let B1 be a point on BC (B lies between C p + a it follows that and B1), such that BB1 = BD = a: Since q = p q 4DBC 4B1DC: Therefore < ) DBC =< ) BB1D+ < ) BDB1 = 2< ) BB1D = 2 < ) BDC:

Problem 11.3 See problem 10.3.


10

Spring Mathematical Tournament Jambol, 24-26 March 2000


Problem 8.1. Let f (x) be a linear function such that f (0) = ;5 and f (f (0)) = ;15: Find all values of m for which the set of the solutions of the inequality f (x):f (m ; x) > 0 is an interval of length
2:

Solution: Let f (x) = ax + b: It follows from f (0) = ;5 that b = ;5 and from f (f (0)) = ;15 that a = 2: Therefore the function is f (x) = 2x ; 5: Consider the inequality (2x ; 5)(2(m ; x) ; 5) > 0 () (2x ; 5)(2m ; 5 ; 2x) > 0: The solution of the last inequality is an 5 and 2m ; 5 : Therefore 5 ; 2m ; 5 = 2 interval with end points 2 2 2 2 so j5 ; mj = 2: Finally, we obtain that m = 3 and m = 7: Problem 8.2 Given an isosceles right angle triangle ABC with < ) ACB = 90 : Point P lies on BC M is the midpoint of AB and let L and N be points from the segment AP such that CN ? AP

and AL = CN: a) Find < ) LMN b) If the area of 4ABC is 4 times greater than the area of 4LMN nd < ) CAP: Solution: a) Let < ) CAP = : We nd < ) ACN = 90 ; and < ) MCN =< ) ACN ; 45 = 45 ; =< ) LAM: Since AM = CM and AL = CN it follows that 4AML = 4CMN: Therefore < ) AML = < ) CMN and so < ) LMN = 90 ; < ) AML+ < ) CMN = 90 : b) It follows from 4AML = 4CMN that LM = MN and since 11

2 2 : Also SABC = AC : Now < ) LMN = 90 we get SLMN = MN 2 2 4MN 2 = AC 2 applies that MN = AC : Denote by Q the midpoint 2 2 2 AC of AC: Thus QM = QN = 2 = MN: Therefore 4QMN is equilateral and < ) QNM = 60 which implies < ) QNA = 60 ; 45 = 15 : But AQ = QN and so < ) CAP =< ) QNA = 15 : Problem 8.3 There are 2000 white balls in a box. There are also su ciently many white, green and red balls. The following operations are allowed: 1) Replacement of two white balls with a green ball 2) Replacement of two red balls with a green ball 3) Replacement of two green balls with a white bal and a red ball 4) Replacement of a white ball and a green ball with a red ball 5) Replacement of a green ball and a red ball with a white ball a) After nitely many of the above opperations there are three balls left in the box. Prove that at least one of them is a green ball. b) Is it possible after nitely many opperations to have only one ball left in the box? Solution: Consider a box with x white, y green and z red balls. Direct veri cation shows that after applying any of the allowed operations the sum x + 2y + 3z does not change modulo 4. Since the initial values are x = 2000 y = z = 0 we obtain that this sum is congruent to 0 modulo 4. a) There are 3 balls in the box and therefore x + y + z = 3: Moreover x +2y +3z 0(mod4): If a green ball is not in the box then y = 0 and so x + z = 3 and x +3z x +3(3 ; x) 1 ; 2x 0(mod4)

12

which is impossible. b) Suppose that there is only one ball left in the box. Therefore x + y + z = 1 and x + 2y + 3z 0(mod4) which is impossible. Problem 9a.1 Find all values of m such that the equation 1 + m ; 2m (jx ; mj ; m) = 0 x + m x ; m m2 ; x2 has exactly one nonnegative root. 1 + m ; 2m = 0 is equivalent Solution: The equation x + m x ; m m2 ; x2 to (when x 6= m) to (m + 1)x = ;m(m + 1): When m = ;1 it has in nitely many roots { all numbers x 6= 1: If m 6= ;1 we obtain x = ;m: The equation jx ; mj ; m = 0 has two roots: x = 0 and x = 2m when m > 0 an unique root: x = 0 when m = 0 and has no roots when m < 0: Let m < 0 and m 6= ;1: In this case the equation has an unique root x = ;m and it is nonnegative. Let m = 0: Then the equation 1 jxj = 0 which obviously has no roots. Let m > 0: The becomes x equation has three roots x = ;m x = 0 x = 2m and two of them are nonnegative. Thus, the desired values of m are m < 0 and m 6= ;1: Problem 9a.2 Given an acute-angled triangle ABC and let and be its angles respectively to A B and C: For an arbitrary interior point M denote by A1 B1 and C1 respectively the feet of the perpendiculars from M to BC CA and AB: Find the locus of M for which the triangle A1B1C1 is a right angle triangle. Solution: Let M be a point such that < ) A1C1B1 = 90 : Since the quadrilateral AC1MB1 is inscribed we have < ) B1C1M =< ) B1AM = x: Analogously < ) A1C1M =< ) A1BM = y: Denote z =< ) AMB: It follows now from the quadrilateral AMBC that x + + y + (360 ; 13

z) = 360 : Since x + y =< ) A1C1B1 = 90 we get z = 90 + : Therefore < ) AMB = 90 + : Thus, the locus of M such that < ) A1C1B1 = 90 is an arc G1 in the interior of 4ABC for which the segment AB is seen under angle 90 + : Analogously one can prove that the locus of M such that < ) C1B1A1 = 90 is an arc G2 in the interior of 4ABC for which AC is seen under angle 90 + : Further, the locus of M such that < ) B1A1C1 = 90 is an arc G3 in the interior of 4ABC for which BC is seen under angle 90 + : The desired locus is the union of the three arcs G1 G2 G3: Problem 9a.3 See Problem 8.3 Problem 9b.1 The real numbers x and y are such that x2 +xy +y2 = 1: If F = x3y + xy3 a) prove that F ;2 b) nd the greatest possible value of F: Solution: a) It follows from the condition of the problem that (x + y)2 = 1 + xy and therefore xy ;1: From the other hand x2 + y2 = 1 ; xy: Thus, xy 1 so z = xy 2 ;1 1]: Hence F = xy(x2 + y2) = z(1 ; z) and the inequality F ;2 is equivalent to the inequality z2 ; z ; 2 0: The latter one is true for any z 2 ;1 2]: b) Assume that the greatest value of F exists and denote it by A: This implies that the system xy(x2 + y2) = A (1) xy + (x2 + y2) = 1 has a solution. It is clear that t1 = xy and t2 = x2 + y2 are roots of the quadratic equation t2 ; t + A = 0: But t2 ; t1 = 1 (x ; y)2 + 1 (x + y)2 0 and the equality is impossible since if 2 2 so then x = y = 0 which is a contradiction to x2 + xy + y2 = 1: 1 : The system Therefore t2 > t1 and so D = 1 ; 4A > 0 i.e. A < 4 14

= t1 and the last is equivalent to (1) is equivalent to x2 xy 2 + y = t2 2 (x ; y) = t2 ; 2t1 The latter system has a solution i the in(x + y)2 = t2 + 2t1: equalities t2 ; 2t1 0 and t2 + 2t1 0 hold true. It su ces to prove 2 that when A < 1 4 the roots t1 < t2 of the equation g(t) = t ; t + A satisfy the inequalities t2 ; 2t1 0 and t2 + 2t1 0: If A = 0 then t1 = 0 and t2 = 1 and the inequalities hold true. It is clear 1 ): Now t > 0 t > 0 that we can consider only the case A 2 (0 4 1 2 and the inequality t2 + 2t1 0 holds true. From the other hahd t1 + t2 = 1 and the inequality t2 ; 2t1 0 is equivalent to t2 2 3: The same inequality is equivalent to t1 1 t1 and 3 : For the roots 1 1 t 2 which is equivalent to g( 3 ) 0 t2 we obtain t1 3 2 2 g( 3 ) 0: 3 p 2 then x = y = 3 satisfy : Conversely, if A = Therefore A 2 9 9 3 the condition of the problem. Problem 9b.2 A line l is drown through the orthocenter of an acute-angled triangle ABC: Prove that the lines symmetric to l with respect to the sides of the triangle intersect in a point. Solution: Let the intersecting points of l with the sides AC and BC be Q and P respectively and let the intersecting point of l with the extention of AB be R: Without loss of generality A lies between R and B: Denote the symmetric points of H with respect to AC BC and AB by B1 A1 and C1 respectively. It is well known that A1 B1 and C1 lie on the circumcircle k of 4ABC: It is clear also that the symmetric lines of l are B1Q A1P and C1R: Let B1Q \ A1P = S and B1Q \ C1R = T: We obtain < ) CB1Q+ < ) CA1P =< ) CHQ+ < ) CHP = 180 and therefore SA1CB1 is inscribed. Thus, S = k \B1Q: From 15

the other hand < ) RC1A =< ) RHA =< ) AB1T and therefore B1ATC1 is inscribed. Hence, T = k \ B1Q: This implies that T S which shows that the three lines intersect in a point. Problem 9b.3 See Problem 8.3. p 3 Problem 10.1 Solve the equation px + p x + 7 = 4 x + 80: Solution: It is clear that x 0: Note that x = 1 is a root of the equation. We shall prove that there are no other roots. Rasp p 2 3 3 ing the p equation to the forth power wepget x + 4( x) x + 7 + p p p 3 2 3 2 6( x) ( x + 7) + 4( x)( x + 7)3 + ( 3 x + 7)4 = x + 80: px)3 p px)2(p 3 3 Let f ( x ) = 4( x + 7 + 6( x + 7)2 p p p +4( x)( 3 x + 7)3 +( 3 x + 7)4: Obviously f (x) is an increasing function. If x > 1 then it follows from the inequalities x2 > x f (x) > f (1) = 80 that x2 + f (x) > x + 80 a contradiction. If x < 1 then it follows from the inequalities x2 < x f (x) < f (1) = 80 that x2 + f (x) < x + 80 a contradiction. This completes the prove. Problem 10.2 The incircle of an isosceles 4ABC touches the legs AC and BC at points M and N respectively. A tangent t is drawn _ to the smaller of the arcs MN and let t intersects NC and MC at points P and Q respectively. Let T be the intersecting point of the lines AP and BQ: a) Prove that T lies on the segment MN b) Prove that the sum of the areas of triangles ATQ and BTP is the smallest possible when t is parallel to AB: Solution: a) Let the incircle touches AB and PQ at points R and S respectively. Let MN and SR intersect QB in points T1 and T2 respectively. Since < ) T1MQ =< ) T1NC = 180 ; < ) T1NB and < ) MT1D =< ) BT1N it follows from the Law of Sines for 4MT1D and 16

It follows now from MQ = SQ and BN = BR that QT1 = QT2 BT1 BT2 so T1 = T2: In the same manner one can prove that AP passes through the intersecting point of MN and SR: This implies that AP BQ MN and SR intersect in T: b) We have SATQ + SBPT = SABQ + SABP ; 2SABT : Since 4ABC is isosceles we get MN k AB and therefore SABT is constant. Thus, SATQ + SBPT is minimal exactly when SABQ + SABP is minimal. The BP:AB sin = latter sum equals to AQ:AB sin + 2 (AQ+BP )AB sin where is the angle to the base of the triangle. 2 Therefore it su ces to nd the minimum of AQ + BP: It is easily seen that AQ + BP = AM + BN + PQ and therefore we have to nd when PQ is minimal. Let r be the inradius and O be the center of the incircle of 4ABC: Using that PQ = r(cotg + cotg ) where =< ) OQP =< ) OPQ we obtain from ABPQ that 2 +2 +2 = r sin 360 : Therefore + = 180 ; : Thus PQ = cos( 2 ; ) + cos : It is clear now that PQ is minimal exactly when cos( ; ) = 1 so = =) PQ k AB: Problem 10.3 There are n 4 points in the plane such that the distance between any two of them is an integer. Prove that at least 1 from the distances between them are divisible by 3. 6 Solution: We show rst that the assertion from the problem is true for n = 4 i.e. for 4 points with integer distances between them at least one distance is divisible by 3. Denote the points by A B C and D (it is easy to be seen that WLOG < ) BAD =< ) BAC + < ) CAD). By the Law of Cosines for 4ABC 4ACD 4ABD we obtain BC 2 = AB 2 + AC 2 ; 2:AB:AC cos 17

DT1 = BT1 so QT1 = MQ : By analogy QT2 = SQ : 4BT1N that MD BN BT1 BN BT2 BR

CD2 = AD2 + AC 2 ; 2:AD:AC cos BD2 = AB 2 + AD2 ; 2:AB:AD cos where =< ) BAC =< ) CAD =< ) BAD = + : Suppose that all distances are integers not divisible by 3. Therefore AB 2 AC 2 AD2 BC 2 CD2 BD2 1(mod 3) and so 2:AB:AC cos 2:AD:AC cos 2:AB:AD cos 1(mod 3): 2 Thus, 2:AB:AC cos :2:AD:AC cos 4:AC :AB:AD: cos cos AC 2:AB:AD: cos cos 1(mod 3): Note that cos cos and cos are rational numbers. Moreover, r where p q and r s are relatively prime if cos = p cos = q s then p q r and s are not divisible by 3. Hence p2 q2 r2 2 s 1(mod cos = cos cos ; sin sin = cos cos ; p 23) and pq 2 ; 2 2 p s ; r : Therefore 2:AC 2:AB:AD: sin sin is divisible q s by 3 and after multiplying 2:AB:AD cos 1(mod 3) by AC 2 we obtain 2:AC 2:AB:AD: cos cos 1(mod 3) a contradiction to 2 AC :AB:AD: cos cos 1(mod 3): Therefore at least one of the distances is divisible by 3. Let n 4: Since there exist n 4 sets with four elements each there exist at least n distances (counted more than once) divisible 4 2 times and we by 3. Each such distance is counted exactly n ; 2 n n : 4 get that the desired number is at least n; =1 2 6 2 2 x2 + 4x + 3 : Problem 11.1 Let f (x) = x 2 + 7x + 14
! ! ! !

18

f (x) 2 x ; 5 x + 10 : b) Find the greatest value of the function x2 + 5x + 20 Solution: a) We shall prove that the greatest value of f (x) equals 2. Since x2 + 7x + 14 > 0 8x we obtain f (x) 2 =) (x + 5)2 0 and an equality occurs only when x = ;5: 2 ; 5x + 10 : Since x2 +5x +20 > 0 8x we obtain b) Let g(x) = x 2 x + 5x + 20 g(x) 3 =) (x + 5)2 0 and equality occurs only when x = ;5: Since x2 ; 5x +10 > 0 8x we have that the function h(x) = g(x)f (x) is correctly de ned. Further, when f (x) > 0 i.e. x 2 ;3 ;1] then h(x) 32 = 9: From the other hand g(x) 1 =) x ;1 and so h(x) 1 when f (x) 0: Therefore the greatest value of h(x) equals 9 and h(x) = 9 when x = ;5: Problem 11.2 A point A1 is chosen on the side BC of a triangle ABC such that the inradii of 4ABA1 and 4ACA1 are equal. Denote the diameters of the incircles of 4ABA1 and 4ACA1 by da: In the same manner de ne db dc: If BC = a CA = b AB = c p = a + b + c and h h h are the altitudes of the triangle ABC and d a b c 2 is the diameter of the incircle of 4ABC prove that: p(p ; a) a) da + a d = ha b) da + db + dc + p ha + hb + hc : Solution: a) Let O1 and O2 be the incenters of 4ABA1 and 4ACA1 and p1 and p2 be semiperimeters of the same triangles. We have SABC = SABA1 + SACA1 = ra:p1 + ra:p2 = rc(p1 + p2) = rc :(p + CC1) a where ra = d 2 : Therefore ra(p + AA1) = SABC : If P and Q are the touching points of the incircles of 4ABA1 and 4ACA1 with
! q

a) Find the greatest value of f (x)

19

BC then the quadrilateral O1PQO2 is rectangle and so O1O2 = PQ = PA1 + QA1 = p1 ; AB + p2 ; AC = p1 + p2 ; AC ; BC = p + AA1 ; AC ; BC: It follows from the similarity of 4IO1O2 and r ; ra =) 1 O2 4IBC where I is the incenter of 4ABC that O = BC r AA1 + p ; AC ; BC = r ; rc : We obtain the system BC r ra(p + AA1) = S AA1 + AB ; p = r ; ra Thus, AA1 = p(p ; a) and ra = c r p p p r p( p ; p ; a) = ha ; r p(p ; a) which rp = a 2 a p + p(p ; a) implies a). pp ; a pp ; b b) By a) the inequality is equivalent to a + b + pp ; c pp pp ; a pp ; b pp ; c c p d : Further p a + b + p c = p;a p;b p;c + + (p ;p b) + (p ; c) (p ; a) + (p ; c) (pp ; a) + (p ; b) p p; a + p p; b + p p; c = p p p p 2 p ; b p ; c 2 p ; ap p ; c 2 p ; a p ; b p p = d: (p ; a)(p ; b)(p ; c) Problem 11.3 See problem 10.3.
q q q q

20

XLIX

National Mathematical Olympiad Third Round, 15-16 April 2000

Problem 1. Find all value of the real parameter a such that the equation 9t ; 4a3t + 4 ; a2 = 0
has an unique root in the interval (0 1):

Solution: After the substitutions x = 3t and f (x) = x2 ; 4ax + 4 ; a2 the problem is equivalent to: nd all values of a such that the equation f (x) = 0 has an unique root in the interval (1 3): It is easily seen that if f (1) = 0 or f (3) = 0 then a = 1 a = ;5 or a = ;13 are not solutions of the problem. Therefore the equation f (x) = 0 has an unique root in the interval (1 3) when f (1):f (3) < 0 or Df = 0 2a 2 (1 3): It follows now that f (1):f (3) < 0 () (a ; 1)2(a +5)(a +13) p < 0 () a 2 (;13 ;5) or 5a2 ; 4 = 0 2a 2 (1 3)
and so a = 2 5 5 :

Problem 2. In 4ABC CH (H 2 AB ) is altitude and CM and CN (M N 2 AB ) are bisectors respectively of < ) ACH and < ) BCH: The circumcenter of 4CMN coincides with the incenter of 4ABC: Solution: Let I be the incenter of ABC: Denote by P Q and R the
common points of the incircle of ABC respectively with AB BC and CA: It follows from IP = IQ = IR and IC = IM = IN that 4IMP 4INP 4ICQ and 4ICR are congruent. Let < ) MIP = 1< < ) NIP =< ) QIC =< ) CIR = : We have that < ) MCN = 2 ) MIN = : Since CM and CN are bisectors we get < ) ACB = 2 < ) MCN = 2 : It follows now from the quadrilateral IQCR that < ) QIR+ 21 Prove that S4ABC = AN:BM 2 :

< ) QCR = 180 () 4 = 180 () = 45 : Therefore 4ABC is a right angle triangle which implies < ) BCM =< ) BMC =< ) BAC + < ) ABC and < < ) BAC ) ANC =< ) ACN =< ) ABC + 2 : Therefore BC = 2 AN:BM : BM and AC = AN and so SABC = AC:BC = 2 2

Problem 3. Let fang1 n=1 be a sequence such that a1 = 43 a2 =

142 an+1 = 3an + an;1 for n 2: Prove that: a) an and an+1 are relatively prime for all n b) for every natural number m there exist in nitely many natural numbers n such that an ; 1 and an+1 ; 1 both are divisible by m: 3an that m divides an;1: By induction m divides both a1 and a2 which is impossible since a1 and a2 are relatively prime. b) Consider the sequence fang de ned by an;1 = an+1 ; 3an for negative indices. Compute a0 = a2 ; 3a1 = 13 a;1 = a1 ; 3a0 = 4 a;2 = a0 ; 3a1 = 1 a;3 = a;1 ; 3a;2 = 1 and so on. We nd a sequences fang1 ;1 of integers such that an+1 = 3an + an;1 for every n: Since the pairs (p(mod m) q(mod m)) are nitely many we get that there exist integers r and s > r such that ar as(mod m) and ar+1 as+1(mod m): It is clear now that ar+i as+i (mod m) for every i: Therefore the sequence fang1 ;1 is periodic. Since a;3 a;2 1(mod m) there exist in nitely many natural numbers n such that both an ; 1 and an+1 ; 1 are divisible by m:

Solution: a) Suppose that there exist natural numbers n and m > 1 such that m divides both an and an+1: It follows from an;1 = an+1 ;

Problem 4. Given a convex quadrilateral ABCD such that < ) BCD


=< ) CDA: The bisector of < ) ABC intersects the segment CD in point E: Prove that < ) AEB = 90 if and only if AB = AD + BC: 22

Solution: Let < ) AEB = 90 : Since < ) CEB < 90 there exists a point F on the side AB such that < ) BEF =< ) BEC: Thus 4BCE = 4BFE which implies BF = BC and < ) BFE =< ) BCE: From the

other hand AE = AF AE = AD : ( ) sin < ) AFE sin < ) AEF sin < ) ADE sin < ) AED Since < ) AED =< ) AEF and < ) AFE + < ) ADE = 180 we obtain AF = AD and so AB = AD + BC: Conversely, let AB = AD + BC: There exists a point F on the segment AB such that AF = AD and BF = BC: Therefore 4BFE = 4BCE so < ) BFE =< ) BCE and < ) BEF =< ) BEC: It follows from ( ) and AF = AD that sin < ) AED = sin < ) AEF: Since ) AED+ < ) AEF < 180 we have < ) AED =< ) AEF and therefore < < ) AEB = 90 :

Problem 5. Prove that for any two real numbers a and b there exists a real number c 2 (0 1) such that
1 > 1: ac + b + c + 1 24 1 and let m and M be respecSolution: Consider f (x) = ax + x + 1 tively the minimum and the maximum value of the function f in the 1 0 interval 0 1]: Since f (0) = 1 f (1) = a + 1 2 and f (x) = a ; (x + 1)2 there are four cases for the di erence M ; m: 1 : Thus f 0(x) 0 for x 2 0 1] and so M ;m = f (0);f (1) = 1. a 4 1 ; a 1: 2 4 23

1 a 1: Thus d = p 1 ; 1 2 0 1] f 0(x) 0 for x 2 0 d] and 3. 4 a 0 f (x) 0 for x 2 d 1]: 1 : Since f (0) f (1) we have M ; m = f (0) ; a 3.1 1 4 2 p p 2 f (d) = (1 ; a) (1 ; 22 )2:

2. a 1: Thus f 0(x) 0 for x 2 0 1] and so M ; m = f (1) ; f (0) = 1: a; 1 2 2

a 1: Since f (1) f (0) we have M ; m = f (1) ; pa ; 1)2 1 (p2 ; 1)2: f (d) = 1 (2 2 2 1 which implies M + b > 1 or m + b < In all four cases M ; m > 12 24 1 : The assertion of the problem follows now by continuity. 24 Problem 6. Find all sets S of four points in the plane such that: for any two circles k1 and k2 having diameters with endpoints - points from S there exists a point A 2 S \ k1 \ k2: Solution: Let S = fA B C Dg: Consider the circles k1 and k2 with diameters respectively AB and CD: It follows that at least one of the angles ACB ADB CAD and CBD is right angle. Without loss of generality < ) ACB = 90 : Let k3 and k4 be circles with diameters respectively AC and BD: Since < ) ABC < 90 we obtain that the common point of k3 and k4 is one of the points A C or D: 1. Let A 2 k3 \ k4: Now < ) BAD = 90 and therefore D 2 l1 l1 ? AB A 2 l1: It is easily seen that < ) BDC < 90 and since < ) BAC < 90 < ) ABD < 90 it follows that the common point of
24

1 3.2 2

the circles with diameters AD and BC is the point C and < ) ACD = 90 : Therefore D = l1 \ BC: 2. Let C 2 k3 \ k4 : Now < ) BCD = 90 and therefore D lies on the line AC: Since < ) BDC < 90 < ) BAC < 90 and < ) ACD < 90 we obtain that the common point of the circles with diameters AD and BC is B and < ) ABD = 90 : Therefore D = l2 \ AC where l2 ? AB B 2 l2: 3. Let D 2 k3 \ k4: Now < ) ADC = 90 and therefore D 2 k3: Since < ) BAC < 90 and < ) ACD < 90 we obtain that the common point of the circles with diameters AD and BC is B or D: In the rst case < ) ABD = 90 and thus D 2 l2 from the other hand l2 \ k3 = which is a contradiction. In the second case < ) BDC = 90 and we get that D is the orthogonal projection of C on AB: In conclusion, all sets S satisfying the condition of the problem are those consisting of the vertices of a right triangle and the foot of the altitude to the hypothenusis.

25

XLIX

National Mathematical Olympiad Fourth Round, 16-17 May 2000

Problem 1. In an orthogonal coordinate system xOy a set consisiting of 2000 points Mi (xi yi) is called "good" if 0 xi 83 0 yi 1 i = 1 2 : : : 2000 and xi = 6 xj for i = 6 j: Find all natural

numbers n with the following properties: : a) For any "good" set some n of its points lie in a square of side length 1: b) There exists a "good" set such that no n + 1 of its points lie in a square of side length 1: (A point on a side of a square lies in the square).

problem. We show rst that for a "good" set some 25 points lie in a square of side length 1. All points lie in the rectangle 1 x 83 0 y 1: Divide this rectangle to 83 squares of side length 1: If some 25 points lie in one of these rectangles then we are done. Conversely, in every square there are at least 26 or at most 24 points. We prove now that there exists a square with at least 26 points and there exists a square with at most 24 points. Indeed, if a square with 26 points does not exist then the points are at most 83:24 = 1992 < 2000: If there is no square with less than 25 points then the points are 83:26 ; 82 > 2000: Further, move the square with more than 25 points towards the square with less that 25 points. Since the number of points in this square changes at most by one we get the assertion of the problem. 83 and y = 0 for i = To prove b) let x1 = 0 xi = xi;1 + 1999 i 0 2 4 : : : 2000 while yi = 1 for i = 1 3 : : : 1999: Let XY ZT be an unit square. WLOG we assume that it intersects the lines 26

Solution: We shall prove that n = 25 is the only solution of the

y = 0 and y = 1 in the points P Q and R S respectively. Then PQ+QR+RS +SP < PY +Y Q+QZ +ZR+RT +TS +SX +XP = 4:
But SP QR 1 and we get PQ + RS < 2: This implies that in the square XY ZT there are no more than 25 points. We found that n = 25 is a solution of the problem. It is unique since 24 < 2000 83 < 25:

Problem 2. Given an acute 4ABC: Prove that there exist unique

points A1 B1 and C1 on BC AC and AB respectively with the following property: Each of the points is the midpoint of the segment with ends the orthogonal projections of the other two points on the corresponding side. Prove that 4A1B1C1 is similar to the triangle formed by the medians of 4ABC:

AB intersects A1B1 at its midpoint. The same is applied for A1 and B1: Therefore the tree perpendiculars intersect at the medcenter of 4A1B1C1; the point P:

Solution: It is easy to be seen that the perpendicular through C1 to

27

Let T be the midpoint of A1B1 < ) PB1A1 = x =< ) PCA1 < ) PA1B1 = y =< ) PCB1: The Law of Sines for 4B1TP and 4A1TP gives: sin = B1T = A1T = sin sin x TP AP sin y x = sin : Further, if CN is the median in 4ABC similar where sin sin y sin sin < ) ACG = sin : Since x + y = arguments imply that sin < ) BCG sin < ) ACG+ < ) BCG = and is acute it follows that x =< ) ACG y = < ) BCG: It is clear now that CP is symmetric to the median in 4ABC through C with respect to the bisector of angle : The same is true for AP and BP: Therefore the point P is unique (and therefore A1 B1 and C1 are unique). Further, < ) B1C1A1 =< ) B1C1P + ) ABG =< ) BGK where < ) A1C1P =< ) B1AP + < ) A1BP =< ) BAG+ < K is the symmetric point of G with respect to the midpoint of BC: By analogy < ) C1A1B1 =< ) GBK and < ) A1B1C1 =< ) GKB: Therefore 4A1B1C1 is similar to the triangle formed by the medians of 4A1B1C1: 28

Problem 3. Let p 3 be a prime number and a1 a2 : : : ap;2 be a sequence of natural numbers such that p does not divide both ak and ak k ; 1 for all k = 1 2 : : : p ; 2: Prove that the product of some
elements of the sequence is congruent to 2 modulo p:

Solution: Consider the sequence 1 a1 a2 : : : ap;2: We shall prove by induction that for any i = 2 3 : : : p ; 1 there exist integers b1 b2 : : : bi each of which is a product of some elements from the above sequence and bm 6 bn(mod p) for m 6= n: Indeed, for i = 2 we can choose b1 = 1 b2 = a1(a1 6 1(mod p)): Suppose we have chosen b1 b2 : : : bi such that bm 6 bn(mod p) for m 6= n: Consider b1ai b2ai : : : biai: It is easily seen that any two of them are not congruent modulo p: Further, if bj ai for any j is congruent to bl for some l we get that b1ai b2ai : : : biai modulo p is a permutation of b1 b2 : : : bi modulo p: Thus,
(b1ai):(b2ai): : : : :(biai) b1:b2: : : ::bi(mod p) and therefore aii 1(mod p); a contradiction. It follows now that for any s = 2 3 : : : p ; 1 there exist few elements of the sequence from the condition of the problem such that their product is congruent to s modulo p:

Problem 4. Find all polynomials P (x) with real coe cients such
that for any real x:

P (x):P (x + 1) = P (x2)

Solution: We show rst that for any natural number n there exists at most one polynomial P (x) of degree n such that P (x):P (x + 1) =
29

P (x2): Indeed, if P (x) = anxn + an;1xn;1 + : : : + a1x + a0 comparing the coe cients in front of the same degrees of x in P (x):P (x + 1) and P (x2) we get an = 1 and an;1 = ; n 2 : It is easily seen that each of the subsequent coe cients is a solution of an equation of rst degree of the form 2x + b = 0 where b is a function of already chosen coe cients. Therefore, if a polynomial of degree n with the desired property exists it is unique one. We prove now that a polynomial P (x) of odd degree such that P (x):P (x + 1) = P (x2) does not exist. Let P (x) = xm(x ; 1)n :Q(x) (m or n could be zero), where Q(x) is a polynomial such that Q(0) = 6 0 and Q(1) 6= 0: A substitution gives (x + 1)m:xn:Q(x):Q(x + 1) = xm:(x + 1)n :Q(x2): If m 6= n the substitution x = 0 leeds to a contradiction. Therefore m = n and so Q(x):Q(x + 1) = Q(x2) where Q(x) is of odd degree. Therefore there exists a real number x0 6= 0 1 such that Q(x0) = 0: It is clear that x0 = ;1 since otherwise the substitution x = ;1 shows that Q(1) = 0 which is impossible. A substitution x = x0 4 gives that Q(x2 0) = 0 thus Q(x0) = 0 and so on. Since x0 6= 1 0 it are in nitely many distinct terms in the sequence follows that there k 2 2 x0 x0 : : : x0 which is impossible. Direct veri cation shows that for any even natural number n = 2k the polynomial P (x) = xk (x ; 1)k is of degree n and is a solution of the problem. Therefore all polynomials such that P (x):P (x + 1) = P (x2) are P (x) = xk (x ; 1)k :

Problem 5. Let D be the midpoint of the base AB of an isosceles acute 4ABC: A point E is chosen on AB and O is the circumcenter
30

of 4ACE: Prove that the line through D perpendicular to DO the line through E perpendicular to BC and the line through B parallel to AC intersect in a point.

respectively the projection points of F 0 and F 00 on the line AB: Since O is inner point for < ) ADC and < ) ACB < 90 we have that F 0 and 00 F lie in the interior of < ) BAC: It su ces to prove that F 0 F 00 i.e. F 0H 0 = F 00H 00: Let O0 and G0 be respectively the projection points of O on AB and of G on OO0 : Since 4DH 0F 0 040OO0D it 0 0 0 OO B H OG DH 0 follows that 0 0 = 0 : Also, < F H 0 DO 0 0 0) GOG = and so F 0H 0 = GG0 : It follows from GG0 = DO O G = DG and < ) DBG = 2 ; 90 0 BD:O D that F 0H 0 = GD = ;tg2 :O0D: Denote BC \ EF 00 = I: Since < ) CBF 00 = 180 ; 2 and BE = 2O0D it is clear that F 00H 00 = sin cos = ;O0Dtg2 = BI sin = ; BE BF 00 sin = cos(180 ;2 ) cos 2 F 0H 0: This completes the prove.

Solution: Let < ) ABC =< ) BAC = and let G be the circumcenter of 4ABC: Consider points F 0 and F 00 on the line through B parallel to AC such that OD ? DF 0 and BC ? EF 00: Denote by H 0 and H 00

c1 c2 : : : cn is called sum of a = a1 a2 : : : an and b = b1 b2 : : : bn if ci = 0 when ai = bi and ci = 1 when ai 6= bi: Let f : A ! A be a function such that f (0) = 0 and if the sequences a and b di er in exactly k terms then the sequences f (a) and f (b) di er also exactly in k terms. Prove that if a b and c are sequences from A such that a + b + c = 0 then f (a) + f (b) + f (c) = 0:

Problem 6. Let A be the set of all binary sequences of length n and let 0 2 A be the sequence with zero elements. The sequence c =

Solution: Consider the sequence e1 = 1 0 : : : 0 0 e2 = 0 1 : : : 0


31

: : : en;1 = 0 0 : : : 1 0 en = 0 0 : : : 0 1: It follows from the condi-

tion of the problem that (since f (0) = 0) for all p 1 p n there exists q 1 q n such that f (ep) = eq: It is clear also that f (ep) 6= f (eq ) for 1 p q n p = 6 q: Therefore (1)

ff (e1) f (e2) : : : f (en)g = fe1 e2 : : : eng:

Consider an arbitrary sequence a = a1 a2 : : : an with t ones. If f (ep) = eq and ap = 1 then the q;th term of the sequence f (a) is also 1 (otherwise ep and a di er at t ; 1 terms whereas f (ep) = eq and f (a) di er at t + 1 terms). By analogy if ap = 0 then the q;th term of f (a) is also 0. Finaly, consider the sequences a = a1 a2 : : : an b = b1 b2 : : : bn and c = c1 c2 : : : cn such that a + b + c = 0: This means that for every i 1 i n the sum ai + bi + ci is even number. Fix i 1 i n and let f (ei) = ej : It follows now that the j ;th terms of the sequences f (a) f (b) f (c) coincide with ai bi ci respectively and using (1) we obtain that f (a) + f (b) + f (c) = 0:

32

Union of Bulgarian Mathematicians Sava Grozdev Emil Kolev

BULGARIAN MATHEMATICAL COMPETITIONS 2001

So a, 2001

Winter mathematics competition Bourgas, 2-4 February 2001


Problem 9.1. a) Draw all points in the plane with coordinates
(x y) such that (j3x ; yj ; 3)(j3x + yj ; 3) = 0 b) Find all x and y for which (j3x ; yj ; 3)(j3x + yj ; 3) = 0 y ; f4xg = 0 ;1 x 1 (For a real number x we denote the unique number in the interval 0 1) for which x ; fxg is an integer by fxg). Solution: a) It is easy to see that these are the points on the four lines l1 : 3x ; y = 3 l2 : 3x ; y = ;3 l3 : 3x + y = 3 and l4 : 3x + y = ;3: b) There are two types of solutions: y = ;3x + 3 1. Solutions of y = f4xg ;1 x 1 y = 3x + 3 = f4xg 2. Solutions of y ;1 x 1 Denote the integer part of 4x by 4x] i.e. 4x = 4x] + f4xg: In the rst case we have ;3x + 3 = f4xg = 4x ; 4x] i.e 7x = 3 + 4x]: 3

Since the right hand side is an integer all posiible values of x are 1 2 : : : 1: Direct veri cation shows that only x = 5 y = 6 0 7 7 7 7 6 3 x = 7 y = 7 x = 1 y = 0 are solutions. In the second case 3x + 3 = f4xg = 4x ; 4x] i.e. x = 3 + 4x]: It follows that x is an integer, i.e. x = ;1 0 or 1: Direct veri cation shows that solution is only x = ;1 y = 0. Thus, there are four solutions: 5 y = 6 x = 6 y = 3 x = 1 y = 0 x = ;1 y = 0: x= 7 7 7 7

Problem 9.2. Points A1 B1 and C1 are chosen on the sides BC CA and AB of a triangle ABC: Point G is the centroid of 4ABC and Ga Gb and Gc are centroids of 4AB1C1 4BA1C1 and 4CA1B1 respectively. The centroids of 4A1B1C1 and 4GaGbGc are denoted
by G1 and G2 respectively. Prove that: a) the points G G1 and G2 lie on a straight line b) lines AGa BGb and CGc intersect in a point if and only if AA1 BB1 and CC1 intersect in a point. Solution: a) Let O be an arbitrary point and ~ 1 = OB ~ + (1 ; )OC ~ OB ~ 1 = OA ~ + (1 ; )OC ~ OA ~ 1 = OA ~ + (1 ; )OB ~ OC where 2 (0 1): The existence of and follows from the fact that A1 B1 and C1 lie on the sides BC CA and AB of 4ABC: Then we have ~ = 1 OA ~ + OB ~ + OC ~ = 1 3OA ~ + 3OB ~ + 3OC ~ OG 3 9 ~1=1 ~ ~ ~ OG 3 OA1 + OB1 + OC1 = 4

1 ( + )OA ~ + ( ; + 1)OB ~ + (2 ; ; )OC ~ : 3 ~ a = 1 (1 + + )OA ~ + (1 ; )OB ~ + (1 ; )OC ~ By analogy OG 3 ~ b = 1 OA ~ + (2 + ; )OB ~ + (1 ; )OC ~ OG 3 ~ + OB ~ + (3 ; ; )OC ~ : ~ c = 1 OA OG 3 ~ 2 = 1 OA ~ 1 + OB ~ 1 + OC ~1 = Thus, OG 3 1 (1 + 2 + 2 )OA ~ + (3 + 2 ; 2 )OB ~ + (5 ; 2 ; 2 )OC ~ : 9 ~ 1 = OG ~ 1 ; OG ~ we have Since GG ~ 1 = 1 ( + ; 1)OA ~ + ( ; )OB ~ + (1 ; ; )OC ~ : GG 3 ~ 2 = OG ~ 2 ; OG ~ = Using the same arguments we obtain GG 1 (2 + 2 ; 2)OA ~ + (2 ; 2 )OB ~ + (2 ; 2 ; 2 )OC ~ : It follows 9 ~ 1 = 3 GG ~ from the last two equalities that GG 2 2 and we are done. b) Since the lines AA1 BB1 and CC1 intersect in a point Ceva's theorem gives AC1 BA1 CB1 = 1: C1B A1C B1A Denote the intersecting points of AGa BGb and CGc with the sides BC CA and AB by A2 B2 and C2 respectively. A necessary and su cient condition for lines AGa BGb and CGc to intersect in a CA2 BC2 AB2 = 1: point is A 2 B C2A B2 C Denote the midpoint of B1C1 by A3: Let h1 and h2 be the altih i h i h i h i h i h i h i

tudes of 4AA2C and AA3B1 from A2 and A3: We have SAA2 C = h1:AC = AA2:AC : SAA3 B1 h2:AB1 AA3:AB1 By analogy SAA2 B = AA2:AB : S AA :AC
AA3 C1
3 1

Dividing the above equalities and using that SAA3 C1 = SAA3 B1 and SAA2 C = CA2 we obtain S BA
AA2 B
2

CA2 = AC AC1 : A2B AB AB1


By analogy

BC2 = CB CB1 AB2 = BA BA1 : C2A CA CA1 B2C BC BC1 Multiplying the above equalities gives CA2 BC2 AB2 = AC AC1 BC CB1 BA BA1 = AC1 BA1 CB1 : A2B C2A B2C AB AB1 AC CA1 BC BC1 C1B A1C B1A
Therefore the lines AGa BGb and CGc intersect in a point i the lines AA1 BB1 and CC1 interect in a point.

Problem 9.3. Let An be the number of sequences from 0's and 1's

of length n such that no four consequtive elements equal 0101: Find the parity of A2001: Solution: Denote the number of sequences of length n, the last three terms of which are ijk where i j k 2 f0 1g and no four consequtive elements equal to 0101 by an ijk : 6

+1 n+1 n n n Obviously an ijk = a0ij + a1ij when ijk 6= 101 and a101 = a110: Adding the above equalities for all values of ijk we obtain An+1 = 2An ; an 010: ;1 n;1 ;2 n;2 n;2 Therefore An+1 an an an 010 001 + a101 000 + a100 + a110 n ; 4 n ; 4 n ; 4 n ; 5 n ; 5 ;5 n ; 3 n ; 3 n ; 4 a000 + a100 a000 + a100 + a010 + a110 a000 + a001 + an 010 + ;5 n;5 n;5 n;5 n;5 an An;5(mod 2): 100 + a111 + a110 + a101 + a011 + Thus, Ak+6m Ak (mod 2) for all k and m: Therefore A2001 A3 = 8 0(mod 2):

Problem 10a.1. Find all pairs (a b) of integers such that the system x2 + 2ax ; 3a ; 1 = 0 y2 ; 2by + x = 0
has exactly three real solutions. Solution: Let (a b) satisfy the condition of the problem. Since there are exactly three real solutions the rst equation has two distinct roots x1 < x2 which implies D1 = 4(a2 + 3a + 1) > 0: The discriminant of the second equation in respect to y equals D2 = 4(b2 ; x) and therefore there are exactly three real solutions i x1 < x2 = b2: These conditions are satis ed p i a and b are integers such that 2 2 a + 3a + 1 > 0 and b = ;a + a2 + 3a + 1: It follows from the latter equality that a2 + 3a + 1 = c2 where c is a positive integer. Therefore the discriminant of a2 +3a +1 ; c2 = 0 is a perfect square, i.e. 9 ; 4(1 ; c2) = d2 where d is nonnegative integer. The last equality can be written in the form (d ; 2c)(d +2c) = 5 which implies that d ; 2c = 1 d + 2c = 5 i.e. d = 3 c = 1: Hence, a2 + 3a + 1 = 1 with roots a = 0 and a = ;3: Respectively b = 1 or b = 2: Direct veri cation shows that all pairs (a b) = (0 1) (0 ;1) (;3 2) and (;3 ;2) satisfy the condition of the problem. 7

through C at points D and E respectively. Prove that if the intersecting point of the tangents to k through C and E lies on the line BD then < ) ABC = 90 : Solution: Let F be the intersecting point of the tangents tC and tE to k at C and E G = BD \ CE and ACBH is parallelogram. CD : We have that < ) ABC = 90 () tC k AB () FD = FB CA CD GD CD From the other hand CA = BH = GB i.e. it su ces to show that FD = GD ( ): Since 4FBE 4FED and 4FBC 4FCD FB GB BE and FB = BC : Further, FC 2 = FE 2 = we obtain FB = FE ED FC CD FD CD:ED FB:FD and so FB = CB:EB : Now the equality from ( ) follows SCED = GD which completes the proof. from CD:ED = CB:EB SCEB GB Problem 10a.3. Ivan and Peter alternatively write down 0 or 1 until each of them has written 2001 digits. Peter is a winner if the number, which binary representation has been obtained, cannot be expressed as a sum of two perfect squares. Prove that Peter has a winning strategy. Solution: First we prove that if the binary representation of a positive integer ends with two ones and even number of zeroes then this integer cannot be represented as sum of two squares. Indeed, such a number is of the form 4k (4s + 3) and if we suppose that x2 + y2 = 4k (4s + 3) then using the fact that x2 + y2 equals 0 modulo 4 i x and y are even we get that there exist integers p q for which p2 + q2 = 4s + 3 which is a contradiction. 8

Problem 10a.2. The tangential point of a circle k through the vertex C of a 4ABC and the line AB is the vertex B . The circle k intersects for a second time the side AC snd the median of 4ABC

The winning strategy of Peter could be: If one of Ivan's digits is 1 then Peter simply repeats all digits written by Ivan. The nal number is of the form 4k (4s + 3) and cannot be written as x2 + y2: If all Ivan's digits are zeroes then the rst three digits of Peter are 1 1 1 after which he writes only zeroes. The nal number is (0101010 : : : 0)2 = 21:41998 and cannot be represented as p2 + q2 since 21 cannot be written in that form.

Problem 10b.1. Find all values of the real parameter a such that
the equation has unique solution. Solution: The equation is equivalent to (x2 + x + a)2 = x4 x > 0 x 6= 1: Further, (x2 + x + a)2 = x4 () p (x + a)(2x2 + x + a) = 0 with roots x1 = ;a and x2=3 = ;1 4 1 ; 8a provided p1 ; 8a ; 1 ; 1 ; 8a 0: Since < 0 we obtain that only x1 = 4 p ;a x2 = ;1 + 4 1 ; 8a can be roots of the equation from the problem. If a 0 then x1 0 and x2 0 which implies that the equation has no roots. If a < 0 then x1 > 0 and x2 > 0: Note that x1 = x2 implies a = 0 which is a contradiction. Therefore if the equation has unique solution then it is necessary to have a < 0 and one of the two roots equals 1. Thus, x1 = 1 ) a = ;1 and x2 = 1 ) a = ;3: logx (x2 + x + a)2 = 4

Problem 10b.2. On each side of a right isosceles triangle with

legs of length 1 is chosen a point such that the triangle formed from these three points is a right triangle. What is the least value of the hypotenuse of this triangle? 9

is right triangle. 1. Suppose < ) B1A1C1 = 90 : Assume that the circle k with diameter B1C1 intersects BC at point X 6= A1: It is easy to be seen that X is an interior point for the line segment BC: Draw a tangent l to k which is parallel to BC and the tangential point Y belongs _ to the smaller of the arcs A1X : Denote the intersecting points of l with AB and AC by C2 and B2 respectively. Consider a homothety of center A such that the image of B2(C2) is C (B ): It is clear that the image of 4B1C1Y is inscribed in 4ABC and its hypotenuse is less than B1C1: Therefore wlog we may assume that BC is tangent to k: Thus, if < ) C1B1A1 = then < ) C1A1B = : From the B + 45 ) = Sine Low for 4BC1A1 we obtain BA1 = 1C1 sinsinsin( 45 B1C1 sin (sin + cos ): Hence (1) 1 = BA1 + A1C = B1C1 cos sin + B1C1 sin (sin + cos ) Therefore 1 = B1C1(sin2 + sin 2 ) () B1C1(1 + 2 sin 2 ; cos 2 ) = 2: From the other hand 2 sin 2 ; cos 2 = p 2 1 cos 2 = p5 sin(2 ; ) p5 where is 5 p sin 2 ; p 5 5 2 : Note that the equality holds 1 such that sin = p and cos = p 5 5 when 2 ; = 90 and since < 90 we have that < 90 : p Further B C (1 + 5) B1C1(1 + 2 sin 2 ; cos 2p) = 2 and 1 1 p 5 ; 1 : It is easily seen that if B C = 5 ; 1 and so B1C1 1 1 2 2 2 ; = 90 p then (1) holds true and therefore there exists triangle with B1C1 = 52; 1 : 2. Suppose < ) A1C1B1 = 90 : Denote the projection of A1 and
!

Solution: Consider a right isosceles triangle ABC with right angle at C . Let A1 2 BC B1 2 CA and C1 2 AB be such that 4A1B1C1

10

B1 on AB by A2 and B2 respectively. Obviously AB2 = B1B2 and BA2 = A1A2: Let P be the midpoint of A1B1 and Q { the midpoint of A2B2: Thus, A1B1 = 2PC1 2PQ = A1A2 + B1B2 and A1B1 A2B2 = AB ; AB2 ; BA2 = AB ; B1B2 ; A1A2: Adding p p the above equalities gives 2A1B1 AB = 2 and so A1B1 22 : p p Since 22 > 52; 1 we obtain that the least possible value equals p 5 ; 1: 2

Problem 10b.3. An element x is chosen from the set A = f1 2 : : : 2n g n 3: Questions of the type: Does x belong

Solution: Lemma There exist n sets each with 2n;1 elements, the sum of the elements of each set equals 2n;2 (2n + 1) with the following property: the elements from the set f1 2 : : : 2n g get as answers distinct n;tuples from "yes" and "no". Proof: Induction by n 3. For n = 3 the sum of the elements of B is 18 and we use the sets B1 = f1 2 7 8g B2 = f1 3 6 8g and B3 = f1 4 6 7g: The table shows that the elements from A get as
answers distinct triples. 1 2 3 B1 = f1 2 7 8g + + B2 = f1 3 6 8g + - + B3 = f1 4 6 7g + - + means "yes", ; means "no". 4 + 5 6 + + 7 + + 8 + + -

to B A where the sum of the elements of B equals 2n;2 (2n + 1) are allowed? Prove that one can nd x with exactly n questions stated in advance.

11

The number of elements in each set is 22 = 23;1 : Suppose the assertion is true for some m: Therefore there exist sets B1 B2 : : : Bm each with 2m;1 elements, the sum of the elements of each set equals 2m;2 (2m +1) and every element from f1 2 : : : 2mg gets distinct m;tuple. Consider the set f1 2 : : : 2m+1g: For any i 1 i m if Bi = fa1i a2i : : : a2m;1i g let Di = fa1i a2i : : : a2m;1i a1i + 2m a2i + 2m : : : a2m;1i + 2m g: It is clear that each set Di 1 i m has exactly 2m elements and the sum of the elements of Di is equal to: 2(a1i + a2i + : : : + a2m;1i)+2m;1 :2m = 2:2m;2 (2m +1)+22m;1 = 22m;1 +2m;1 +22m;1 = 2m;1 (2m+1 + 1): It is easily seen that only the elements t and t + 2m for 1 t 2m get equal m;tuples. Let P and Q be nonintersecting sets such that jP j = jQj = 2m;1 and P Q = f1 2 : : : 2m g: Consider a set Q obtained from Q by adding 2m to each of its elements. The set Dm+1 = P Q has 2m elements and the sum of its elements is equal to s2P s + s2Q s + 2m;1 :2m = 2m;1 (2m + 1) + 22m;1 = 2m;1 (2m+1 + 1): It is clear that exactly one element of each pair (t t + 2m ) for 1 t 2m belongs to Dm+1 : Hence, we have found the desired sets D1 D2 : : : Dm+1: It is obvious now that the sets, given by the Lemma solve the problem.
P P

Problem 11.1. A sequence a1 a2 : : : an : : : is de ned by a1 = k a2 = 5k ; 2 and an+2 = 3an+1 ; 2an n 1


where k is a real number. a) Find all values of k such that the sequence fang1 n=1 is convergent. 12

b) Prove that if k = 1 then 2 n+1 ; 8an an+1 n 1 an+2 = 7a 1 + an + an+1 where x] denotes the integer part of x: Solution: a) Write the given reccurent relation in the form an+2 ; an+1 = 2(an+1 ; an) and consider the sequence cn = an+1 ; an: It is obvious that if the sequence fang1 n=1 is convergent then the 1 sequence fcngn=1 is also convergent. From the other hand it follows from c1 = a2 ; a1 = 4k ; 2 and cn+1 = 2cn that the sequence fcn g1 n=1 is an arithmetic progression and cn = (4k ; 2):2n;1 for any positive integer n: Thus, if 4k ;2 6= 0 then the sequence fcn g1 n=1 is unbounded and therefore is not convergent one. Therefore 4k ; 2 = 0 i.e. k = 1 2: 1 For this value of k all terms of the sequence fang1 n=1 are equal to 2 and therefore the sequence is convergent. b) It easily follows by induction that an = 2n ; 1: From the other hand the equality from the condition of the problem is satis ed for n = 1 2: Suppose it is true for n = m and n = m + 1: Then m+1 ; 1)2 ; 8(2m ; 1)(2m+1 ; 1) 7a2 m+1 ; 8am am+1 = 7(2 = 1 + am + am+1 1 + 2m ; 1 + 2m+1 ; 1
" # " # " #

3:22m+2 ; 2m+2 + ;1 = 2m+2 ; 1 = a m+1 3:2m ; 1 3:2m ; 1 which completes the proof.
" #

Problem 11.2. On each side of a triangle with angles 30 60 and

90 and hypotenuse 1 is chosen a point such that the triangle formed from these three points is a right triangle. What is the least value of the hypotenuse of this triangle? 13

Solution: Consider triangle ABC with angles and . Let A1 2 BC B1 2 CA and C1 2 AB be such that 4A1B1C1 is right

with < ) A1C1B1 = 90 : Suppose that the circle k of diameter A1B1 intersects AB at point X 6= C1: It is easy to see that if 90 and 90 then X is an interior point for line segment AB: Draw a tangent l to k which is parallel to AB and the tangential point _ Y belongs to the smaller of the arcs C1X : Denote the intersecting points of l and lines AC and BC by B2 and A2 respectively. Consider homothety of center C for which the image of B2(A2) is the point A(B ): It is obvious that the image of 4B1A1Y is inscribed in 4ABC and its hypotenuse is less than B1A1: Therefore wlog we may suppose that AB is tangent to k: In this case if < ) B1A1C1 = then < ) B1C1A = : The Sine Low for 4AC1B1 and 4BC1A1 gives ; + ): AC1 = B1A1 sinsinsin( + ) BC1 = B1A1 cos sin(90 sin Since AB = AC1 + C1B we obtain sin( + ) + B1A1 cos sin(90 ; + ) (1) AB = B1A1 sin sin sin This equality is equivalent to 2AB = A1B1(cotg + cotg + 2 sin 2 ; (cotg ; cotg ) cos 2 ): From the other hand 2 sin 2 ; (cotg ; cotg ) cos 2 = 4 + (cotg ; cotg )2 sin(2 ; ) 4 + (cotg ; cotg )2 where cotg ; cotg 2 sin = : cos = 4 + (cotg ; cotg )2 4 + (cotg ; cotg )2
q q q q

14

It is easily seen that such an angle always exists. Finally, the least value is 2AB B1A1 = : cotg + cotg + 4 + (cotg ; cotg )2
q

2AB cotg + cotg + 4 + (cotg ; cotg )2 and 2 ; = 90 then (1) holds true and therefore there exists a 2AB triangle with B1A1 = : cotg + cotg + 4 + (cotg ; cotg )2 It remains to compute the above expression for the three possible ways to inscribe a right triangle p in triangle p of pangles 30 p 60 and 90 39 ; 3 21 ; 3 3 : The least and hypotenuse 1. We obtain 12 4 4 p p 39 ; 3 which is the answer of the problem. value is 12 Problem 11.3. The plane is divided into unit squares by lines parallel to coordinate axes of an orthogonal coordinate system. Find the number of paths of length n from the point with coordinates (0 0) to the point with coordinates (a b) moving along the sides of the unit squares. Solution: Divide the path into unit paths, i.e. paths between two neighbouring points of integer coordinates. Denote the number of moves up by x1 down by y1 right by x2 and left by y2: The condition of the problem gives x1 + x2 + y1 + y2 = n x1 ; y1 = b x2 ; y2 = a: a + b ; x y = n ; a + b ; x : Since Thus, y1 = x1 ; b x2 = n + 2 1 2 1 2 jaj + jbj = jx2 ; y2j + jx1 ; y1j x1 + x2 + y1 + y2 = n we obtain that it is necessary to have a + b n and a + b n(mod 2): Let It is clear that if B1A1 =
q q

15

us rst x the moves up and rigth. This can be done by n+a+b 2 n + a + b positions x the ways. After that in these already xed 2 a ; b positions x the moves up and nally, in the remaining n ; 2 moves left. We obtain
n+a+b
2

n+a;b
2 X

n+a+b !
2

i=b

n;a;b ! 2 n;a+b ; i :
2

cient in front i 2 of x 2 in the expansion of (1 + x) n+2a+b (1 + x) n;2a;b = (1 + x)n: Therefore the sum equals n;n a+b : The number of paths is:
2

The sum

n+a;b
2 X

n+a+b !

i=b n;a+b

n;a;b ! 2 n;a+b ; i equals to the coe


!

n+a+b
2

n;a+b
2

:
! !

Answer: If jaj + jbj > n or a + b 6 n(mod 2) the number of n paths is 0, otherwise the number of paths is n+n a+b n;a+b :
2 2

16

Spring mathematics tournament Kazanlak, 30 March - 1 April 2001


Problem 8.1. Let a be a real parameter such that 0 a 1: Prove
that the solutions of the inequality 1j 1 jxj + jax + 2

form an interval of length greater or equal to 1. 1 of Solution: If a = 0 then all numbers from the interval 1 ; 2 2 length 1 are solutions of the problem. If a = 1 then the inequality becomes jxj + jx + 1 2 j 1: The solutions of this inequality are all numbers from the interval ; 3 1 which is of length 1. 4 4 Further, assume that 0 < a < 1 and let x 0: In this case 1 : Let x < 0: 1 > 0 and therefore the solutions are x ax + 2 2(1 + a) 1 1 If ax + 2 0 i.e. x ; 2 the solutions satisfy x ; 2(1 1 ; a) : If 1 < 0 i.e. x < ; 1 the solutions satisfy x ; 1 : ax + 2 2a 3(1 + a)

1 1 and so all numbers x Let a 1 : Then ; ; 2 2(1 ; a) 2a 1 1 > ; ; 2(1 ; a) are solutions of the inequality. Since ; 3(1 1 + a) 2a it has no other solutions. Therefore all solutions form an interval 1 1 + 1 = 1 > 1: ; 2(1 1 of length ; a) 2(1 + a) 2(1 + a) 2(1 ; a) 1 ; a2 3 < ; 1 and the solutions are x : Then ; Let a > 1 2 2(1 + a) 2a
" #

17

1 or x 2 ; 3 ; 1 : Therefore all sox ; ; 2(1 3 + a) 2a 2(1 + a) 2a 1 2 lutions form an interval ; 2(1 3 of length + a) 2(1 + a) 1 + a 1:
" #! " #

Problem 8.2. Given a square ABCD of side length 1. Point M 2 BC and point N 2 CD are such that the perimeter of 4MCN
is 2. a) Find < ) MAN b) If P is the foot of the perpendicular from A to MN nd the locus of the point P:

) KAN: Furthermore < ) KAB =< ) NAD which im< ) MAN = 1 2< plies that < ) KAN =< ) BAD = 90 : Therefore < ) MAN = 45 : b) It follows from 4AMN = 4AMK that < ) AMN =< ) AMK: Thus, 4APM = 4ABM and AP = AB = 1: Therefore P lies on a circle of center A and radius 1: Finally, the locus of P is an arc of a circle with center A and radius 1 excluding points B and D:

Solution: a) Let K be a point on the extension of CB such that BK = DN: Then MK = MB + BK = MB + DN = 1 ; CM + 1 ; CN = 2 ; (CM + CN ) = MN (using that CM + CN + MN = 2). Since 4ABK = 4ADN we have that AK = AN: Therefore 4AMN = 4AMK and it follows that < ) MAN =< ) MAK i.e.

Problem 8.3. a) Prove that


12 + 22 + holds true for any positive integer n: 18

n + 1) + n2 = n(n + 1)(2 6

b) Find the least integer n n > 1 for which 12 + 22 + + n2 n is a perfect square.

Solution:
a) Direct veri cation shows that n(n + 1)(2n + 1) + (n + 1)2 = (n + 1)(n + 2)(2n + 3) 6 6 for any integer n: Using this equality the proof is easily done by induction. b) It follows from a) that n(n + 1)(2n + 1) = 6m2: Since 2n + 1 is odd then n + 1 is even, i.e. n is odd. Let n = 2k ; 1: Then k(4k ; 1) = 3m2: Therefore 3=k or 3=4k ; 1 i.e. 3=k or 3=k ; 1: Let k = 3l: Then l(12l ; 1) = m2: Since (l 12l ; 1) = 1 we have l = n2 and 12l;1 = v2: The latter equality is impossible (contradiction both modulo 4 and modulo 3). Let k = 3l +1: Then (3l +1)(4l +1) = m2: Since (3l + 1 4l + 1) = 1 we get that 3l + 1 = u2 4l + 1 = v2 u > 1 v > 1: Verifying consequently for v = 3 5 7 9 11 13 : : : we obtain l = 2 6 12 20 30 42 56 : : : : Thus, 3l + 1 = 7 19 37 61 91 127 169 ( rst perfect square). So, the least n is 2k ; 1 where k = 3l + 1 = 3:56 + 1 = 169 and therefore n = 337: a) Find all values of a for which the equation f (x) = 0 has at least one real root. b) If x1 and x2 are the real roots of f (x) = 0 (not necessarily distinct) nd the least value of the expression a ; 4a2 ; 70a3 + 1 A = (1 +9x (1 ; 6a ; x1)(1 ; 6a ; x2) : 1 )(1 + x2 ) 19

Problem 9.1. Let f (x) = x2 + 6ax ; a where a is a real parameter.

the equation f (x) = 0 has at least one real root i 9a2 + a 0, so 1 0 +1). giving a 2 ;1 ; 9 b) Since f (x) = (x ; x1)(x ; x2) we obtain f (;1) = (;1 ; x1)(;1 ; x2) = (1 + x1)(1 + x2) = (;1)2 + 6a(;1) ; a = 1 ; 7a and f (1 ; 6a) = (1 ; 6a ; x1)(1 ; 6a ; x2) = (1 ; 6a)2 + 6a(1 ; 6a) ; a = 1 ; 7a: Therefore the denominators of the two fractions of A are equal to 1 ; 7a and 3 4a2 + 9a ; 1 = (1 ; 7a)(10a2 + 2a ; 1) = 10a2+2a;1: A = ;70a ; 1 ; 7a 1 ; 7a The quadratic function g(a) = 10a2 + 2a ; 1 attains its minimal 1 62 ;1 ; 1 0 +1): Therefore the minimal value for a = ; 10 9 value of A equals to the smallest of the numbers g(0) and g ; 1 , 9 89 1 i.e. this value is g ; 9 = ; 81 .

Solution: a) The discriminant of f (x) is D = 4(9a2 + a). Therefore

Problem 9.2. Given a convex quadrilateral ABCD such that

OB:OD where O is the intersecting point of its diagoOA = OC + OD nals. The circumcircle of 4ABC intersects the line BD at point Q: Prove that CQ is the bisector of < ) DCA: Solution: Let CQ1 Q1 2 BD be the bisector of < ) DCO: Therefore DQ1 = DC Q1O CO This equality, combined with the condition of the problem gives + DQ1 CD = OB:OD OA(OC + OD) = OB:OD () OA Q1O Q1O
20

Therefore the quadrilateral ABCQ1 is cyclic. Thus, Q1 Q: integers such that non of them can be written in the form j7x2 + 9xy ; 5y2j, where x and y are integers.

DO = OB:OD () OA:CO = Q O:OB: () OA:CO: Q 1 1O

Problem 9.3. Prove that there exist eight consecutive positive


f (0 1) = 5, f (1 0) = 7, f (1 1) = 11 and f (0 2) = 20, rst possible sequence of eight positive integers is 12 13 : : : 19. We shall prove that non of these integers can be written in the form j7x2 +9xy ;5y2j, where x and y are integers. Let f (x y) = k, where x and y are integers. It su ces to prove that f (x y) = k has no solutions for k 2 f12 13 : : : 19g. Suppose k is even. Then x and y are also even. If x = 2x1 and y = 2y1 we get the equality 4f (x1 y1) = k which implies that k is divisible by 4. Thus, k 6= 14 and k 6= 18. Let k = 16 and consider the equation 4f (x1 y1) = 16 which is equivalent to f (x1 y1) = 4. As above we conclude that x1 and y1 are both even and let x1 = 2x2 and y1 = 2y2. Therefore f (x y) = 1. By analogy if k = 12 we get the equation f (x y) = 3. Multiply the equation f (x y) = k by 28 and write it in the form (14x + 9y)2 ; 221y2 = 28k: Since 221 = 13:17 it is appropriate to consider modules 13 and 17. Denote t = 14x + 9y and consider all possibilities for k, i.e. k 2 f1 3 13 15 17 19g. 1) If k = 13 then t2 28:13 (mod 17) and so t2 7 (mod 17). Raising this congruence to 8-th power gives t16 ( 7)8 ;1
21

Solution: Denote f (x y) = 7x2 + 9xy ; 5y2. Since f (0 0) = 0,

(mod 17) which is a contradiction to Fermat's theorem. Therefore k 6= 13. 2) If k = 15 then t2 28:15 5 (mod 17). Raising in 8-th 16 8 power gives t ( 5) ;1 (mod 17), a contradiction. The cases k = 17 19 1 3 are treated similarly.

Problem 10a.1. Let a and b be positive numbers such that both of the equations (a + b ; x)2 = a ; b and (ab + 1 ; x)2 = ab ; 1 have
two distinct real roots. Prove that if the two bigger roots are equal then the two smaller roots are also equal. equations have two distinct roots i a > b and ab > 1: Since a > 0 1 and it follows that a2 > ab > 1 i.e. a > 1: Therefore a > b > a a > 1: It follows from a+b+ p p the condition p of the problem thatp a ; b = ab + 1 + ab p ; 1 i.e. pa ; b = (a ; 1)(b ; 1) + ab ; 1: If a > b 1 then a ; b ab ; 1 () (a + 1)(b ; 1) 0 () b 1: Therefore b = 1: Conversely if 1 < b 1 then a p p a;b ab ; 1 () (a + 1)(b ; 1) 0 () b 1 i.e. b = 1: Thus, the two bigger roots are equal i b = 1 and a > 1: In this case the two smaller roots are also equal. that if < ) A1EC = 90 and the points A B A1 E lie on a circle, then AA1 = BA1:

Solution: Let a and b satisfy the condition of the problem. Both

Problem 10a.2. Let A1 and B1 be points respectively on the sides BC and AC of 4ABC D = AA1 \ BB1 and E = A1B1 \ CD: Prove Solution: Let F = AE \ BC: We prove that EA1 is the bisector of < ) BEF which solves the problem. Indeed, then we have < ) BAA1 =< ) BEA1 =< ) FEA1 =< ) ABA1 i.e. AA1 = BA1:
22

Using Ceva's and Menelaus's theorems for 4AA1C we obtain AD A1F CB1 = 1 AD A1B CB1 = 1 and so A1D CF AB1 A1D CB AB1 A1F = CF (1) A1B CB Let B 0 be a point on the ray A1B ! such that < ) B 0EA1 =< ) A1EF: 0 Then EA1 is the bisector of < ) B EF and since < ) A1EC = 90 it follows that EC is the external bisector of the same angle. Therefore A1F = CF and it follows from (1) that A1B = A1B 0 i.e. B = B 0 A1B 0 CB 0 CB CB 0 which completes the proof.

Problem 10a.3. Find all positive integers x and y such that x3 + y3 ; x2y2
(x + y)2 is a nonnegative integer.

(x + y)2 is a nonnegative integer. Substitute a = x + y and b = xy and write the expression in the form b2 +3ab ; a2(a ; z) = 0: The discriminant of this quadratic equation a2(4a + 9 ; 4z) is a perfect square, so (4a + 9 ; 4z) = (2t + 1)2: Thus, a = t2 + t + z ; 2 and from the equation for b we obtain that b = a(t ; 1): Since t 2 we have (x ; y)2 = a2 ; 4a(t ; 1) < (a ; 2(t ; 1))2: From the other hand a t2 and therefore a2 ; 4a(t ; 1) (a ; 2(t ; 1) ; 2)2: Since a2 ; 4a(t ; 1) 6= (a ; 2(t ; 1) ; 1)2 (the two numbers are of di erent parity) it follows that (x ; y)2 = a2 ; 4a(t ; 1) = (a ; 2(t ; 1) ; 2)2: 23

Solution: Let x and y be positive integers such that 3 3 2 2 z = x +y ;x y

Thus, a = t2 i.e. t + z = 2 and so t = 2 z = 0 which implies a = b = 4: Therefore x = y = 2 and these are the only positive integers satisfying the condition of the problem.

Problem 10b.1. Solve the equation: p 3log (cos x+sin x)+ ; 2log (cos x;sin x) = 2:
3 1 2 2

equation becomes p p p ( 3 ; 1) cos x + ( 3 + 1) sin x = 2: This equation is equivalent to p p p p 6 ; 2 cos x + 6 + 2 sin x = 1 : 4 4 2 p p p p 2 2 and sin 15 = 6; we have Since cos 15 = 6+ 4 4 sin(x + 15 ) = 1 2: From the roots of the later equation only x = 15 k360 are admissible. Therefore the roots are x = 15 k360 :

Solution: Note that the admissible values are those x for which cos x + sin x > 0 cos x ; sin x > 0: After simple calculations the

Problem 10b.2. Given a triangle ABC: Let M be such an interior point of < ) BAC that < ) MAB =< ) MCA and < ) MAC =< ) MBA: Analogously, let N be such an interior point of < ) ABC that < ) NBA =< ) NCB and < ) NBC =< ) NAB and let P be such an inte) PAC: rior point of < ) ACB that < ) PCA =< ) PBC and < ) PCB =< Prove that lines AM BN and CP intersect in a point on circumcircle of 4MNP:
24

larly. Denote the intersecting point of line CP and line segment AB by C1. Since PC1 is the bisector of < ) APB (< ) APC1 =< ) BPC1 = AP AC 1 =< ) ACB ) we get BC = BP : Triangles APC and BPC are sim1 AP AC BC =) AP = AC 2 = b2 : ilar which gives CP = BC and BP = CP AC BP BC 2 a2 By analogy, if A1 and B1 are the intersecting points of AM and 2 2 BA c CB a 1 1 BN with BC and AC respectively then A C = b2 and B A = c2 : 1 1 AC 1 :BA1:CA1 Thus, C B:A C:B A = 1 i.e. according to Ceva's theorem lines 1 1 1 AM BN and CP intersect in a point. Dnote this point by K: Since < ) APB =< ) AOB = 2 where O is the circumcenter of 4ABC we _ get that A P O and B lie on a circle and PC1 intersects the arc AB at its midpoint C1. Therefore OC2 is a diameter of this circle and < ) OPC2 =< ) OPC = 90 : It easily follows now that M N and P lie on a circle of diameter OK: alphabet of two letters a and b: Denote by QP the set of all words over the same alphabet which do not contain as subwords the words from P . Prove that: a) if QP is nite then it does not contain words of length 11 b) there exists a set P such that QP is nite and it contains word of length 10.

Solution: Suppose < ) C < 90 : The case < ) C > 90 is treated simi-

Problem 10b.3. Consider a set P of six four-letter words over an

Solution: a) Suppose that QP contains a word of length 11. We

will show that QP contains words of any length. Let ! be a word of length 11. There are 9 subwords of ! of length 3. Since there are 8 distinct words of length 3 it follows that there exists a word that appears as subword of ! twise. Let = 1 2 3 where i 2 fa bg: 25

Consider the subword of ! obtained after the second appearance of i.e. consider the word

:::

1 2 3 1

:::

1 2 3

Write 1 after 3. Obviously the new word does not contain subwords from P: ::: 2 3 1 2::: 2 3 1 By analogy write 2 and so on. Thus, we nd words of any length without subwords from P: This contradiction shows that there are no words of length 11 in QP . b) A direct veri cation shows that the set

P = f0000 1000 1001 1010 1101 1111 g


is such that QP is nite and 0001011100 2 QP :

Problem 11.1. Prove that there exist unique numbers and


such that cos =
2

tg = 1 and 0 < < < 1:

Solution: The function f (x) = cos x ; x2 is continuous in the interval (0 1) f (0) = 1 > 0 and f (1) = cos 1 ; 1 < 0: Therefore there

exists such that cos = 2: In the interval (0 1) the function cos x is decreasing one, whereas the function x2 is increasing. Therefore there exists unique 2 (0 1) such that cos = 2: We show now that in the interval 0 1] there exists a unique such that g( ) = 0 where g(x) = xtgx ; 1. The function g(x) is increasing one because xtgx is increasing as product of two increasing functions. Moreover tg 4 = 4 ; 1 < 0 g(1) = tg1 ; 1 = tg1 ; tg 4 > 0 and the uniquennes of follows. Since sin x < x for positive 26

x it follows that g( ) = tg ; 1 = sin ; 1 < 0: Further, g(x) is increasing in the interval 0 1] and so < :

Problem 11.2. Let AA1 and BB1 be the altituds of obtuse nonisosceles 4ABC and O and O1 are circumcenters of 4ABC and 4A1B1C respectively. A line through C intersects the line segments Solution: Let F and F1 be the feet of the perpendiculars from O and O1 to CD: Since CO1 ? AB we have
(1) EO1 = CF1 = CO1 cos < ) O1CF1 = CO1 sin < ) BDC EO CF CO cos < ) OCF CO sin(< ) DCB + < ) BAC ) : From the other hand the Low of Sine's for 4A1B1C 4ABC 4A1D1C and 4BDC gives A1C 1C = (2) CO1 = 2 sin A < ) A1B1C 2 sin < ) ABC (3) (4) CD1 = CA1 (5)

AB and A1B1 at points D and D1 respectively and E is point on CD1 : 1 = the line OO1 such that < ) ECD = 90 : Prove that EO EO CD

CO = 2 sin BC < ) BAC


) BAC sin < ) B1A1C = CA1 sin < sin(< ) B1A1C + < ) D1 CA1) sin(< ) BAC + < ) DCB ) sin < ) ABC CD = BC sin < ) BDC 27

It follows now from (1),(2),(3),(4) and (5) that sin < ) BAC sin < ) BDC CD1 EO1 = A1C = EO BC sin < ) ABC sin(< ) BAC + < ) DCB ) CD which completes the proof.

Problem 11.3. There are 2001 towns in a country every one of

which is connected with at least 1600 towns by direct bus line. Find the largest n for which there exist n towns any two of which are connected by direct bus line.

Solution: Let S1 snd S1 be two towns connected by direct bus line.


If k is the number of towns connected to both S1 and S2 by bus line then (1599 ; k)+(1599 ; k)+ k 1999 which implies that k 1198: Therefore there exists town S3 connected to both S1 and S2: Further, let k be the number of towns connected to all S1 S2 and S3: Therefore (1197 ; k) + (1598 ; k) + k 1998 so giving k 797: Therefore there exists town S4 connected to all S1 S2 and S3: By analogy let k be the number of towns connected to all S1 S2 S3 and S4: We have (796 ; k) + (1597 ; k ) + k 1997 which implies k 396: Therefore there exists S5 connected to all S1 S2 S3 and S4: For the number n from the condition of the problem we obtain n 5: We show that n = 5: For, number the towns by S1 S2 : : : S2001 and connect Sk and Sm with direct line for all k and m for which k 6 m(mod5): Since 2001 5 = 400 we have that each town is connested to 1600 or 1601 other towns, i.e. the condition of the problem is satis ed. For arbitrary 6 towns the numbers of at least two are equal modulo 5 and therefore they are not connected to each other. Therefore n < 6 and so n = 5: 28

L National Mathematics Olympiad 3rd round, 28-29 April 2001


Problem 1. For which values of the real parameter a the equation lg(4x2 ; (8a ; 1)x +5a2 )+ x2 +(1 ; 2a)x +2a2 = lg(x2 ; 2(a +1)x ; a2 )
has exactly one root?

Solution: Write the equation in the form:


2 1)x + 5a2 = lg(4x2 ; (8a ; 1)x + 5a2) + 4x ; (8a ; 3 2 2 x ; 2( a + 1) x;a : 2 2 lg(x ; 2(a + 1)x ; a ) + 3 Since lg t + t is an increasing function this equality is equivalent 3 to

4x2 ; (8a ; 1)x + 5a2 = x2 ; 2(a + 1)x ; a2 x2 ; 2(a + 1)x ; a2 > 0 The equation in the above system is equivalent to f (x) = 0 where f (x) = x2 ; (2a ; 1)x + 2a2: Substitution x2 = 2ax ; x ; 2a2 in the inequality gives x < ;a2: Therefore we have to nd those a for which the equation f (x) = x2 ; (2a ; 1)x + 2a2 = 0 has exactly one root such that x < ;a2: There are three cases to be considered: 1. f (x) = 0 has two roots and ;a2 is between the roots. In this case f (;a2) < 0 ) a2(a + 1)2 < 0 which is impossible. 2. f (x) = 0 has two roots one of which equals ;a2 and the second one is less than ;a2: in this case f (;a2) = 0 and so a = ;1 29

or a = 0: When a = ;1 we obtain x1 = ;2 x2 = ;1 which implies that a = ;1 is a solution. If a = 0 we get x1 = ;1 x2 = 0 which implies that a = 0 is also a solution. 3. f (x) = 0 has one root which is less than ;a2p : Since D = 0 we ; 1 2 : It is easy to get that ;4a2 ; 4a + 1 = 0 and so a1=2 = 2 check that for both values of a the corresponding root is less than ;a2: p ; 1 Therefore the solutions are a = ;1 a = 0 a = 2 2 :

Problem 2. Diagonals AC and BD of a cyclic quadrilateral ABCD

intersect in a point E: Prove that if < ) BAD = 60 and AE = 3CE then the sum of two of the sides of the quadrilateral equals the sum of the other two. Sine's we obtain AE = sin x CE = sin y AB sin(120 + y ; x) BC sin(60 + x ; y) and AB = sin(120 ; x) . Therefore 3 = AE = sin x: sin(120 ; x) . BC sin(60 ; y) CE sin y: sin(60 ; y) Hence 3(cos(2y ; 60 ) ; cos 60 ) = cos(2x ; 120 ) ; cos 120 i.e. 1 ; cos(2x ; 120 ) = 3(1 ; cos(2y ; 60 )) and so sin2(x ; 60 ) = 3 sin2(y ; 30 ). Therefore sin(x ; 60 ) cos 60 = cos 30 sin(y ; 30 ) i.e. sin x ; sin(120 ; x) = (sin y ; sin(60 ; y)): Again fom the Low of Sine's we get AD ; AB = (CD ; BC ) i.e.

Solution: Set < ) ABD = x and < ) CBD = y. From the Low of

AD + BC = AB + CD or AD + CD = AB + BC:
30

Problem 3. Find the least positive integer n such that there exists
a group of n people such that: 1. There is no group of four every two of which are friends 2. For any choice of k 1 people among which there are no friends there exists a group of three among the remaining n ; k every two of which are friends.

Solution: Consider a group of 7 people A1 A2 : : : A7 such that Ai i = 1 2 : : : 7 is not friend only with Ai+1 and Ai;1 (we set that A8 = A1 and A0 = A7). It is easily seen that there are no four every two of which are friends. Also, for any choice of k 1 people (in this case k is 1 or 2) every two of which are not friends there exists a group of three among remaining 7 ; k people every two of which are friends. Therefore k 7: We prove that for any group of 6 which satisfy the condition 1) it is possible to choose a group of k 1 every two of which are not friends such that among remaining 6 ; k a group of three every two of which are friends does not exist. Denote the people by A1 A2 : : :A6: If some of them is friend with the other 5 (wlog suppose this is A1) it is clear that there are no three among A2 A3 : : :A5 every two of which are friends. Therefore the choice of A1 solves the problem. Suppose one of then is friend with exactly four others (wlog assume A1 is friend with A2 A3 A4 and A5) Then the choice of A1 and A6 solves the problem. Therefore each person has 0 1 2 or 3 friends. It is obviuos that there exists a group of three any two of which are friends (otherwise the problem is trivial). Assume that the group A1 A2 and A3 has this property. Wlog A1 and A4 are not friends. Therefore among A2 A3 A5 and A6 there is a group of three any two of which are
31

friends. If this is A5 and A6 together with one of A2 or A3 then there exists a person who is friend with at least four others, a contradiction. Therefore wlog suppose that this group is A2 A3 and A5. Since A1 and A5 are not friends and since among the others there is a group of three friends we obtain that either A2 or A3 has at least four friends which is a contradiction. Therefore k = 7: Problem 4. Given a right triangle ABC with hypotenuse AB: A point D distinct from A and C is chosen on the ray AC ! such that the line through incenter of 4ABC parallel to the bisector of < ) ADB is tangent to the incircle of 4BCD: Prove that AD = BD: suppose the contrary, i.e. < ) ADB > 90 : Denote the tangential point of incircle k(I r) of 4BCD and the side BD by P and the tangential point of k and the line through the center J parallel to bisector l of < ) ADB by T . Since l ? DI we get T 2 DI: Thus, ) CBD =< < ) IJT = < ) IBP and since IT = r = IP we have that 2 4IJT and 4IBP are congruent. In particular IJ = IB which implies that < ) BJC < 90 a contradiction. Let A0 be such point on the ray DA! that DA0 = DB and E is the midpoint of A0B: Denote the tangential point of incircle of 4A0BC by F and its incenter by J 0. Then we have 1 jA0B ; (A0B + A0C ; BC )j = EF = jA0E ; A0F j = 2 0C j = 1 jBC ; (A0D ; CD)j = j BC ; A =1 2 2 1 jBC + CD ; BDj = r: 2 Therefore the line J 0F parallel to l is tangent to k: This implies that J 0 2 JF: From the other hand J 0 2 JC and since JC is not 32

Solution: First we show that C lies on the line segment AD: For,

1< parallel to JF (< ) ACJ = 45 > 2 ) ADB =< ) ADI ) we get J = J 0: Thus, < ) ABJ =< ) CBJ =< ) A0BJ i.e. A = A0 which completes the proof. Problem 5. Find all triples of positive integers (a b c) such that a3 + b3 + c3 is divisible by a2b b2c and c2a: a b c is Solution: If d = gcd(a b c) it is easy to see that d d d also a solution of the problem. Thus, it su ces to nd all triples (a b c) such that gcd(a b c) = 1: Let gcd(a b) = s and suppose s > 1: If p is a prime divisor of s then p divides a p divides b and p divides a3 + b3 + c3: Hence p divides c3 and so p divides c: This is a contradiction to gcd(a b c) = 1: Therefore gcd(a b) = 1 and by analogy gcd(a c) = gcd(b c) = 1: It follows from a2=(a3 + b3 + c3) b2=(a3 + b3 + c3) c2=(a3 + b3 + c3) and gcd(a2 b2) = gcd(a2 c2) = gcd(b2 c2) = 1 that a3 + b3 + c3 is divisible by a2b2c2: In particular a3 + b3 + c3 a2b2c2: Wlog we may assume that a b c: Then 2b2 a 3c3 a3 + b3 + c3 a2b2c2 ) c : Suppose a > 1: Then 3 a2b2 > b2 b2 + a(a ; b) = a2 ; ab + b2 and the inequality (1) c > 3 a2 ; ab + b2 holds. From the other hand since b a 2 we have that b2 2b a + b and since c > b2 we obtain (2) c > a + b: Now 2 b2)(a + b) < 1 () a3 + b3 < 1 which (1) and (2) give (a ; ab + c2 c2 3 +b3 a is impossible since c2 is an integer. Thus, a = 1: In this case we have that 1+ b3 + c3 is divisible by b2c2: Consider the following cases: 1) if b = c it is easy to see that b = c = 1: Indeed, (1 1 1) is a solution of the problem. 2) if b = 1 we obtain the same solution. 3) if b = 2 we obtain c = 3: Triple (1 2 3) is a solution of the
!

33

problem. Supose now that c > b 3: Since 1 + b3 + c3 b2c2 it follows 2 that 2c3 > 1+ b3 + c3 and so 2c > b2 or c > b2 : It follows from 2c > b2 2 that 2c > b2 ; b + 1 i.e. (3) b ;cb + 1 < 2: From the other hand c > b2 > b + 1 ) (4) b + 1 < 1 hold. when b 5 the inequalities 2 4 c 2 3+1 b Multiplying (3) and (4) gives c2 < 1 which is impossible since this number is an integer. Direct veri cstion shows that when b = 3 or b = 4 we get no new solutions. Therefore all solutions of the problem are triples (k k k) and (k 2k 3k) (and its permutations) for arbitrary positive integer k: Problem 6. Given a pack of 52 cards. The following opperations are allowed: 1. Swap the rst two cards 2. Put the rst card on the last place. Prove that using these opperations one can order the cards in arbitrary manner. Solution: First we show that it is possible to change any two neighbouring cards. Indeed, using 2) we can move the two cards on the rst two positions. After that apply 1) and again 2) to put all remaining cards on their initial positions. Consider two arbitrary cards ai and ai+j : We can change these two cards by changing ai and ai+1 then ai and ai+2 and so on, up to ai and ai+j . After that we change ai+j and ai+j;1 and so on, up to ai+j and ai+1: Since we can change any two cards we can order the cards in arbitrary manner. 34

L National Mathematics Olympiad 4th round, 19-20 May 2001


Problem 1. Consider the sequence fang such that a0 = 4, a1 = 22 and an ; 6an;1 + an;2 = 0 for n 2. Prove that there exist sequences 2+7 fxng and fyng of positive integers such that an = xyn ; y for any
n n

n 0.

Then xn = 3xn;1 +4yn;1 and yn = 2xn;1 +3yn;1 . Since an = xn +yn, 2 yn + 7 , i.e. x2 = 2y2 + 7. it su ces to prove that xn + yn = x n n n ; yn We prove this by induction. The assertion is obvious for n = 0. 2 Suppose that x2 n;1 = 2yn;1 + 7. Writing this equality in the form 2 (3xn;1 + 4yn;1)2 = 2(3yn;1 + 2xn;1)2 + 7 gives x2 n = 2yn + 7 which completes the proof. Problem 2. Given nonisosceles triangle ABC . Denote the tangential points of the inscribed circle k of center O with the sides AB BC and CA by C1 A1 and B1 respectively. Let AA1 \ k = A2 BB1 \ k = B2 and let A1A3, B1B3 be bisectors in triangle A1B1C1 (A3 2 B1C1 B3 2 A1C1). Prove that: a) A2A3 is bisector of < ) B1A2C1 b) if P and Q are the intersecting points of circumcircles of triangle A1A2A3 and triangle B1B2B3 then the point O lies on the line PQ. Solution: a) From the Low of Sine's we obtain AB1 = sin < ) B1A1A2 AC1 = sin < ) C1A1A2 AA1 sin 1 AA1 sin 1 35

Solution: Let xn = an +2an;1 , x0 = 3 and yn = an ;2an;1 , y0 = 1.

=< ) A1B1C1 1 = A1C1B1: Since AB1 = AC1 we get ) B1A1A2 = sin 1 = A1B1 = A3B1 A2B1 = sin < A2C1 sin < ) C1A1A2 sin 1 A1C1 A3C1 which implies that A2A3 is the bisector of < ) B1A2C1: b) Let M = AA1\BB1: It follows from MA1 : MA2 = MB1 : MB2 that M lies on the line PQ. Therefore it su ces to prove that OM ? O1O2, where O1 and O2 are circumcenters of 4A1A2A3 and 4B1B2B3. It follows from a) that the diametrically opposite point of A3 in k1 { the circumcircle of 4A1A2A3, lies on the line B1C1. Therefore O1 2 B1C1. Moreover < ) B1B3A1 =< ) CA1A3 = + 1 . 2 It easily follows now that O1 coinsides with the intersecting point of B1C1 and BC . Let OO1 \ A1A2 = N and OO2 \ B1B2 = K . It 2 follows from 4OA1O1 that ON : OO1 = OA2 1 = r and by analogy 2 OK : OO2 = r where r is the radius of k. Since O, N , M and K lie on the circle k3 of diameter OM we have that the line O1O2 is the image of k3 by inversion of center O and degree r2, i.e. OO1 ? OM . Problem 3. For a permutation a1 a2 : : : an of the numbers 1 2 : : : n it is allowed to change the places of any two consecutive blocks, i.e. from a1 : : : ai ai+1 ai+2 : : : ai+p ai+p+1 ai+p+2 : : : ai+q ai+q+1 : : : an where
1
| {z

by replacing A and B one can obtain a1 : : : ai ai+p+1 ai+p+2 : : : ai+q ai+1 ai+2 : : : ai+p ai+q+1 : : : an.
| {z

} |

{z

Find the least number of such changes after which from n n ; 1 : : : 1 one can obtain 1 2 : : : n.

} |

{z

Solution: Call the change of two blocks a move. We shall prove that the least number of moves such that from n n ; 1 : : : 1 one can
36

1 . obtain 1 2 : : : n is n + 2 Consider the number of pairs ai ai+1 such that ai < ai+1. This number is 0 in the initial permutation n n ; 1 : : : 2 1 and is n ; 1 in the nal permutation 1 2 : : : n ; 1 n. First we show that a move changes the number of pairs ai ai+1 such that ai < ai+1 at most by two. For, consider a move of the blocks a : : : b and c : : : d of the permutation : : : p a : : : b c : : :d q : : : As a result we obtain : : : p c : : : d a : : :b q : : : It is clear that at most three pairs can change the ordering. Suppose the elements in all three pairs change the ordering, i.e. from p > a, b > c and d > q we get p < c, d < a and b < q. Adding the rst three inequalities gives p + b + d > a + c + q and adding the last three implies p + b + d < a + c + q, a contradiction. Therefore a move changes the number of pairs ai ai+1 for which ai < ai+1 at most by two. It is easily seen that the rst and the last moves change this number by one. Therefore if x is desired number we have 1 . It remaines to nd 2 + 2(x ; 2) n ; 1 which implies x n + 2 n + 1 a sequence of 2 moves such that from n n ; 1 : : : 2 1 we get 1 2 : : : n ; 1 n. Let n be even number, i.e. n = 2k. Number the positions from right to left by 1 2 : : : n. First change the places of the blocks from positions 1 2 : : : k ; 1 and k k + 1. Next, change the places of the blocks from positions 2 3 : : : k and k + 1 k + 2. Third, change the blocks 3 4 : : : k +1 and k +2 k +3 and so on. On the k-th step change the blocks from positions k k +1 : : : 2k ; 2 and 2k ; 1 2k. As a result we obtain k + 1 k + 2 : : : 2k 1 2 : : : k. The 37

last change is 1 2 : : : k and k + 1 k + 2 : : : 2k. When n is odd, i.e. n = 2k + 1 the rst change is of the blocks from positions 1 2 : : : k and k + 1 k + 2 after that 2 3 : : : k + 1 and k + 2 k + 3 and so on. The last k + 1 change is 1 2 : : : k and k + 1 : : : 2k and we obtain 1 2 : : : 2k + 1. Problem 4. Let n 2 be xed integer. At any point with integer coordinates (i j ) we write i + j modulo n. Find all pairs (a b) of positive integers such that the rectangle with vertices (0 0), (a 0), (a b), (0 b) has the following properties: 1) the remainders 0 1 : : : n ; 1 written in its interior points appear equal number of times 2) the remainders 0 1 : : : n ; 1 written on its boundary appear equal number of times. Solution: Let pi and si for i = 0 1 : : : n ; 1 be the number of residues i on the sides and in the interior of the rectangle respectively. If a0 = a + kn for an integer k then the corresponding numbers for (a0 b) are p0i = pi + 2k and s0i = si + k(b ; 1). Therefore if (a b) is a solution then (a + kn b) and by analogy (a b + ln) are also solutions. Thus, wlog we may suppose that 1 a b n. If n = 2 then all possible values of (a b) are (a b) = (1 1) (1 2) (2 1) (2 2): Pair (2 2) is not a solution because there is unique interior point (s0 = 1, s1 = 0) for the rectangle. The remaining three cases give solutions. Therefore for n = 2 solutions are all pairs (a b) where either a or b is an odd number. Let n > 2 and suppose (a b) is a solution of the problem for 38

which 1 a b < n. In the vertices (0 0) and (a b) are written 0 and a + b modulo n respectively, whereas all residues 1 2 : : : a + b ; 1 appear on the boundary even number of times. (once on the boundary (1 0) : : : (a 0) (a 1) : : : (a b ; 1) and once on the boundary (0 1) : : : (0 b) (1 b) : : : (a ; 1 b)). If n does not divide a + b then 0 and the residue of a + b appear odd number of times whereas at least one of the remaining residues (n > 2) appears even number of times. Therefore n divides a + b and so a + b = n or a + b = 2n. When a + b = 2n we have that a = b = n. This pair is not a solution since the number of interior points is (n ; 1)2 and is not divisible by n. It remains to consider the case a + b = n. If a > 1 and b > 1 then the rectangle has interior points. For any such point (i j ) we have 0 < i < a, 0 < j < b, 0 < i + j < a + b = n and therefore the residue modulo n is not 0. Therefore a = 1, b = n ; 1 (or a = n ; 1, b = 1) which is a solution. Therefore for n > 2 all solutions are a = 1+ kn, b = n ; 1+ ln and a = n ; 1+ kn, b = 1+ ln, where k l = 0 1 2 : : :. Problem 5. Find all real numbers t for which there exist real numbers x y z such that 3x2 + 3xz + z2 = 1 3y2 + 3yz + z2 = 4 x2 ; xy + y2 = t:

Solution. We shall prove that the answer is t 2 3 ;2 5 1 : Let x y t satisfy the conditions of the problem. Consider four points A B C O in the plane such that AO = x BO = y CO = z and < p ) AOB = 60 < ) BOC =< ) COA = 150 : By the cosine 3 p 2 CA = p 1: theorem it follows that t > 0 AB = t BC = p 3 3 Since < ) ACB << ) AOB = 60 and < ) BAC << ) BOC = 150 we
!

39

have (1)
1 p 2 3

2 p 3 1 p p 2 2 3

p ; ( t)2
2

1 = cos < ) ACB > 2

(2)

( t)2 +
!

2p

1 p 3

2 p 3

2 p 3

= cos < ) BAC > ; 23 :

Conversely, let t 2 3 ;2 5 1 : Then we can construct a trip 1 : By (1) and 2 CA = p angle ABC such that AB = t BC = p 3 3 (2) it follows that < ) ACB < 60 and < ) ABC << ) BAC < 150 : Let kA be the circle through B and C such that the arc BC , lying in the half-plane (with respect to BC ) containing the point A is equal to 60 : Denote by kB the analogous circle trough A and C: Then it is easy to see that the second common point O of kA and kB lies in the interior of 4ABC: (Indeed, assume the contrary. If O lies in < ) ACB then < ) AOB =< ) AOC + < ) BOC = 150 + 150 = 300 which is impossible. If O lies in the opposite angle of < ) ACB then < ) AOB =< ) AOC + < ) BOC = 30 + 30 = 60 which contardicts to the inequalities < ) AOB << ) ACB < 60 : The other cases for the position of O can be rejected in the same manner.) Hence < ) AOB = 360 ; < ) AOC ; < )p BOC = 360 ; 150 ; 150 = 60 : Set x = AO y = BO z = CO 3: Applying the cosine theorem for triangles AOB BOC and COA it follows that x y z t satisfy the conditions of the problem.
!

These inequalities are equivalent to t < 1 and t + t ; 1 > 0 i.e. p 5 3 ; t2 2 1 :

40

Problem 6. Given the equation (p + 2)x2 ; (p + 1)y2 + px + (p + 2)y = 1


where p is xed prime number of the form 4k + 3: Prove that: a) If (x0 y0) is a solution of the equation where x0 and y0 are positive integers, then p divides x0 b) The given equation has in nitely many solutions (x0 y0) where x0 and y0 are positive integers.

perfect squares which is impossible since the second one is of the form 4k + 3: Let q be a common divisor of these numbers. It follows from (3) that q=x and so q=z: Since q=(p + 1)(z + x) + p we have q=p giving q = p which completes the proof. b) It su ces to prove that (3) has in nitely many solutions in positive integers. Let x = px1 and z = pz1: Then x2 1 = (z1 ; x1 )((p + 1)(z1 + x1) + 1) and therefore there exist positive integers a and b such that z1 ; x1 = a2 x1 = ab and (p + 1)(z1 + x1) + 1 = b2: It follows now that (4) (p + 2)b2 ; (p + 1)(a + b)2 = 1: Let p + 2 + p + 1 = mk p + 2 + nk p + 1 for any k = 0 1 : : : where mk and nk are positive integers. It is obvious that ( p + 2 ; p + 1)2k+1 = mk p + 2 ; nk p + 1 2 and after multiplying we obtain that (p+2)m2 k ;(p+1)nk = 1 i.e. b = mk and a + b = nk are solutions of (4): Hence, x = pmk (nk ; mk) and
q q q q q q

Solution: a) Set y ; 1 = z and write the equation in the form (3) x2 = (z ; x)((p + 1)(z + x) + p): If z ; x and (p + 1)(z + x) + p are relatively prime then they are

2k+1

41

z = pnk (nk ;mk ) are solutions of (3): The assertion of b) follows from the fact that both sequences m1 m2 : : : and n1 n2 : : : are strictly increasing.

42

SPRING MATHEMATICAL COMPETITION


1995

Grade 8.

Problem 1. Find all values of a, for which the system x + 4jyj = jxj
jyj + jx ; aj = 1
has exactly two solutions. Solution. Let (x y ) be a solution with x 0. Then y = 0 and jx ; aj = 1, i.e. x = a 1. It is obvious that when a 1 the system has two solutions with x 0, namely (a ; 1 0), (a +1 0). When ;1 a < 1 the system has only one solution (a + 1 0) with x 0. Let x < 0. Then x . Since jx ; aj 0, then 1 + x 0 or x ;2. and consequently j x ; a j = 1 + jyj = ; x 2 2 2 x x We have x ; a = 1 + 2 and x = 2(a + 1), x ; a = ;1 ; 2 . From here x = 2 3 (a ; 1). From 2 ;2 2(a + 1) < 0 we get;2 a < ;1 and from ;2 3 (a ; 1) < 0 we get ;2 a < 1. Obviously if (x y ) is a solution of the system with x < 0, then y 6= 0 and therefore (x ;y ) is a solution too. Thus, when a < ;2 the system has no solution. When a = ;2 we get 2 (a ; 1) = ;2, i.e. the system has exactly two solutions (;2 1). x = 2(a + 1) = ;2 and x = 3 2 (a ; 1) and the two values of x give solutions, thus we get four If 1 > a > ;2, then 2(a + 1) 6= 3 solutions with x < 0. Therefore the system has two solutions only when a 1, namely (a ; 1 0) and (a + 1 0) and when a = ;2, namely (;2 1). Problem 2. Let M be the midpoint of the side BC of the parallelogram ABCD, N be the common point of AM and BD, while P be the common point of AD and CN . Prove that a) AP = AD b) CP = BD i AB = AC . Solution. a) Consider 4ABC . The lines AM and BD are medians and thus N is the center of gravity. If Q is the intersection point of CP and AB , then Q is the midpoint of AB . It's easy to see that 4APQ = 4BCQ (Figure. 1), from where AP = BC = AD. b) Let AB = AC . Then 4ABC is isosceles and AM is a median in it. It is easy to see that 4NBC is isosceles and BN = CN . Analogously NA is a median and an altitude in 4PND, thus PN = DN , i.e. PC = PN + CN = DN + BN = BD (Figure 2). Let CP = BD. Trough B we draw a line BF , parallel to CP . Obviously PFBC is a parallelogram and therefore 6 CPD = 6 BFP . Since CP = BF , then 4DBF is isosceles. Consequently 1

NDA = 6 BFP = 6 CPD and DN = NP . A is the midpoint of PD and NA is perpendicular to AD and BC . We get that AM is perpendicular to BC and we deduce from here that 4ABC is isosceles, i.e. AB = AC .
Figure 1. Figure 2.

Problem 3. A convex polygon with n sides, n 4, is given. No four vertices of it lie on one and the same circle. a) Prove that there exists a circle through 3 vertices of the polygon which contains the remaining vertices in its interior. b) Prove that there exists a circle through 3 consecutive vertices of the polygon which contains the remaining vertices in its interior. Solution. a) Let AB be a side of the polygon. All the vertices lie in one of the half-planes with respect to AB . The segment AB is seen under di erent angles from the vertices which are di erent from A and B . Let C be the vertex from which AB is seen under the smallest angle. The circle we are looking for is de ned by A, B and C . b) We shall use the following two lemmas. Lemma 1. Let the segment AB be seen from the point X under the angle and from the point Y under the angle , where 0 < < < 90 . Then the radius of the circumcircle of 4ABX is greater than the radius of the circumcircle of 4ABY . Lemma 2. For each convex polygon there exist a side and a vertex from which this side is seen under an acute angle. Proof. Let Ai , Ai+1 and Ai+2 be three consecutive vertices of the polygon (Figure 3). At least one of the angles Ai+1 Ai Ai+2 and Ai Ai+2 Ai+1 is acute. This is enough for the proof.
Figure 3. Figure 4.

Let now M be the set of all pairs, that are consisted of a side and a vertex from which this side is seen under an acute angle. Let (AB , C ) be an element of M . Consider the circumcircle of 4ABC and let N be the set of all such circles. We denote by k the circle in N with the biggest radius. Let k be the circumcircle of 4A1A2 Am , where A1 A2 is a side of the polygon (Figure 2

4). We shall show that k is the circle we are looking for. Assume that there exists a vertex Ap which is outside k. Then 6 A1 Ap A2 < 6 A1Am A2 < 90 and the circumcircle of 4A1A2 Ap is from N and its radius is greater than the radius of k (Lemma 1). We get a contradiction. Consider one of the acute angles in 4A1A2 Am . Let = 6 A2 A1 Am be acute. We shall prove that Am = A3 , which means that k passes through 3 consecutive vertices of the polygon. Assume that Am 6= A3. Then A3 is situated in the way which is shown in the Figure 4. If = 6 A2 A3 Am , then + > 180 , i.e. 180 ; < < 90 . Thus 6 A2 Am A3 < 90 , which means that (A2A3 , Am ) is an element of M . Let c be the circumcircle of 4A2 A3Am and X be the intersection point of c and the segment bisector of A2 Am . Since 6 A2 XAm = 180 ; < = 6 A2 A1Am , then the radius of c is greater than the radius of k (Lemma 1). This is a contradiction.
Grade 9.

ABC . The points P and Q are orthogonal projections of M on AC and BC , while P1 and Q1 are orthogonal projections of P and Q on AB . 3. a) Prove that P1 Q1 = 4 b) Find the position of M for which the segment PQ is with the smallest length. Solution. a) We have SABC = SACM + SBCM = Figure 5. 1 (AC:MP + BC:MQ) = 1 (MP + MQ) (Figure 5). On 2 2 p p 3 the other hand SABC = 4 . Thus MP + MQ = 23 . Now from the rectangular p triangles P1MP p and MQ1Q we evaluate P1 M = 23 MP and MQ1 = 23 MQ. From p 3. here P1 Q1 = P1 M + MQ1 = 23 (MP + MQ) = 4 b) The orthogonal projection of the segment PQ on AB is the segment P1Q1 and thus PQ P1 Q1 . Therefore PQ is with minimal length when it is parallel to P1Q1. The last is true exactly when AP = BQ. We get that 4AMP = 4BMQ and hence AM = BM . So, PQ is minimal when M is the midpoint of AB . Problem 2. The quadratic function f (x) = ;x2 + 4px ; p + 1 is given. Let S be the area of the triangle with vertices at the intersection points of the parabola y = f (x) with the x-axis and the vertex of the same parabola. Find all rational p, for which S is an integer. Solution. The discriminant of f (x) is D = 4(4p2;p+1) and D > 0 for all real p. Consequently f (x) has two real roots x1 and x2, i.e. f (x) intersects the x-axis in two di erent points | A and B . The vertex C of the parabola has coordinates 2p and h = f (2p) = 4p2 ; p + 1 > 0. We
have

Problem 1. Let M be an arbitrary point on the side AB = 1 of the equilateral triangle

AB = jx1 ; x2j = (x1 ; x2 )2 = (x1 + x2 )2 ; 4x1x2 = 2 4p2 ; p + 1: 3 Now we nd S = S = AB:h = (4p2 ; p +1) 2 : Denote q = 4p2 ; p +1. Since q is rational
ABC

and q 3 = S 2 is an integer, then q is an integer too. Then 3

S is rational and S 2 = q is integer, q q

2 2 2 thus S q is integer too. Therefore q = n , where n is a positive integer, i.e. 4p ; p + 1 ; n = 0. The quadratic equation (with respect to p) has a rational root exactly when its discriminant 16n2 ; 15 is a square of a rational number. Consequently 16n2 ; 15 = m2 , and we can consider m to be a positive integer. From the equality (4n ; m)(4n + m) = 15 we get 4n ; m = 1, 4n + m = 15 or 4n ; m = 3, 4n + m = 5. From here n = 2, m = 7 or n = 1, m = 1. The rational 1 ;3. numbers we are looking for are 0 1 4 4 Problem 3. Let n be a positive integer and X be a set with n elements. Prove that a) The number of all subsets of X (X and included) is equal to 2n . b) There exist 2n;1 subsets of X each pair of which is with common element. c) There do not exist 2n;1 + 1 subsets of X , each pair of which is with common element. Solution. a) We use induction with respect to n. The base of the induction is obvious. Assume that the assertion is true for a set with n ; 1 elements and let X be with n elements. We can assume that X = f1 2 : : : ng. Let Y = f1 2 : : : n ; 1g. All subsets of X are divided into two groups: I group | those which do not contain n and II group | those, which contain n. Both groups have one and the same number of elements because each set of the II group is obtained from exactly one set of the I group by annexing n. Thus the number of the elements of X is twice greater than the number of the subsets of the I group. But the subsets of the I group are exactly the subsets of Y and according to the inductive assumption their number is 2n;1 . Thus the number of the subsets of X is 2n . b) According to a) the number of the subsets of X from the II group is 2n;1 and each pair of them has a common element | the number n. c) If A X , let A = X n A. All subsets of X are divided into pairs fA Ag and the number of these pairs is 2n;1 . Now if we have 2n;1 + 1 arbitrary subsets of X , according to the pigeonhole principle, it is not possible that they are in di erent pairs of the type fA Ag. Consequently among the given 2n + 1 subsets there exist two pairs of the type fA Ag, which obviously have no common element.

Grade 10.

Problem 1. Find all values of the real parameters p and q, for which the roots of the equations x2 ; px ; 1 = 0 and x2 ; qx ; 1 = 0 form (in a suitable order) an arithmetic progression with four members. Solution. Denote by x1 , x2 the roots of the equation x2 ; px ; 1 = 0 and by y1 , y2 the roots of x2 ; qx ; 1 = 0. It is clear that for all p and q the numbers x1 , x2, y1 , y2 are real and x1 x2 = y1 y2 = ;1. Assume that x1 < 0 < x2 and y1 < 0 < y2 . If four numbers a, b, c and d form an arithmetic progression then the numbers d c b a form an arithmetic progression too. So, we can assume that x1 , x2, y1 , y2 in a suitable order form an increasing arithmetic progression. If x1 < y1 (the case y1 < x1 is analogous) then there are two possibilities: I. The arithmetic progression is x1, y1, y2, x2. Then x1 + x2 = y1 + y2, from where p = q, i.e. x1 = y1 , x2 = y2 which is impossible. II. Thep arithmetic progression is x1, y1 , x2, y2 . Then x2 ; x1 = y2 ; y1 , from where p p2 + 4 = q2 + 4, i.e. p2 = q 2 and since p 6= q, then p = ;q 6= 0. x2 = p and therefore p2 ; pq ; 1 = 0. Since p = ;q, In the same way we have y1 = x1 + 2 2 4 2 2 2 2 2 . 2 4 2 then p = 3 and p = p . Hence (p q ) = p ; p , ; p p 3 3 3 3 3
4

Problem 2. Triangle ABC with AB = 22, BC = 19, CA = 13 is given. a) If M is the center of gravity of 4ABC , prove that AM 2 + CM 2 = BM 2 . b) Find the locus of points P from the plane of 4ABC , for which AP 2 + CP 2 = BP 2 .
1 p2b2 + 2c2 ; a2 = p105 2 1 p2a2 + 2b2 ; c2 = 8 2 2 2 1 p2a2 + 2c2 ; b2 = 13: BM = 3 mb = 3 2 Hence AM 2 + CM 2 = 105 + 64 = 169 = BM 2 . b) Let E be the midpoint of AC and D be symmetric to B with respect to E (Figure 6). We shall prove that the locus we are looking for is a circle k with center D and radius 26. For an arbitrary point P we have 4PE 2 = 2PA2 + 2PC 2 ; AC 2 and 4PE 2 = 2PB 2 + 2PD2 ; BD2, from where 2(PA2 + PC 2 ; PB 2 ) = 2PD2 ; (BD2 ; AC 2). But BD = 2BE = 39, i.e. PA2 + PC 2 ; PB 2 = PD2 ; (26)2. It follows from here that the equality PA2 + PC 2 = PB 2 is equivalent to PD = 26. c) Let the circle k intersects the line BD at the points M and N . It follows from b) that BP is minimal when P coincides with M and then BP = BM = 13. BP is maximal when P coincides with N , which gives BN = 65. Problem 3. Find the smallest positive integer n, for which there exist n di erent positive integers a1 a2 : : : an satisfying the conditions: a) the smallest common multiple of a1 a2 : : : an is 1995 b) for each i j 2 f1 2 : : : ng the numbers ai and aj have a common divisor 6= 1 c) the product a1a2 : : :an is a perfect square and is divisible by 243. Find all n-ples (a1 a2 : : : an), satisfying a), b) and c). Solution. Since 1995 = 3:5:7:19 and ai =1995, then for all i 2 AM = 2 3 ma = 3 2 CM = 2 m = c 3 3 c) Find the minimal and maximal values of BP , if AP 2 + CP 2 = BP 2 . Solution. a) We have Figure 6.

ai = 3 5 7 19
i i i

()

where the numbers i , i , i, i are equal to 0 or 1. There is no ai which is divisible by 9. We have a1 a2 : : :an = k2 , where k is a positive integer and since 243 = 35 divides k2 , then 36 divides k2 and since among the numbers ( ) there is no one which is divisible by 9, then n 6. The numbers ( ), which are divisible by 3 are 3 3:5 3:7 3:19 3:5:7 3:5:19 3:7:19 3:5:7:19 ( ) They are 8 in number. Let n = 6. Then the numbers a1 a2 : : : an are among ( ). It is easy to see that the product of any 6 numbers from ( ) is not a perfect square. Thus n 7. Let n = 7. It is not possible that all the numbers a1 a2 : : : an are divisible by 3, because in such a case 37=k2 and 38 ; = k2 . Therefore 6 numbers from a1 a2 : : : an are divisible by 3, i.e. they are from ( ) and at least one of them (for example a1) is not divisible by 3. It follows from a) that among a1 a2 : : : a7 there is at least one which is divisible by 5, at least one which is divisible by 7 and at least one which is divisible by 19. Since each pair of these numbers has a common divisor, and 3 ; = a1, then a1 must be divisible by 5:7:19, i.e. a1 = 5:7:19. At 5

last note that the product of all numbers ( ) is equal to 38 :54:74:194. The only possibility is a1a2 : : :a7 = 36 :54:74:194. From here a2 a3 : : :a7 = 36:53:73:193 and the only possibility is

fa2 a3 : : : a7g = f3 5 3 7 3 19 3 5 7 3 5 19 3 7 19g: Therefore n = 7 and fa1 a2 a3 a4 a5 a6 a7g = f3 5 3 7 3 19 3 5 7 3 5 19 3 7 19 5 7 19g:


Grade 11.

+ 1 21 + 22 + + 2n , n = 1 2 3 : : :. Prove that Problem 1. Let an = n 2n+1 1 2 n a) an+1 an for all n 3 b) the sequence fan g1 n=1 is convergent and nd its limit.
Solution. a) We have

+ 2 2 1 + 22 + an+1 = n 2n+2 1 2 From here

2n+1 = n + 2 (a + 1): +n +1 2(n + 1) n

2(an+1 ; an ) ; (an + 1) an+2 ; an+1 = (n + 2) 2( n + 1)(n + 2)

n = 1 2 3 : : ::

5 and Since an > 0 for all n, then if an+1 ; an 0, we have an+2 ; an+1 0. But a3 = 3 a4 = 5 3 , i.e. a4 ; a3 = 0. Hence a5 ; a4 0, a6 ; a5 0, : : :, an+1 ; an 0. b) The sequence a1 a2 a3 : : : is decreasing when n Figure 7. 3 and it is bounded (an > 0 for all n). Therefore this sequence is convergent. Let nlim a = a. From the !1 n n + 2 equality an+1 = 2(n + 1) (an +1) after passing to in nity we get a = 1 2 (a + 1), i.e. a = 1. Problem 2. The quadrilateral ABCD is inscribed in a circle with center O. The diagonals AC and BD intersect each other in the point M , M 6= O. The line through M which is perpendicular to OM intersects the sides AB and CD of the quadrilateral ABCD in the points X and Y , respectively. Prove that AB = CD i BX = CY . Solution. If AB = CD, then ABCD is isosceles trapezoid. Hence OM ?AD and OM ?BC , from where XY kBC and BX = CY . Let BX = CY . Denote by P and Q the midpoints of AB and CD, respectively. The quadrilaterals OPXM and OQY M are inscribed, thus 6 OPM = 6 OXM and 6 OQM = 6 OY M . We shall prove that 6 OPM = 6 OQM . The triangles ABM and DCM are congruent and 6

MP and MQ are medians in them. Therefore 4MPB 4MQC and 6 MPB = 6 MQC . Then 6 OPM = 90 ; 6 MPB = 90 ; 6 MQC = 6 OQM: (Or 6 OPM = 6 MPB ; 90 = 6 MQC ; 90 = 6 OQM .) Hence 6 OXM = 6 OY M and OX = OY . Therefore 4OXB = 4OY C . From here 6 OBA = 6 OCD and the isosceles triangles ABO and DCO are equal, i.e. AB = CD Remark. If X = P and Y = Q, then PB = QC = 1 AB = 1 CD. 2 Problem 3. Let n be a positive integer and let 2 f (x) = xn + (k + 1)xn;1 + (2k + 1)xn;2 + + ((n ; 1)k + 1)x + nk + 1: a) Prove that f (1 ; k) = n + 1. b) Prove that if n 3 and k is an integer (k = 6 0), then the equation f (x) = 0 has no integer
solution. Solution. a) Since xn;1 + 2xn;2 + + (n ; 1)x + n = (xn;1 + + x + 1) + (xn;2 + + x + 1) + + (x2 + x + 1) + (x + 1) + 1 n n;1 3 2 = x ;1 + x ;1 + + x ;1 + x ;1 + x;1 x;1 x;1 x;1 x+1 x;1 then ! n+1 ; 1 1 x n ;1 n ;2 x + 2x + + (n ; 1)x + n = x ; 1 x ; 1 ; (n + 1) n+1 (n + 1)x + n = x ; (x ; 1)2 when x 6= 1. Thus f (x) = xn + xn;1 + + x + 1 + k(xn;1 + 2xn;2 + + n) n+1 n+1 x+n = x ; 1 + k x ; (n + 1) x;1 (x ; 1)2 i.e. n+2 n+1 k(n + 1) + 1]x + kn + 1 f (x) = x + (k ; 1)x (; x ; 1)2 when x 6= 1. From here

; k) + kn + 1 = n + 1 f (1 ; k) = ; k(n + 1) + 1](1 2 k when 1 ; k 6= 1. If 1 ; k = 1, i.e. k = 0, then f (1) = n + 1. b) Let n 3 and k be an integer (k 6= 0). Assume that f (a) = 0. Obviously a 6= 0. We have an + an;1 + + a + 1 = ;k(an;1 + 2an;2 + + (n ; 1)a + n): If a = ;1, the left hand side of the equation is equal to 0 or 1, while the right hand side is 6 0). Thus a = 6 ;1. neither 0 nor 1, because jn ; (n ; 1) + (n ; 2) ; j > 1 when n 3 (k = The equation f (a) = 0 can be written in the following way: (;a ; k + 1)(an;1 + 2an;2 + + (n ; 1)a + n) = n + 1:
7

Hence rn (a) = an;1 + 2an;2 + + (n ; 1)a + n divides n + 1. If a > 1, then rn (a) > n + 1 when n 3 and this is impossible. From a 6= 0 and a 6= ;1 it follows that a ;2. We shall prove that the inequality jrn (t)j n + 2 is satis ed for all integers t ;2 and for all n 3 n+1 except t = ;2, n = 3 and t = ;2, n = 4. Since rn (;2) = (;2) + 3n + 2 , then r3(;2) = 3, 9 r4(;2) = ;2 and r5(;2) = 9, i.e. jr5(;2)j 7. From r3(t) = t2 +2t +3 it follows that jr3(t)j 5 when t ;3. Now we shall use induction. If jrn (t)j n + 2 for t ;2 and n 3, then rn+1 (t) = t:rn (t)+ n +1 and jrn+1 (t)j jtj:jrn(t)j; (n +1) 2(n +2) ; n ; 1 = n +3, i.e. jrn+1(t)j n +3. Hence jrn (t)j n + 2 when n 3 and t ;2 except the cases n = 3 t = ;2 and n = 4 t = ;2. Thus rn (a) does not divide n + 1 when n 3 and a ;2, because r3(;2) = 3, r4(;2) = ;2. For all others n 3 and a ;2 we have jrn (a)j n + 2.
Grade 12.

a) Solve the equations f (x) = x and F (x) = x. b) Solve the inequality F (x) > x. c) If a0 2 (0 1), prove that the sequence fan g1 n=0 , determined by an = f (an;1 ) for n = 1 2 : : : is convergent and nd its limit. Solution. The functions f (x) F (x) and F (x) ; x are de ned for all x 2 p 0 1]. p 5 . It is clear that a) The equation f (x) = x, i.e. 1 ; x = x has only one root = ;1 + 2 2 (0 1) and the roots of the equation F (x) = x are x1 = 0, x2 = 1, x3 = . b) In 0 ] the function F (x) ; x has a constant sign. The contrary would imply that there is 2 (0 ) such that F ( ) = and this contradicts the result from a). Analogously in 1] 1 2 (0 ), 3 2 ( 1) and the function F (x) ; x has a constant sign. On the other hand 4 p! 4 1 3 3 ; 1 + 1 5 . F 4 ; 4 > 0, F 4 ; 4 < 0. From here F (x) > x i x 2 0 2 c) Let a0 = . It follows from a) that an = for n = 0 1 2 : : : and hence the sequence is convergent and its limit is . Let now a0 < . Since f 0 (x) = ; p 1 < 0 for all x 2 0 1), 2 1;x then f (x) is decreasing. From here frac(a0) > frac( ) = , i.e. a1 > . By induction a2n 2 (0 ) and a2n+1 2 ( 1) for all n = 0 1 2 : : :. On the other hand it follows from the result of b) that for all x 2 (0 ) we have F (x) > x, while for x 2 ( 1) we have F (x) < x, respectively. Also F 0 (x) = f 0 (f (x)):f 0(x) > 0, i.e. F (x) is increasing and hence F (x) 2 (x ) if x 2 (0 ) and F (x) 2 ( x) if x 2 ( 1). By induction we get

Problem 1. The function f (x) = p1 ; x (x 1) is given. Let F (x) = f (f (x)).

a0 < a2 < a1 > a3 >

< a2n < < > a2n+1 > > :

Both sequences are convergent and let their limits be 1 and 2, respectively. We have F (a2n ) = a2n+2 and F (a2n+1 ) = a2n+3 for n = 0 1 2 : : :. The function F (x) is continuous and thus F ( 1 ) = 1 and F ( 2 ) = 2 . We get 1 = 2 = because the only solution of F (x) = x in (0 1) is . Therefore the sequence fan g1 n=0 is convergent and its limit is . The case < a0 is analogous. 8

a) Prove that AC = BC . b) If cos 6 BAC = 3 4 , nd the ratio of the radii of k and the incircle of 4ABC . Solution. a) Let I be the center of the incircle of 4ABC , N be the common point of this circle with AC and M be the midpoint of AC . Let r1 be the radius of k and r be the b ; r and from the radius of the incircle of 4ABC . From the condition it follows that IM = 2 1 2 b ; r ; r2. On the other hand AN = b + c ; a , i.e. rectangular 4INM we get MN 2 = 2 1 2 j c ; a j MN = jAM ; AN j = 2 . Therefore (c ; a)2 = (b ; 2r1)2 ; 4r2 : (1) (c ; b)2 = (a ; 2r1)2 ; 4r2 : (2) Assume that a 6= b. From (1) and (2) we get (b ; a)(2c ; a ; b) = (b ; a)(a + b ; 4r1), i.e. 2c ; a ; b = a + b ; 4r1. From here b ; 2r1 = c ; a and from (1) it follows that r = 0, which is impossible. Thus a = b. 3 b and b) Let rr1 = t and 6 BAC = . Then c = 2b cos = 2 Analogously

Problem 2. The sides AC and BC of the triangle ABC are diameters of two circles, each of which touches internally a circle k, which is concentric to the incircle of 4ABC .

c tan = b cos tan = b cos r=2 2 2


p

1 ; cos = 3 b: p 1 + cos 4 7

b)2 + 4r2 and substituting c and r we Since b > 2r1, from (1) it follows that t = b ; (c ; 2r p 2 get t = 3 ( 7 ; 2). Problem 3. n points (n > 4), no three of which are colinear are given in the plane. More than n triangles are constructed with vertices among these points. Prove that at least two

triangles have exactly one common vertex. Solution. Assume the contrary and let k be the smallest number for which the assertion is not true. This means that there are constructed at least (k + 1) triangles using k points. It follows from the pigeonhole principle that there exists a point A which is a vertex of at least 4 triangles. Let ABC be the rst triangle. At least one of the points B and C is a vertex of the second triangle, which we denote by ABD. If ACX is the third triangle then X = D. Thus the forth triangle must contain B or C , which is impossible. Therefore if A and B are vertices of two triangles then they are vertices of all the four triangles. Let A be a vertex of t triangles, t 4. These triangles are of the kind ABA1 , ABA2 , : : :, ABAt , where all the points A1 A2 : : : At are pairwise di erent. Obviously it is not possible to exist a triangle of the type BXY , where X and Y are points which are di erent from A1 A2 : : : At. Triangles BAi Aj and Ai Aj Am do not exist too. Hence the points A, B , A1 , A2 , : : :, At are vertices only of the triangles ABA1 , ABA2 , : : :, ABAt . In such a way we use t + 2 points and get t triangles. It is not possible that t + 2 = k, because all triangles are t < k. The number of remaining points is k0 = k ; t ; 2 and by them there are constructed at least k + 1 ; t > k0 triangles, such that no two of them have 9

exactly one common vertex. The triangles are more than the points k0, and thus k0 > 4. We have found a number k0 < k for which the assertion is not true. This contradicts the choice of k.

10

SPRING MATHEMATICAL COMPETITION


1996

Grade 8

1. graphs of the functions y = 1 ; jx ; 1j and y = j2x ; aj is less than 3 Solution. Firstly, we shall nd the common points of the given functions. For this purpose we solve the equation j2x ; aj = 1 ; j1 ; xj: (1) a a a Since 1 < a < 2, then 2 < 1, and we shall consider the cases: x 2 , 2 < x < 1 and x 1. We have: the equation (1) takes the form a ; 2x = x. Then x = a 1. When x a 2 3 , which satis es a a (1), because 3 < 2 . 2. When a 2 < x < 1 the equation (1) takes the form 2x ; a = x, i.e. x = a, which does not satisfy (1), because a > 1. 2 3. When x 1 the equation (1) takes the form 2x ; a = 2 ; x. Then x = a + 3 , which 2 > 1 when a > 1. satis es (1), because a + 3 Thus, the graphs of the two functions have two common Figure 1. points (Figure 1), the rst of which (denoted by A) has coora , while the second one (denoted by B) dinates xA = a , y = A 3 3 2 a+2 4;a has coordinates xB = a + 3 , yB = 2 ; 3 = 3 . Denote by C , D and E the points, with coordinates xC = 1, yC = 1 , y = 0 and xE = 2, yE = 0, respectively. xD = a 2 D Then, the gure encountered by the two graphs is the quadrilateral ACBD and its area S is obtained by subtracting the areas of the triangles ODA and BDE from the area of the triangle OEC . Therefore,

Problem 1. Prove that for all real a 2 (1 2) the area of the gure encountered by the

S = SOEC ; SODA ; SBDE 1 DE y = 1; 1 OD y ; A B 2 12 = 1; 1 a a ; 1 2; a 4;a


2 2 3 2 2 3 1

1 a2 ; 1 (4 ; a)2 = 1 ; 12 12 1 ;a2 + 4a ; 2 = 6 1 2 = 1 6 2 ; (a ; 2) < 3 : Problem 2. The altitude CD of the rectangle triangle Figure 2. ABC (6 ACB = 90 ) is a diameter of the circle k, which meets the sides AC and BC in E and F , respectively. The intersection point of the line BE and the circle k, which is di erent from E , is denoted by M . Let the intersection point of the lines AC and MF be K , and the intersection point of the lines EF and BK be P . a) Prove that the points B , F , M and P are concyclic b) If the points D, M and P are colinear, nd the angles A and B of the triangle ABC . Solution. a) Since 6 ECF = 90 , then EF is a diameter of the circle k and consequently 6 EMF = 90 = 6 BMK . Then BC and KM are altitudes in 4BEK (Figure 2), which means that the point F is the altitude center of this triangle. It follows from here that EP ? BK , i.e. 6 BPF = 90 . Thus, 6 BPF = 6 BMF = 90 and consequently the points B , F , M and P lie on a circle with diameter BF . b) We have: Figure 3. 6 BDM = 90 ; 6 MDC = 90 ; 6 MEC = 6 CBE = 6 FBM = 6 FPM: If the points D, M and P are colinear (Figure 3), then the equality between the angles 6 BDM and 6 FPM implies that the lines AB and EF are parallel, i.e. EF and CD are diameters of the circle k, which are perpendicular to each other. This means that CD is the bisector of 6 ACB . Consequently, AC = BC and 6 BAC = 6 ABC = 45 . Problem 3. In a state every town is connected with the nearest town by a straight way. The distances between the pairs of towns are pairwise di erent. Prove that a) no two ways have common points b) every town is connected by ways with at most 5 other towns c) there is no closed piecewise line, consisted of ways. Solution. a) Suppose that the ways AC and BD meet each other (Figure 4) and let C be the nearest town to A, while D be the nearest town to B . Then AC < AD and BD < BC , from where AC + BD < AD + BC . On the other hand, if O is the common point of AC and BD, then AO + OD > AD and BO + OC > BC , from where AC + BD > AD + BC . This is a contradiction. b) Let the town X be connected by ways with the towns A and B (Figure 5). Then AB is the longest side of 4XAB . Indeed, if we assume that for example AX is the longest side, then 2

A should not be the nearest town to X , and X should not be the nearest town to A as well. Consequently, the way AX should not exist.
Figure 4. Figure 5. Figure 6.

Therefore 6 AXB is the biggest angle in 4XAB , from where 6 AXB > 60 . Now, if we assume that the town X is connected with at least 6 towns, then the sum of the angles at X would be greater than 6 60 = 360 , which is impossible. Thus, every town is connected with at most 5 other towns. c) Assume that there exists a closed piecewise line A1A2 : : :An , consisted of ways (Figure 6). The distances A1 An and A1 A2 are di erent. Let A1An < A1A2 . Then A2 is not the nearest town to A1 and consequently (because the way A1 A2 exists) A1 is the nearest town to A2 . It follows from here that A1 A2 < A2 A3 . Proceeding in this way we obtain the chain: A1 An < A1A2 < A2 A3 < : : : < An A1 , which leads to contradiction.
Grade 9

Problem 1. Find the values of the real parameter b, for which the di erence between the maximal and the minimal values of the function f (x) = x2 ; 2bx +1 in the interval 0 1] is equal to 4.
Figure 7. Figure 8. Figure 9.

b 0

1 b

0 b

Solution. It is clear that the minimal value of the quadratic function f (x) is obtained when x = b. We shall consider the following three cases: 1. Let b < 0. In this case the function f (x) is increasing in the interval 0 1] (Figure 7) and the maximal value is f (1) = 2 ; 2b, while the smallest one is f (0) = 1. From the condition 3 , which is a solution of the problem. f (1) ; f (0) = 1 ; 2b = 4 we nd b = ; 2

2. Let b > 1. Now the function f (x) is decreasing in the interval 0 1] (Figure 8) and the maximal value is f (0) = 1, while the minimal one is f (1) = 2 ; 2b. From the condition 5 , which is a solution of the problem. f (0) ; f (1) = 2b ; 1 = 4 we nd b = 2 3. Let 0 b 1 (Figure 9). The minimal value of f (x) in the interval 0 1] is f (b) = 1 ; b2, while the the maximal one is f (0) or f (1). From f (0) ; f (b) = b2 = 4 we nd b = 2, which are not solutions (because in this case 0 b 1). From f (1) ; f (b) = (1 ; b)2 = 4 we nd b = 3 or b = ;1, which are not solutions too. 3 and b = 5 . Finally, the answers are: b = ; 2 2 Problem 2. The quadrilateral ABCD is inscribed in a Figure 10. circle with radius 1, a circle can be inscribed in it and AB = AD. Prove that: a) the area of the quadrilateral ABCD does not exceed 2 p b) the inradius of the quadrilateral ABCD does not exceed 2. 2 Solution. We have (Figure 10) AB + CD = AD + BC and from AB = AD we get: BC = CD. Let AB = AD = x, BC = CD = y. The triangles ACB and ACD are equal (by SSS), from where 6 B = 6 D. But ABCD is inscribed in a circle and consequently 6 B + 6 D = 180 . Therefore 6 B = 6 D = 90 . Then AC is diameter of the circumcircle of the quadrilateral and particularly AC = 2 and x2 + y2 = AC 2 = 4. Let S , p and r be the area of the quadrilateral, its semiperimeter and inradius, respectively. a) We have S = SABC + SACD = 2SACB = xy and q 2 y2 = AC 2 = 2 xy = x2y2 x + 2 2 Thus, S 2 (and S = 2 only if x = y and then the quadrilateral ABCD is a square). b) We shall make use of the formula S = pr, which is true for all polygons that can be inscribed in a circle. We have:
2 S 2 = fracS 2x2 + y2 + 2xy = S 2 : r2 = S = p2 (x + y)2 4 + 2S

Now

2 () r2 2

() S 2 ; S ; 2 0 () S 2

1 2

()

1 4 + 2S 2 () (S + 1)(S ; 2) 0

S2

(because p S > 0). The last inequality is true according to a) and consequently the inequality r 22 is true too. Problem 3. This problem is the same as problem 3, grade 8. 4

Grade 10

Problem 1. Find in the plane the locus of points with coordinates (x y), for which there exists exactly one real number z , satisfying the equality:
xz4 + yz3 ; 2 (x + jyj) z2 + yz + x = 0:
Let (x y ) be the coordinates of a point from the locus, we are looking for. This means that there exists exactly one real number z , for which
Solution.

xz4 + yz3 ; 2 (x + jyj) z2 + yz + x = 0:


If x = 0, the above equality takes the form

z yz2 ; 2jyjz + y = 0:
This equality is satis ed for at least two di erent values of z (for example z = 0 and z = 1, if y 0 and z = 0 and z = ;1, if y < 0), which shows that the condition of the problem is not veri ed. Consequently, the points from the y -axis do not belong to the locus. Let x 6= 0. Thus, if z satis es the given equality, then z 6= 0. It is easy to see that in this 1 satis es the same equality and consequently z = 1 . From here z 2 = 1, i.e. case the number z z z = 1. Case 1. Let z = 1. Then x + y ; 2 (x + jyj) + y + x = 0, from where; y ; jyj = 0. This shows that y 0. If y = 0 and then the given equality takes the form x z 2 ; 1 2 = 0, and consequently it is satis ed also by z = ;1. Thus, we can assume that y > 0. Then,

xz4 + yz3 ; 2(x + y)z2 + yz + x = 0


(z ; 1)2 xz 2 + (y + 2x)z + x = 0: We have one of the following possibilities: (i) The number z = 1 is the only to satisfy the equality xz 2 + (y + 2x)z + x = 0. This is true if x + y + 2x + x = 0 and D = (y + 2x)2 ; 4x2 = y (y + 4x) = 0, from where y = ;4x and y > 0. (ii) There is no real z , which satisfy the equality xz 2 + (y + 2x)z + x = 0. Then, D = (y + 2x)2 ; 4x2 = y (y + 4x) < 0, from where y < ;4x and y > 0. Therefore, every time when y ;4x and y > 0, the point (x y ) belongs to the locus. Case 2. Let z = ;1. Analogously to the previous case we nd that y < 0 and then the given equality takes the form (z + 1)2 xz 2 + (y ; 2x)z + x = 0: We have the following possibilities: (i) the number z = ;1 is double root of the equation from where

xz2 + (y ; 2x)z + x = 0:
(ii) the equation (*) has no real root. By computing we deduce that here y 4x and y < 0. 5

()

Finally, the locus (Figure 11) consists of the internal as well as of the boundary points of the angle which is de ned by the graphs of the linear functions y = 4x and y = ;4x when x < 0, without the points from the negative part of the x { axis. Problem 2. In the triangle ABC , ha and hb are the altitudes from A and B respectively, `c is the internal bisector of 6 ACB, while O, I and H are the circumcenter, the incenter and the altitude center, respectively. Prove that if `c + `c = 2, then OI = IH . ha hb Solution. Let BC = a, AC = b, 6 ACB = , CL = `c Figure 12. 1 (Figure 12). Since SABC = SALC + SBLC , then 2 `c a sin 2 + 1 ` b sin = 1 ab sin , from where ` = 2ab cos 2 . But c 2 c 2 2 a+b `c + `c = 2a cos 2 + ha = b sin and hb = a sin . Then h a hb (a + b) sin 2b cos 2 1 a + b 1 , which = = (a + b) sin sin 2 a + b a + b sin 2 shows that sin 2 = 1 2 , i.e. 2 = 30 , because 2 < 90 . Consequently, = 60 . Let CL meet the circumcircle of 4ABC in M . Then OM ? AB. But CH ? AB, and therefore 6 OMC = 6 MCH . On the other hand OM = OC = R (R is the circumradius). Consequently, 6 OMC = 6 OCM = 6 MCH . Let H2 be the foot of 2 (because 6 H2 HC = 90 ; 6 ACH the altitude from B . From 4HCH2 we have CH = CH sin = 6 CAB = ). But CH2 = a cos , i.e. CH = a cos = 2R cos . Since = 60 , then sin CH = R. We consider 4COI and 4CHI . Since the point I lies on CM , then 6 OCI = 6 HCI . Also, CO = R = CH and consequently 4COI = 4CHI . Thus, OI = IH . Problem 3. Let A1, A2, : : :, An (n 4) be n points in the plane, no 3 of which are colinear. a) Prove that there is at most one point As , such that all triangles As Ai Aj (i j = 1 2 : : : n) are acute. b) Let among A1, A2 , : : :, An be a point, which is a vertex of an acute triangles only. We consider the angles de ned by the given points. Denote by Nk the number of the acute angles 6 Ai Ak Aj (i j = 1 2 : : : n) for which the point Ak is their vertex. Find the minimal value of Nk . Solution. a) Assume that there is more than one point with the given property and let A and B be such two points, while X and Y be any two of the remaining points. There are two possibilities for the points A, B , X and Y : (i) A, B , X and Y de ne a convex quadrilateral. Because the sum of its internal angles is 360 , then at least one of these angles is not acute. Consequently, at least one of the triangles with vertex A or B is not acute. (ii) One of the points A, B , X and Y is inside the triangle de ned by the other three. Then, the sum of the three angles with a vertex inside is 360 . Consequently, in this case a triangle with vertex in A or B is not acute again. The contradictions show that it is not possible to exist more than one point which is a vertex 6

of acute triangles only. b) Let A1 be the point which is a vertex of acute triangles only. We consider the angles 6 Ak A1 As (Figure 13). Let A2 A1 An be the biggest one. Because all angles with vertex A1 are acute, the points A3 , A4 , : : :, An;1 are inside the acute angle 6 A2 A1An . We can assume that the points are enumerate in such a way that 6 A2A1 Ak < 6 A2A1 Ak+1 for k = 3 4 : : : n. Consider 6 Ai Ak Aj , where 2 i < k < j n. Figure 13. We assume that Ak is internal for 4A1 Ai Aj . Then

acute, which is a contradiction. Therefore, the points A1 , Ai , Ak , Aj de ne a convex quadrilateral. If we assume that 6 Ai Ak Aj 90 , then there exists an angle of this quadrilateral with vertex A1 , Ai or Aj which is 90 . This is impossible. Therefore, 6 Ai Ak Aj > 90 , and it is clear that 6 Ai Ak Aj < 90 when i j < k or i j > k. Particularly, it follows from here that no of the angles Ai Ak Aj is right. The number of all n ; 2) . If T is the number of the obtuse angles with angles with vertex Ak is equal to (n ; 1)( k 2 n ; 2) ; T . Thus, N is minimal when T is maximal. vertex Ak , then Nk = (n ; 1)( k k k 2 It is easy to see that T1 = T2 = Tn = 0. Let 3 k n ; 1. It is clear that the number of the points Ai for which 2 i < k is k ; 2, and the number of the points Aj for which k < j n is n ; k. Then Tk = (k ; 2)(n ; k) for k = 3 4 : : : n ; 1. 2)2 , and the equality is reached when k ; 2 = n ; k, i.e. when But (k ; 2)(n ; k) (n ; 4 2 . We have two possibilities: k = n+ 2 2 is integer and consequently the maximal value of T (i) The number n is even. Then n + k 2 2 2) . Therefore, the minimal value of N in this case is N n+2 = (n ; 1)(n ; 2 ; is T n+2 = (n ; k 2 2 2 2 2 (n ; 2) = n(n ; 2) . 2 4 2 are n + 1 and n + 3 . It is (ii) The number n is odd. Then, the nearest integers to n + 2 2 2 n +1 = n + 1 easy to see that now the maximal value of Tk is T n+1 ; 2 n ; = T n+3 = 2 2 2 2 (n ; 1)(n ; 3) . Consequently, the minimal value of N in this case is k 4 (n ; 1)(n ; 2) ; (n ; 1)(n ; 3) = (n ; 1)2 : N n+1 = N n+3 = 2 2 2 4 4
Grade 11

A1Ak Ai + 6 A1Ak Aj > 180 and at least one of the angles 6 A1 Ak Ai and 6 A1Ak Aj will not be
6

12x4 + ax3 + 12x2 ; 6x + 1 0 is satis ed for all real x. 7

Problem 1. Find the values of the real parameter a, for which the inequality x6 ; 6x5 +

1 . It is clear that x > 0 () t 2 and x < 0 () t ;2. But Denote t = x + x 1 = t2 ; 2 and x3 + 1 = t3 ; 3t. x2 + x 2 x3 We consider the function f (t) = t3 ; 3t ; 6 t2 ; 2 + 12t+ = t3 ; 6t2 + 9t + 12 + a. The problem is reduced to nd such a, that f (t) 0 for all t 2 and f (t) 0 for all t ;2 simultaneously. But f 0 (t) = 3t2 ; 12t + 9 = 3(t ; 1)(t ; 3), from where it is Figure 14. easy to obtain that (Figure 14) f (t) 0 for all t 2 i f (3) 0 and f (t) 0 for all t ;2 i f (;2) 0. Since f (3) = a + 12 and f (;2) = a ; 38, we nd for a: a ;12 and a 38. Finally, ;12 a 38. _ Problem 2. The point D lies on the arc BC of the circumcircle of 4ABC which does not contain the point A and D 6= B , D 6= C . On the rays BD! and CD! there are taken points E and F , such that BE = AC and CF = AB . Let M be the midpoint of the segment EF . a) Prove that 6 BMC is right. _ b) Find the locus of the points M , when D describes the arc BC . Figure 15. Figure 16.

When x = 0 the given inequality is satis ed for all a. Thus, it is enough to nd such a, that x3 + 13 ; 6 x2 + 12 + 12 x + 1 + a 0 for all x > 0 and x3 + 13 ; x x x x 1 1 2 6 x + + 12 x + +a 0 for all x < 0.
Solution.

x2

Solution. Denote by , , the angles corresponding to the vertexes A, B , C of 4ABC , and by a, _ b, c the lengths of the sides BC , CA, AB, respectively. Let 6 BCD = '. It follows from D 2BC that 0 < ' < , i.e. ' 2 (0 ). ; ! 1 ;! ;! ;! ;! ;! ; ; ! CF + BE + BC and CM = a) We have (Figure 15) ; BM = 2 BF + BE = 1 2 1 ;! ;! 1 ;! ;! ;! ; ; ! ; ; ! CF + CE = CF + BE ; BC . Then we nd for the scalar product BM CM : 2 2 ! ;! ;! 2 ;!2 ; ; ! ; ; ! 1 BM CM = 4 CF + BE ; BC

(We have used that jBE j = jAC j = b, jCF j = jAB j = c and 6 (;! BE ;! CF ) = 6 BDC = ; .) ; ; ! ; ; ! Consequently, BM ? CM , i.e. the angle 6 BMC is right. b) It is clear that the points M and A are in di erent semiplanes with respect to the line BC . According to a) 6 BMC is a right angle and consequently if the point M is from the locus, then M lies on a semicircle k with diameter BC , k and A are in di erent semiplanes with respect to the line BC . Let 6 BCM = . Then CM = a cos (because 6 BMC = 2 ) and
; ; ! ;!

;!2 ;!2 ;! ;! 2 CF + BE + 2 CF BE ; a = 1 4 2 2 2 = 1 4 c + b + 2bc cos( ; ) ; a 2 2 2 = 1 4 c + b ; 2bc cos ; a = 0:

CM CB = a(a cos ) cos = (a cos )2:


On the other hand by the sine theorem we nd: ; ; ! ;! 1 ;! CM CB = 2 CB + ;! BE + ;! CF ;! CB 2 = 1 2 a + ab cos ( ; ( ; ')) + ac cos ' b c 2 = 1 a 1 ; cos( ; ') + cos ' 2 a a sin cos ' ; sin cos( ; ') 1 = 2 a2 1 + sin 1 sin( ; ' ) ; sin( ; + ') = 2 a2 1 + 2 sin 2 = 1 a2 (1 + cos( + ')) = a cos + ' : 2 2 ' . But + ' < + < and therefore = + ' , i.e. Consequently, cos2 = cos2 + 2 2 2 2 2 + ' and 6 BCM 2 + . In addition we have 6 CBM = ; 6 BCM = 6 BCM = 2 2 2 2 + ; ' and 6 CBM 2 + . 2 2 2 Let K and N be points from the semicircle k, for which 6 BCK = 2 and 6 CBN = 2 . Then + . 6 CBK = angleBCN = 2 ; 2 = + and ; = 2 2 2 2 _ It follows from the above considerations that when the point D describes the arc BC , then CM ! describes the interior of 6 (CK ! CN !), while BM ! describes the interior of _ 6 (BK ! BN ! ). Consequently, the locus, we are looking for is the arc NK from the semicircle k (Figure 16). Problem 3. is the same as problem 3, grade 10. 9

Spring mathematics tournament|1997


Problem 8'1. Given the equation jx ; aj + 15 = 6jx + 2j, where a
is a real parameter. (a) Prove that for any value of a the equation has exactly two distinct roots x1 and x2. (b) Prove that jx1 ; x2j 6 and nd all values of a for which jx1 ; x2j = 6.

Solution: (a) When x ;2, the equation is equivalent to jx ; aj = ;6x ; 27, which has a solution only if ;6x ; 27 0, i. e., if 9 . Considering the cases of both x a and x a shows that x ;2 if x ;2, the given equation has a unique root x1, and if x > ;2,
it has a unique root x2: 8 8 a ; 27 9 > > < < 7 a < ;2 x = x1 = > a + 27 9 2 > : :; a ; 5 2 1 3;a a 1 53 2 a+ 1 7 a> 2

(b) It follows from (a) that 8 156 ; 12a 9 > > a < ; > 2 1 < 35 9 jx1 ; x2j = > 6 ;2 a 2 > > : 12a + 204 a > 1 35 2 It remains to be seen that 156 ; 12a > 6 when a < ; 9 and 35 2 12a + 204 > 6 when a > 1 . Therefore jx ; x j 6 and equal1 2 35 2 1 ]. ity obtains only when a 2 ; 9 2 2 of the convex quadrilateral ABCD and let 6 DAC = 6 DBC . The midpoints of AB and CD are respectively M and N and P and Q are points on AD and BC respectively such that OP ? AD and OQ ? BC . Prove that MN ? PQ.

Problem 8'2. Let O be the intersecting point of the diagonals

and 6 MEP = 6 MEO + 6 OEP = 6 MEO + 26 DAC = 6 MFO + 26 DBC = 6 MFQ (MFOE is a parallelogram). Therefore MP = MQ. The case when E and F are interior points for 6 PMQ is treated similarly (prove that there are no other possibilities). By analogy we conclude that NP = NQ. Hence M and N lie on the axis of symmetry of PQ and so MN ? PQ. 2

Solution: Denote by E and F the midpoints of AO and BO. Then 1 AO = PE , QF = 1 OB = ME 4PEM = 4MFQ since MF = 2 2

Problem 8'.3. Find all natural numbers n such that there exists
an integer number x for which 499(1997n + 1) = x2+x.

Solution: Let n be a solution of the problem. Then (2x + 1)2 = 1996 1997n + 1997. If n = 1 we get (2x + 1)2 = 19972 and so 2x + 1 = 1997. Therefore x = 998 and x = ;999 satisfy the

conditions of the problem. Let n 2. Now (2x + 1)2 is divisible by 1997, which is a prime number, and so (2x + 1)2 is divisible by 19972 . But this is impossible, since 1996 1997n +1997 is not divisible by 19972 when n 2. The only solution is n = 1.

exactly three distinct roots. the equations

Problem 8.1. Find all values of the real parameter m such that the equation (x2 ; 2mx ; 4(m2 + 1))(x2 ; 4x ; 2m(m2 + 1)) = 0 has

Solution: Suppose m satis es the conditions of the problem and


x2 ; 2mx ; 4(m2 + 1) = 0 x2 ; 4x ; 2m(m2 + 1) = 0
share the root x0. After subtracting we get (2m ; 4)x0 = (2m ; 4)(m2 +1) and so x0 = m2 +1 (note that if m = 2, the two equations coincide). Substituting x0 in any of the equations gives the equation (m2 + 1)(m2 ; 2m ; 3) = 0 with roots m = ;1 and m = 3. Direct veri cation shows that the condition is satis ed only for m = 3. Let now (1) and (2) share no roots. Since D1 = 4 + 5m2 > 0 (1) always has two distinct roots and therefore (2) should have equal 3

roots. Thus D2 = 4 + 2m(m2 + 1) = 0 and so m = ;1. But this case has already been considered. Thus we determine that m = 3.

Problem 8.2. The area of the equilateral triangle ABC is 7. Points


M and N are chosen respectively on AB and AC so that AN = BM . Denote by O the intersecting point of the straight lines BN and CM . The area of BOC is 2.
(a) Prove that MB : AB = 1 : 3 or MB : AB = 2 : 3. (b) Find 6 AOB .

Solution: (a) Denote MB AB = x. Therefore SABN = 7x = SBMC and so SBOM = 7x ; 2 and SAMON = SBOC = 2. Further SCON = x S = x(5 ; 7x) , 7 ; 2 ; 2 ; (7x ; 2) = 5 ; 7x, SANO = 1 ; x CNO 1;x 1 ; x 1 ; x SAMO = x SBOM = x (7x ; 2). It follows from SAMON = ; 7x) + 1 ; x (7x ; 2), and thus 9x2 ; SANO + SAMO that 2 = x(5 1;x x

1 and x = 2 . 9x + 2 = 0: The roots of the above equation are x1 = 3 2 3 6 6 (b) Since 4ABN = 4BMC, we get BOM = BCM + 6 CBO = 6 MBO + 6 CBO = 60 . Further 6 MAN + 6 MON = 180 and therefore the quadrilateral AMON is inscribed in a circle. Let MB = 1 , i. e., AM = 2BM = 2AN . Denote by Q the midAB 3 point of AM . Triangle AQN is isosceles and has an angle equal to 60 , so it is equilateral. Therefore Q is the circumcentre of AMON and 6 AOM = 6 ANM = 90 . Thus 6 AOB = 150 . Similarly, if 4

MB = 2 , i. e., 2AM = MB = AN , we get 6 AMN = 6 AON = 90 , AB 3 so 6 AOB = 90 .

Problem 8.3. Given n points, n

5, in the plane such that no three lie on a line. John and Peter play the following game: On his turn each of them draws a segment between any two points which are not connected. The winner is the one after whose move every point is an end of at least one segment. If John is to begin the game, nd the values of n for which he can always win no matter how Peter plays.

Solution: Call a point isolated if it is not an end of a segment. John wins exactly when there are 1 or 2 isolated points before his last move. Peter is forced to reach the above only if before his move there are exactly 3 isolated points and any of the remaining n ; 3 points are connected by a segment. Indeed, if there are at least 4 isolated points he could connect one of them with a non-isolated point. If the isolated points are 3 but not all of the remainig n ; 3 points are connected he could draw a missing segment. Since the n ; 4) , number of segments with ends in n ; 3 points is (n ; 3)( 2 ( n ; 3)( n ; 4) is an odd we determine that John wins only when 2 integer number. This is true when n is of the form 4k + 1 or 4k + 2.
3 where a is real paramProblem 9.1. Let f (x) = x2 ; 2ax ; a2 ; 4 eter. Find all values of a such that the inequality jf (x)j 1 holds for any x in the interval 0 1]. 5

of f (x) in the interval 0 1]. Then the condition of the problem is equivalent to M 1 and m ;1. There are three cases to consider. 3 and M = f (0) = Case 1: a 2 0 1]. Then m = f (a) = ;2a2 ; 4 3 or M = f (1) = ;a2 ; 2a + 1 . It follows from m ;1 and ;a2 ; 4 4 p p 2 2 1 1). In this M 1 that a 2 ; 4 4 ] and a 2 (;1 ; 3 ] ; 2 2 p 2 case the solution is a 2 0 4 ]. Case 2: a < 0. Now m = f (0) and M = f (1). From m ;1 1 ]: Therefore a 2 ; 1 0). and M 1 we get that a 2 ; 1 2 2 2 Case 3: a > 1. Now m = f (1) and M = f (0). It follows from 5 1 ] which is a contradiction with m ;1 and M 1 that a 2 ; 2 2 a > 1. p 1 Thus the solution is a 2 ; 2 42 ].

Solution: Let M and m be the maximum and minimum values

Problem 9.2. Let I and G be the incentre and the centre of 4ABC with sides AB = c, BC = a, CA = b. (a) Prove that (if a > b) the area of CIG equals (a ; b)r where r

6 is the inradius of ABC: (b) If a = c +1 and b = c ; 1, prove that the segment IG is parallel to AB and nd its length. 6

CL and CM be respectively the bisector and the median from C . 2 CM that S = 2 S . Thus S It follows from CG = 3 CIG CIM = 3 CIM r = LM (h ; r). We nd from SCLM ; SILM = LM2 hc ; LM 2 2 c AL = b that AL = bc and so LM = AM ; AL + BL = c and BL a a+b bc c ( a ; b ) S ; r = 2pr ; r = c AL = 2 ; a + b = 2(a + b) . Also, hc ; r = 2c c r(2p ; c) = r(a + b) . Therefore S = 2 S = 2 c(a ; b) CIG c c 3 CIM 3 4(a + b) r(a + b) = (a ; b)r . c 6 (b) The distances from I and G to AB are respectively r and hc . Hence r = S = chc = chc = hc and so IGkAB . Therefore 3 p 2p 3c 3 the altitude from C of triangle CIG equals 2 3 hc = 2r. Thus SCIG = b)r = r and so IG = 1 . IG r. On the other hand SCIG = (a ; 6 3 3

Solution: (a) We shall use the usual notation for a triangle. Let

Problem 9.3. Let n n be an even number and A be a subset of f1 2 : : : ng. Consider the sums of the form "1x1 + "2x2 + "3x3,

where x1 x2 x3 are integer numbers in A (not necessarily distinct), "1 "2 "3 (at least one of which is not 0) belong to f;1 0 1g and none of the elements of A appears with coe cients 1 and ;1 in any of the sums. Call A a `free' set if n divides none of the above sums. (a) Construct a `free' set having n 4 ] elements ( x] is the least integer number less than or equal to x). 7

(b) Prove that no set of n 4 ] + 1 elements is `free'.

n ] elements (prove it!). 4 (b) Let n = 4m and suppose that A f1 2 : : : ng is a `free' set having n 4 ] + 1 = m + 1 elements. Without loss of generality assume that A f1 2 : : : 2mg (since we can replace x 2 A by n ; x). We shall show that there exist two elements of A whose sum is equal to another element of A. Indeed, let a1 < a2 < < am+1 be the elements of A and consider the set B = fa1 + ai : i = 1 2 : : : m + 1g. There are 2m + 2 integer numbers a1 a2 : : : am+1 2a1 a1 + a2 : : : a1+am+1 from A B and they lie in the interval a1 a1+am+1], which contains exactly am+1 + 1 2m + 1 integer numbers. This gives ai = a1 + aj for some i j . But then a1 + aj ; ai = 0, which is impossible. The case n = 4m + 2 is settled in a similar fashion. Notice that 2m + 1 cannot be an element of a `free' set.

Solution: (a) The set A = f1 3 : : : 2 n 4 ] ; 1g is `free' and has

Problem 10.1. Find the least natural 3 number a such that the x + )+2 = 0 2 equation cos (a ; x) ; 2 cos (a ; x)+cos 2ax cos( 2a 3
has a root.

Solution: The roots of the our equations are the common roots of x + ) + 1 = 0. The roots of cos (x ; a) = 1 and cos 32ax cos( 2a 3
8

the rst one are x = a + 2n n = 0 1 2 : : : and the roots of the second one are x = 2a(k ; 1 ) k = 0 1 : : : : Therefore a = 6n 3 6k ; 5

for some integer numbers n and k. It is easy to see now that the least natural number with the required property is a = 6. and is such that DF = CF . Let E be the intersecting point of AC and BD and O1 and O2 are circumcentres of ADF and FBC respectively. Prove that the straight lines FE and O1O2 are orthogonal.

Problem 10.2. Point F lies on the base AB of a trapezoid ABCD

the intersecting points of the two circles be points P and Q. It is well known that PQ ? O1O2. On the other hand, if L is an arbitrary point and two lines through L intersect k1 and k2 in points A, B and C , D respectively. Then L 2 PQ if and only if LA LB = LC LD. Let k1 and k2 be the circumscribed circles of 4AFD and 4FBC and let G be the intersecting point of FE with CD. Denote by C1 and D1 those points on DC for which AD1jjCF and BC1jjDF , i. e., such that the quadrilaterals AFCD1 and BFDC1 are parallelograms. Using that FD = FC we get 6 CFB = 6 FCD = 6 FDC = 180 ; 6 BC1C . This means that F , B , C and C1 lie on a circle and so the line DC intersects k2 in C and C1. By analogy 6 AFD = 6 FDC = 6 FCD = 6 AD1D and so line DC meets k1 in points D and D1. In accordance with the initial notes FE is perpendicular to OO1 if and only if GC GC1 = GD GD1 . It follows from 4GCE 4FAE , 4GDE 4FBE and 4DCE 4BAE CE = DC and GD = DE = DC . Thus GC = that GC = AF EA AB BF EB AB DC AF and GD = BF DC . On the other hand GC = jDC ; 1 1 AB AB DC j = BF jAB ;DC j, j = BF j 1 ; DGj = jBF ;DGj = BF j1; DG BF AB AB 9

Solution: Let k1 and k2 be circles with centres O1 and O2 and let

DC j = GD1 = jCD1 ; CGj = jAF ; CGj = AF j1 ; CG j = AF j 1 ; AF AB AF jAB ;DC j. Therefore GC GC = DC AF BF jAB ;DC j = 1 AB AB 2 GD GD1 .

Problem 10.3. Find all natural numbers n for which a convex n-

gon can be partitioned into triangles through its diagonals in such a way that there is an even number of diagonals from each vertex. (If there is a vertex with no digonals through it, assume that there is an even number (zero) of diagonals from this vertex). titioned into triangles through d non-intersecting diagonals then n = d + 3. Let n be a natural number and A1A2 An is a convex n-gon which can be partitioned into triangles through d diagonals in a way that there is an even number of diagonals through each vertex. Since n = 3 is a solution we may assume that n 4. It is clear that at least one side of each triangle is a diagonal of the n-gon. We say that a triangle is of type tk , (k = 1 2 3) if exactly k of its sides are diagonals. Denote by xk the number of triangles of type tk . It is easy to see that 2x1 + x2 = n = d +3 abd x1 +2x2 +3x3 = 2d. It follows now that x1 = x3 + 2, so x1 > 0. Therefore there exists a triangle two of whose sides are sides of the n-gon. Let that be Aj;1Aj Aj+1. Diagonal Aj;1Aj+1 is a side of another triangle|e. g., Aj;1Aj+1As. Assume that Aj;1As or Aj+1As is a side of the n-gon. If it is Aj;1As then s = j ; 2. It follows now that there are no diagonals from Aj;1 distinct from Aj;1Aj+1 because such a diagonal intersects Aj+1As. This contradicts the premise that there is an even 10

Solution: It is easy to see by induction that if an n-gon is par-

number of diagonals from each vertex. Therefore both Aj;1As and Aj+1As are diagonals so Aj;1Aj+1As is of type t3. Hence there is a triangle of type t3 adjacent to each triangle of type t1. If distinct triangles of type t1 are adjacent to distinct triangles of type t3 then x1 x3 = x1 ; 2 < x1, a contradiction. Therefore there are at least two triangles of type t1 adjacent to one and the same triangle of type t3. Without loss of generality assume these are the triangles A1AnAn;1 and An;1An;2An;3 . Consider the polygon A1A2 An;3. Obviously the diagonals partition this polygon into triangles and there is an even number of diagonals through each vertex. Conversely, if the polygon A1A2 An;3 can be partitioned in the required way, then adding the vertices An;2, An;1 , An and diagonals An;3An;1 and A1An;1 shows that the same is true for the polygon A1A2 An. Therefore a natural number n 6 is a solution if and only if n ; 3 is a solution. It is easy to see that n = 3 is a solution, whereas n = 4 and n = 5 are not. Thus all natural numbers satisfying the conditions of the problem are n = 3k, k = 1 2 : : :.

Problem 11.1. For any real number b denote by f (b) the maximal value of j sin x + 3 + 2 sin x + bj. Find the minimal value of f (b).
2 + b. Since Solution: Substitute t = sin x and g(t) = t + 3 + t g(t) is an increasing function in the interval ;1 1], it follows that 3 j). Now from the graph f (b) = max(jg(;1)j jg(1)j) = max(jbj jb + 2 3. ) = of the function f (b) we conclude that min f (b) = f (; 3 4 4 11

ABC = 6 BCD. Let H and O be respectively the orthocentre and the circumcentre of 4ABC . Prove that H , O and D lie on a line.
6

Problem 11.2. A convex quadrilateral ABCD is such that 6 DAB = Solution: Let 6 CAB = , 6 ABC = , 6 BCA = . Note that

< and < . There are three cases to consider: < 90 , = 90 and > 90 . Suppose rst that < 90 . Then O and H are interior points for 4ABC and 6 ACO = 6 CAO = 6 HCB = 6 HAB = 90 ; . Therefore O is an interior point for 4HAC and 6 HAO = ; = 6 ACD, 6 HCO = ; = 6 CAD, 6 HAD = 6 HCD = 2 ; 90 . It follows from the Sine Theorem for 4AHD, sin 6 AHD = AD , sin 6 HCD = HD , 4CHD and 4ACD that sin 6 HAD HD sin 6 CHD CD sin 6 CAD = CD . By multiplying the above equalities we get sin 6 ACD AD

sin 6 AHD sin 6 HCO sin 6 CAO = sin 6 CHD sin 6 HAO sin 6 ACO: It follows now from Ceva's Theorem that AO, CO and HD intersect in a point and so H , O and D lie on a line. In the case of = 90 we obtain that H B , O is the midpoint of AC and AHCD is a rectangle. Therefore H , O and D lie on a line. Finally, let > 90 . In this case B and O are interior points for 4AHC and 4ADC respectively. Similarly to the case < 90 we get that the points H , O and D lie on a line. 3 denote by m(n) the maximum number of points which can be placed inside or on the outline of a regular n-gon with side 1 in a way that the distance between any two of them is greater than 1. Find all n for which m(n) = n ; 1. 12

Problem 11.3. For any natural number n

be one of the above numbers and let B1 B2 Bn be points satisfying the conditions of the problem for the regular n-gon A1A2 An of side 1 and centre O. It is obvious that OBi 1 and therefore no three points O, Bi, Bj , 1 i 6= j n lie on a line. Furthermore at least one of the angles OBi Bj is less than 2n 90 and it follows from the Cosine Theorem that BiBj2 OBi2 + OBj2 ; 2OBi OBj cos 2n = Bi0Bj0 2 where Bi0 and Bj0 are points on the segments OA1 and OA2 such that OBi0 = OBi , Bj0 = OBj . When n = 4 5 6 the greatest side in 4OA1A2 is A1A2 = 1 and therefore BiBj Bi0Bj0 1, which is a contradiction. Thus m(n) n ; 1 if n = 4 5 6. It is easily seen that for these n there exist n ; 1 points on the outline of a regular n-gon A1A2 An with the required property. Therefore m(n) = n ; 1 if n = 4 5 6. We shall prove now that if n 7 then m(n) n. Let Bi 2 AiAi+1 be points such that AiBi = 2i;1 ", 1 i n ; 1 where 0 < " < n1;1 is arbi2 trary chosen. From n 7 we obtain cos 6 A1A2A3 = ; cos 2 < ; 1 : n 2 It follows now that B1B22 > (1 ; ")2 +4"2 +2"(1 ; ")2 = 1+3"2 > 1: Similarly BiBi+1 > 1 when 1 i n ; 2. Further, it is clear that B1Bn;1 > A1An = 1. Since OA1 = OA2 = = OAn > 1, AiAj > 1 when ji ; j j 2 and Bi ! Ai when " ! 0, it follows that we can make " so small that OBi > 1 when 1 i n ; 1 and BiBj > 1 when ji ; j j 2. Then the points B1 B2 Bn;1 O satisfy the conditions of the problem and so m(n) n when n 7. Since it is obvious that m(3) = 1, we come to the conclusion that m(n) = n ; 1 only when n = 4 5 6. 13

Solution: We prove rst that m(n) = n ; 1 for n = 4 5 6. Let n

Spring mathematics tournament { 1998


equivalent.

Problem 8'.1. Find all values of the real parameter a such that a ; 8 < ; 1 are the inequalities jx + 1j + j2 ; xj < a and 6x 5 ; 5a + 5 2 Solution: We begin with the rst inequality. jx + 1j + j2 ; xj < a. a , i. e., If x < ;1, it is equivalent to ;x ; 1 ; x + 2 < a or x > 1 ; 2 1 ; a < x < ;1, which has a solution if 1 ; a < ;1, i. e., if a > 3. 2 2 If x 2 ;1 2], then jx + 1j + j2 ; xj = x + 1 + 2 ; x = 3 and the

above equation has a solution only when a > 3. Finally, if x > 2, 1 , i. e., 2 < x < a + 1 , which then x + 1 + x ; 2 < a or x < a + 2 2 has a solution when a > 3. Therefore when a 3, the inequality has no solution, and when a > 3, the solutions form the interval 1;a 1+a . 2 2 1

+ 5a ; 11 < Let us rewrite the second inequality in the form 66x x ; 5a + 5 11 ; 5 a 5 a ; 5 or the 0. Its solutions form either the interval 6 6 5 a ; 5 11 ; 5 a interval 6 6 , depending on which of the two numbers 5a ; 5 and 11 ; 5a is greater. 6 6 Therefore the two inequalities are equivalent if 1 + a = 5a ; 5 1 ; a = 11 ; 5a 2 6 2 6 or 1 + a = 11 ; 5a : 1 ; a = 5a ; 5 2 6 2 6 The rst pair is satis ed by a = 4, the second one by a = 1. But if a = 1, the rst inequality has no solution, whereas the second one does. Thus the only solution is a = 4.

line perpendicular to the bisector of HMN intersects the line AC in point P such that HP = 1 (AB + BC ) and HMN = 45 deg. 2 a.) Prove that 4ABC is isosceles. b.) Find the area of 4ABC if HM = 1.
6 6

Problem 8'.2. Let M and N be the midpoints of the sides BC and AC and BH (H 2 BC ) the altitude in 4ABC . The straight

Solution: a) Since HM is a median to the hypothenuse BC of the right triangle 4BHC and MN is a middle segment in 4ABC ,
1 BC (if H it follows that HM = 2 2

C , again HM = 1 2 BC ) and

90 deg ; . The statement 2 of the problem implies that MP is the bisector of NMK (external to 4HMN ), i. e., NMP = KMP . Therefore 4PNM = 4PKM and PNM = PKM = 90 deg ; 2 . On the other hand, PNM = 45 deg + , i. e., 90 deg ; 2 = 45 deg + , whence = 30 deg and HNM = 105 deg. Also MHB = 60 deg and since HM = MB , it follows that HMB = 60 deg. Thus BMN = 105 deg = ANM , i. e., ABMN is a isosceles trapezoid AN = BM . Therefore AC = BC .
6 6 6 6 6 6 6 6 6 6 6

1 AB . Therefore HP = HM + MN . Denote HMN = . MN = 2 There are two cases to consider for the points H N and P : 1. N lies between H and P 2. H lies between N and P . 1. Let us nd a point K on the extension of HM such that MK = MN . Then HP = HK and HKP =
6 6

and P and let K be a point such that MK = MN and HP = HK . It follows from the isoscelesness of 4KHP that HPK = HKP = 2 . Since MP is the bisector of NMK in the isosceles 4NMK , it follows that MP is the axis of symmetry of NK . Thus NP = KP and 4NMP = 4KMP . Therefore PNM = 2 and since 4NMP , it follows that + 2 + 45 deg = 180 deg. and hence = 90 deg. Therefore MH ? AC and since BH ? AC , we get that H coincides with C and BCA = 90 deg, ABC = BAC = 45 deg and so AC = BC . b) 1. When = 30 deg : ACB = 30 deg, HM = HB = 1, 1 AC BH = 1. AC = BC = 2. Then SABC = 2 2. When = 90 deg : AC ? BC , AC = BC = 2BH = 2. Then SABC = 1 2 AC BC = 2.
6 6 6 6 6 6 6

2. Let H lie between N

Problem 8'.3. (Problem for the UBM award)


4

Is it possible to nd 100 straight lines in the plane such that there

are exactly 1998 intersecting points?

Solution: Consider 99 lines such that 73 of them are parallel and

the remaining 26 lines pass through a single point and intersect all 73 parallel lines. Then the total number of intersecting points is 73 26 + 1 = 1899. Choose the last line in such a way that it intersects all lines and does not pass through any of the points. Now there are 1899 + 99 = 1998 intersecting points.

the coordinate axes Ox and Oy in A and B correspondingly.

Problem 8.1. The graph of a linear function is parallel to the 5 x + 95 , passing through M (;1 ;25), and intersects graph of y = 4 4
(a) Find the coordinates of A and B . (b) Consider the unity grid in the plane. Find the number of squares containing points of AB (in their interiors).

5 x + 95 when the linear function is of the form y = graph of y = 4 4 5 x+b. From the condition that M belongs to its graph we determine 4 95 . The coordiantes of A and B are ;25 = ; 5 + b , whence b = ; 4 4 respectively the solutions of the systems 5 x ; 95 5 x ; 95 y = y=4 and 4 4 4 y=0 x=0 5

Solution: (a) The graph of a linear function is parallel to the

whence we get A(19 0) and B 0 ; 95 4 . (b) The coordinates of the points on AB satisfy the conditions 5 x ; 95 y = (1) 4 4 : 0 x 19 Find the number of points with integer coordinates lying on the segment AB , i. e., the number of integer solutions of (1). It follows 5 x ; 95 that y = x ; 23 + x ; 3 and if x and y are from y = 4 4 4 x ; 3 integer, then 4 = t is integer. Conversely, if t is integer, then x = 4t + 3 and y = 5t ; 20 are integer. Since 0 x 19, we nd 0 4t + 3 19, i. e., ; 3 4 t 4. This observation implies that the points with integer coordinates lying on AB satisfy the conditions x = 4t + 3, y = 5t ; 20 for t = 0 1 2 3 4. Therefore there are 5 such points. Further: the segment AB exits a square and enters another one only when it intersects a line of the grid. The number of such intersections is 23 + 18 = 41. But when AB passes through a knot (i. e., through a point with integer coordinates), the passage from a square to another one involves crossing two lines. This happens 4 times (not counting A). Therefore the required number is 41 ; 4 + 1 = 38.

Problem 8.2. Let l1 and l2 be the loci of the centroid G and the
(a) Find l1 and l2. 6

incentre I of the right triangle ABC whose hypotenuse AB is a given segment of length c.

(b) Find the area of 4ABC when the length of GI is minimal. semiplane cwith respect to AB . The centroid G lies on a circle k1 of radius 6 and centred at the midpoint O of segment AB . The incentre I lies on one of the two arcs which are the locus of the points X such that AXB = 135 deg. One of the arcs is part of a circle k2 with centre Q and radius R and the other one is part of a circle k2 with centre Q and the same radius and Q and Q lie in di erent semiplanes with respect to AB . Conversely: Let G be a point on k1 distinct from the intersecting points M and N of k1 and AB . Then the vertex C of 4ABC is c . Since AO = BO = CO it follows uniquely determined by CO = 2 that 4ABC is a right triangle. Choose that of the points Q and Q which does not lie in the same semiplane as C does with respect to AB . Without loss of generality this is Q . Denote the intersecting point of the bisector of BAC and k2 by I . We shall prove that I is the incentre 4ABC . Since ABI = 45 deg ; BAI , we get 2 ABI = 90 deg ;2 BAI = ABC . Therefore BI is the bisector of ABC . Let I be a point of k2 such that AIB = 135 deg. Then the vertex C is uniquely determined by: AI is the bisector of BAC , BI is the bisector of ABC and C lies in one and the same semiplane deg, it follows with I with respect to AB . Since BAI + ABI = 45 c that 4ABC is a right triangle. Therefore CO = 2 and hence the intersecting point G of CO and k1 is the centroid of 4ABC . These observations imply that l1 is the circle k1 without M and
6 0 0 00 00 0 00 0 00 0 6 0 6 6 6 6 6 6 0 6 6 6 6 6

Solution: (a) The vertex C of 4ABC could be placed in either

N , and l2 consists of two arcs k2 and k2 such that if X 2 k2 or X 2 k2 , then AXB = 135 deg. (b) We shall show that R > 2 3 c and it will follow (how?) that k1 lies in the interior of both k2 and k2 . Let P be the intersecting point of OQ and k1 which lies between O and Q . c = c+c = Since PQ = 3 6 6 OP + ON and 4PNO, it follows that OP + ON > PN and so PQ > PN . Hence PNQ > PQ N . Therefore PNQ + 45 deg > PQ N + 45 deg, so ANQ > AQ N . 2 c. Let G 2 l and It follows for 4AQ N that AQ > AN , i. e.,R > 3 1 I 2 l2 be such that Q , O, G and I lie in a line in this order. We shall show that GI is the least possible segment.
0 00 0 00 6 0 00 0 0 0 0 6 6 0 0 6 0 6 0 6 0 6 0 0 0 0

Case 1: G1 2 l1, I1 2 l2 and O lie on a line. Then G1I1 > GI . Indeed, it follows from 4Q I1O that OQ + OI1 > Q I1, i. e., OQ + OG1 + G1I1 > R = OQ + OG + GI and so G1 I1 > GI .
0 0 0 0 0

Case 2: G2 2 l1 , I2 2 l2 and O are not on a line. Let OI2 intersect k1 in a point G1. Then G2I2 > G1 I2. Indeed, it follows from 4OG2I2 that OG2 + G2I2 > OI2. But OI2 = OG1 + G1I2 = OG2 + G1I2 and therefore G2I2 > G1I2. But from case 1 we get that G1I2 > GI and so G2I2 > GI . Thus 4ABC having the required property is such that Q , O, G and I lie on a straight line. Similarly, Q determines a triangle equal to the rst one. 2Since OQ ? AB , 4ABC is an isosceles triangle and so SABC = c4 .
0 00 0

Given n points on a circle such that no three chords with ends in the given points intersect in a point. Prove that there exists n 2 ; 3n + 4 n such that there are chords with ends in the given points 2 partitioning the interior of the circle into 1998 regions.

Problem 8.3. (Problem for the UBM award)

Solution: First we shall prove the following Lemma: The number


of regions into which the interior of a circle is divided by drawing all ! n chords with ends in n given points, provided no three chords 2

n + 1: intersect in a point, is n + 4 2
9

Use induction by n. It is easy to see that the above formula holds for n = 2. Suppose it is true for some n. To obtain the result we count how many new regions are added when a new point appears on the circle. It is easily seen that if a chord intersects t other chords, then it `adds' t + 1 new regions. Therefore the new regions are:
n 1 X
;

k=0

(k(n ; k ; 1) + 1):

1)n and 12 +22 + +(n ; 1)2 = Given that 1+2+ + n ; 1 = (n ; 2 (n ; 1)n(2n ; 1) , we easily obtain that the above sum equals 6 ! ! ! ! ! n+1 + n+1 +1; n + n +1 4 2 4 2 which completes the proof. 2 n+4 = We show now that if n = 17 it is possible to draw n ; 3 2 121 chords such that there are 1998 regions. Let us draw all chords with ends in 16 of the given points (there are 120 such chords). ! 16 It follows then that the interior of the circle is divided into 4 + ! 16 + 1 = 1941 regions. Draw a chord connecting the 17th point 2 with one of the rst 16 in a way that there are 8 and 7 points on the two sides of the drawn chord. This chord intersects 8 7 = 56 chords and therefore there are 57 new regions. Therefore the total number of regions is 1941 + 57 = 1998. 10

Problem 9.1. Find all parameters a such that the inequality jax2; 3x ; 4j 5 ; 3x holds for any x 2 ;1 1]. Solution: Observe that the inequality is equivalent to the system ax2 ; 9 0 ax2 ; 6x + 1 0:
It follows from the second inequality that a > 0, because if a 0 then ax2 ; 6x + 1 0 is not true for x = 1. Further, the second 9 . Since this inequality is true for x 2 ;1 1], inequality gives x2 a 9 ) a 9. Let D = 9 ; a be the discriminant of we get 1 a ax2 ; 6x + 1. There are two cases to consider: 1) D > 0. The solution of the second inequality is x 2 (;1 x1] x2 +1), where x1 < x2 are the roots of ax2;6x+1 = 0. Therefore ;1 1] (;1 x1] or ;1 1] x2 +1). 1.1) ;1 1] (;1 x1]. It follows from the above that a ;7 a(;1)2 ; 6(;1) + 1 0 a ;7 2 a 5 a 1 ;6 1+1 0 () a 5 () 3 x1 + x2 > 1 a<3 2 a>1 which is impossible. 1.2) ;1 1] x2 +1). Therefore a ;7 a(;1)2 ; 6(;1) + 1 0 5 a:12 ; 6:1 + 1 0 () a x1 + x2 < ;1 3 < ;1 2 a 11

which is also impossible. 2) D 0 ) the inequality ax2 ; 6x + 1 0 holds for any real value of x. It then follows that 9 ; a 0 () a 9. Therefore the solution is a = 9.

Problem 9.2. The quadrangle ABCD is inscribed in a circle. The

tangents to the circle passing through A and C intersect at the point P . If PA2 = PD PB and P does not lie on DB , prove that the intersecting point of AC and BD is the midpoint of AC .

Solution:

AP : (1) BP Also, 4APB 4DCP because APB = DPC = APC ; AP = BP and so AB = BP : (2) APD and DP CP DC CP
6 6 6

Let E be the second intersecting point of PB and the circle. Then PA2 = PE PB ) PD = PE . Hence APD = EPC . (If O is the centre of the circle, the above follows from the similarity of 4ODP and 4OEP ). Therefore 4ADP 4PCB because APD = BPC and AP = DP , so AD = BP CP BC
6 6 6 6

12

AB AD = AP BP = 1. On the From (1) and (2) we get BC DC BP CP other hand, 1 AB AD sin DAB = AB AD = 1 SABD = 2 1 SCBD 2 BC DC sin DCB BC DC ( DAB = 180 deg ; DCB ): Therefore SABD = SCBD , i. e., the diagonal BD halves AC .
6 6 6 6

Problem 9.3. (Problem for the UBM award)


See problem 8.3. the inequality x4 + ax3 + (a + 3)x2 + ax + 1 > 0 holds for all real values of x.

Problem 10.1. Find all values of the real parameter a such that

Solution: If x = 0, the inequality holds for any a2 . Suppose x 6= 0.

Dividing both sides of the inequality by x2 gives x + ax + a + 3 + a + 1 > 0 or x + 1 2 + a x + 1 + a + 1 > 0. Substitute x x2 1 x x t = x + x . If x > 0, it is true that t 2, and if x < 0, it is true that t ;2. Therefore we want to nd all a such that the inequality t2 + at + a + 1 > 0 holds for any t 2 (;1 ;2] 2 +1). Denote f (t) = t2 + at + a + 1. The discriminant of f (t) is D = a2 ; 4a ; 4. p p Let D < 0, i. e., a 2 (2 ; 2 2 2 + 2 2). Then the inequality f (t) > 0 holds for any real t, in particular for t 2 (;1 ;2] 2 +1). p p Let D 0, i. e., a 2 (;1 2 ; 2] 2 + 2 +1). Then the inequality f (t) > 0 holds for any t 2 (;1 ;2] 2 +1) if and 13

only if

f (2) > 0 f (;2) > 0 p p The solutions of this system when a 2 (;1 2 ; 2] 2 + 2 +1)
are 5 2+2 2 . a 2 ;5 2 ; 2 2 . So a 2 ; 3 3
i

;2 < ; a 2 <2

Problem 10.2. A quadrangle with perpendicular diagonals AC


and BD is inscribed in a circle with centre O and radius 1. a.) Calculate the sum of the squares of the sides of the quadrangle. b.) Find the area of ABCD if a circle with centre I is inscribed in 1. it and OI = p 3

4 cos2 + 4 sin2 + 4 cos2 = 8. b.) The statement of the problem implies that AB + CD = AD + BC or 2 cos +2 sin = 2 cos +2 sin , i. e. sin( +45 deg) = sin( +45 deg). Therefore = or = 90 deg ; . We shall consider only the case = (the case = 90 deg ; is settled in a similar fashion). Now AD = AB , CD = BC and ABC = ADC = 90 deg. The points O and I lie on AC and when 45 deg, I lies on
6 6

AB = 2 cos . Therefore AB 2 + BC 2 + CD2 + DA2 = 4 sin2 +

Solution: a.) Denote BAC = , CAD = . It follows from the Sine Theorem for 4ABC and 4ABD that BC = 2 sin , AD = 2 sin ABD = 2 sin(90 deg ; ) = 2 cos . Analogously CD = 2 sin ,
6 6 6

14

the segment AO. In 4ACD (DI is a bisector) determine AI = 2 cos sin ; cos = p 1. . Consequently OI = AO ; AI = sin + cos sin + cos 3 1 ; sin 2 1 Raising both sides to the 2nd power gives 1 + sin 2 = 3 . Hence sin 2 = 1 2 . It then follows that the area of ABCD is equal to S = 2SABC = AB BC = 4 cos sin = 2 sin 2 = 1.

Problem 10.3. (Problem for the Atanas Radev award)

Find all natural numbers n such that: If a and b are natural numbers and n divides a2b + 1, then n divides a2 + b.

2. Let a be a natural number coprime to n. It follows from Bezout's Theorem that there exists a natural number b such that a2b + 1 is divisible by n. Further, n divides a2 + b and since a4 ; 1 = a2(a2 + b) ; (a2b + 1), it follows that n divides a4 ; 1. Let n = 2 k, where k is an odd number and 0. Suppose k 3. Then n and k ; 2 are coprime and therefore 2 k divides (k ; 2)4 ; 1, so k divides 15. Hence n is of the form n = 2 3 5 , where 0 0 1. Consequently n divides 114 ; 1 and we conclude that 4. It is easy to see now that n divides 24 3 5 = 240. Conversely, let n be a divisor of 240. Then n satis es the condition of the problem. Indeed, if 3 divides a2b + 1, then 3 divides both a2 ; 1 and a2 + 1 and therefore 3 divides a2 + b. Similarly, if 5 divides a2b + 1, then 5 divides a2 + b. If we can show that the 15

Solution: Obviously the condition holds for n = 1 and suppose


n

same property holds for 2 4 8 and 16, we will be done. Assume 2k divides a2b + 1, where 1 k 3. Then a is an odd integer number and therefore a2 ; 1 is divisible by 8. Consequently b + 1 is divisible by 2k and thus a2 + b is divisible by 2k as well. Assume 16 divides a2b +1. Then a is an odd integer number and it is easy to verify that a2 is congruent to 1 or 9 modulo 16. Further, b should be congruent to 15 or 7 modulo 16 correspondingly and again a2 + b is divisible by 16. The required numbers are all divisors of 240.

Problem 11.1.
a.) Let p be a positive real parameter. Find the least values of p and g(x) = x + p in the interval the functions f (x) = x + x x2 (0 +1). b.) p Let a1 ap 2 a3 be positive real numbers. Prove that 3(a1 + a1a2 + 3 a1a2a3) 4(a1 + a2 + a3): (x ; pp)(x + pp) p , we get Solution: a.) Since f (x) = 1 ; x2 = 2 x p that the functionp f (x) decreases in the interval (0 p) and increases in the interval ( p p +1). p Therefore the minimal value of f (x) in (0 +1) equals f ( p) = 2 p. Analogously it follows from g (p x) = q 3 2 p 3 1; x3 that the least value of g(x) in (0 +1) equals g( 3 2p) = 22p . 3 a a a our b.) After substituting x = a1 y = pa1a2 z = p 1 2 3 2 3 1 4 y 4 z inequality becomes 0 3 x + 3 x ; y + 3 y2 ; z. It follows from a.)
0 0

16

2 3 2 4 1 4 y 1 4 z 1 4 y that 3 y 3 x + 3 x and 3 y + 3 y2 z. Therefore 3 x + 3 x ; z3 ; z 1 y + 4 z3 ; z 0, which completes the proof. Note y+4 3 y2 3 3 y2 that equality occurs when x = 2y = 4z, i. e., a1 = 4a2 = 16a3. Note further that b.) could also be solved by the following inequalities: p 1p 3 3 a1 + pa1a2 + p a1a2a3 = a1 + 1 a 14a2 + 2 4 a14a216a3 1 (a + 4a ) + 1 (a + 4a + 16a ) = 4 (a + a + a ): a1 + 4 1 2 2 3 12 1 3 1 2 3

Problem 11.2. Let I and r are the incentre and inradius of 4ABC , and N is the midpoint of the median through C . Prove that if r = CN ; IN , then AC = BC or ACB = 90 deg :
6

Solution: Use the standart notation for the elements of 4ABC and apply the formula for a median in a triangle. Since IN and IM are medians in 4CIM and 4AIB , respectively, we get IN 2 = 1 (2CI 2 + 2MI 2 ; CM 2) = = 1 (2CI 2 + AI 2 + BI 2 ; 1 AB 2 ; CM 2). 4 4 2
Therefore 1 (a2 + b2 ; 2(p ; c)2 ; 2r2 ; (p ; a)2 ; r2 ; (p ; b)2 ; r2) = (p ; c)c ; r2: 4 2 It follows from the statement of the problem that IN 2 = (CN ; 2 r) , so CN 2 ; IN 2 + r2 = 2CN r. It follows from what we have proved above that 2CM r = (p ; c)c. Taking the square of both 17 1 AB 2 ; 2CI 2 ; AI 2 ; BI 2) = 2 CN 2 ; IN 2 = 1 (2 CM + 4 2

sides of this equality and using the formul 4CM 2 = 2a2 + 2b2 ; c2 b)(p ; c) , we get (2a2 +2b2 ; c2)(p ; a)(p ; b) = and r2 = (p ; a)(p ; p 2 p(p;c)c . After some simple calculations the above equality becomes (a2 + b2 ; c2)(a ; b)2 = 0. Therefore a = b, i. e., AC = BC or a2 + b2 = c2, i. e., ACB = 90 deg.
6

Problem 11.3. (Problem for the Atanas Radev award)


See problem 10.3.

18

Spring mathematics tournament|Kazanl^ ak, 30 March{1 April 1999


Problem 8'1. Given an inequality jx ; 1j < ax, where a is a real
parameter: a) Solve the inequality. b) Find all values of a such that the inequality has exactly two integer solutions. Chavdar Lozanov, Kiril Bankov, Teodosi Vitanov

Solution: a) I. Let x 1. Then the inequality is equivalent to x ; 1 < ax () (1 ; a)x < 1.


1 1 > 1 () a > 0. 1. 1 ; a > 0 a < 1 =) x < 1 ; a 1;a 1 Therefore 0 < a < 1 1 x < 1 ; a . 1

2. 1 ; a = 0 =) a = 1 =) 0:x < 1. Therefore a = 1 x 1. 3. 1 ; a < 0 =) 1 < a =) x 1. Therefore 1 < a x 1. II. Let x < 1. Then the inequality is equivalent to 1 ; x < ax () 1 < (a + 1)x. 1 < x < 1 1 < 1 () a > 0. 1. a +1 > 0 () a > ;1 a + 1 a+1 1 Therefore a > 0 a + 1 < x < 1, when ;1 < a 0 no solution exists. 2. a + 1 = 0 () a = ;1 =) 1 < 0 x, so no solution exists. 1 < 0. Therefore 3. a + 1 < 0 () a < ;1 =) x < a + 1 1 a < ;1 x < a + 1 .
1 So when a < ;1, then x < a+1 for ;1 a 0 no solution 1 1 exists when 0 < a < 1, then a+1 < x < 1; a, then a when 1 1 x. b) It follows from a) that the inequality could have two integer 1 <1< 1 , solutions only if 0 < a < 1. Since in this case 0 < a + 1 1;a we nd that there are exactly two integer solutions if and only if 1 2< 1; a 3: 2. < a Therefore the answer is 1 2 3 2

Problem 8'2. Let M be the midpoint of the side BC of 4ABC


and 6 CAB = 45
6

ABC = 30 .

BC . b) Prove that AM = AB 2AC

a) Find 6 AMC .

Chavdar Lozanov

Solution: a) Draw CH ? AB . Now 6 ACH = 45 = 6 CAH and 6 HCB = 60 . For 4ACH it is true that AH = HC . Further it follows from 4CHB that CH = 1 = HM . Therefore AH = 2 CB 6 HM , so 6 MAH = 6 AMH = 180 ; AHM . Note that 6 CHM =

S = AM AC . . Therefore 45 ; 15 = 30 , and thus CP = AC 2 2 4 AB CB AM AC AB BC Now 4 = 2 , and we obtain AM = 2AC .

2 60 and 6 AHM = 90 + 60 = 150 . Therefore 6 AMH = 15 . We obtain that 6 AMC = 6 HMC ; 6 AMH = 60 ; 15 = 45 . b) Let S be the area of 4ABC . We know that S = AB CH = 2 AB CB . Since AM is a median, it follows that S S AMC = . If 4 2 S AM CP CP ? AM , then 2 = 2 . But 6 CAM = 6 CAB ; 6 MAB =

Problem 8'3. Consider all points in the plane whose coordinates


3

(x y) in an orthogonal coordinate system are integer numbers and 1 x 19 1 y 4. Each point is painted green, red or blue.

Prove that there exists a rectangle with sides parallel to the coordinate axes whose vertices are all of the same colour. Kiril Bankov

Solution: Since the number of coloured points is 4 19 = 76 and

there are three di erent colours, it follows that there are at least 26 points of the same colour (say blue). Denote by p1 p2 : : : p19 the lines parallel to the ordinate axis and passing through the points (1 0) (2 0) : : : (19 0), respectively. Let n1 n2 : : : n19 be the number of blue points on the lines p1 p2 : : : p19. It is clear that 0 ni 4 for i = 1 2 : : : 19. Without loss of generality assume n1 n 2 n19. Since n1 + n2 + + n19 = 26, we obtain that n1 2. 1. Let n1 = 4. Then the remaining 22 blue points lie on 18 lines and so n2 2. Therefore there are two blue points on each of the lines p1 and p2 all having the same ordinates. These points form the required rectangle. 2. Let n1 = 3. Then the remaining 23 blue points lie on 18 lines and therefore n2 2. a) If n2 = 3, then there is a blue rectangle with vertices on p1 and p2. b) Let n2 = 2. Then n3 = n4 = n5 = n6 = 2. The number of ways of choosing two blue points on p1 is 3 and the corresponding number for each of p2 p3 : : : p6 is 1. The total number of blue pairs is 3+5 1 = 8, which is greater that 6|the number of ways of choosing two horizontal lines out of 4 lines. Therefore there exists a blue rectangle. 4

3. Let n1 = 2. Then n2 = n3 = : : : = n7 = 2. Since 7 > 6, we apply the same reasoning as in 2b).

Problem 8.1. Find all rational numbers a such that j4a ; 2j 1 and A = 4a ; 1 is integer. Ivan Tonov
27a4 1 a 3 . Also, it is Solution: It follows from j4a ; 2j 1 that 4 4 1 1

clear that when a 4 , then A 0 and A = 0 only if a = 4 . Let k be a positive integer such that A = k. Then 27a4 ; 4al + l = 0, where l = 1 . Multiply the above equality by 3 and write it in the k following way: 81a4 ; 18a2 + 1 + 18a2 ; 12al + 3l ; 1 = 0 () (9a2 ; 1)2 + 2(3a ; 1)2 ; 12a(l ; 1) + 3(l ; 1) = 0 so (9a2 ; 1)2 + 2(3a ; 1)2 + 3(l ; 1)(1 ; 4a) = 0. Therefore 3(l ; 1 ) only if l 1 or k 1. 1)(1 ; 4a) 0, which is possible (recall a > 4 But since k is a positive integer, it follows that k = 1 and a = 1 3. 1. and a = Therefore the required values are a = 1 4 3

Problem 8.2. Given a 4ABC . Let M be the midpoint of AB ,


6

CAB = 15 and 6 ABC = 30 .


a) Find 6 ACM .

BC . b) Prove that CM = AB 2AC

Chavdar Lozanov

Solution: a) Let AH ? BC . Then 6 HAB = 60 and AH = AB = HM . It follows from 6 HAC = 6 HAB ; 6 CAB = 45 2 6 that AH = HC . Thus HM = HC and 6 HCM = 180 ; 2 MHC . But 6 MHC = 6 AHB ; 6 AHM = 90 ; 60 = 30 . Therefore 6 HCM = 75 , so 6 ACM = 6 HCM ; 6 HCA = 75 ; 45 = 30 . b) Let S be the area of 4ABC . We know that S = BC AH = 2 BC AB . Since CM is a median, we obtain S = S . If MP ? ACM 4 2 S AC PM AC , then 2 = 2 . It follows from 4PMC that PM =
1 MC (6 PCM = 30 ). Therefore S = AC MC , so AC MC = 2 2 4 2 BC AB , which implies MC = AB BC . 4 2AC

Problem 8.3. Given n points on a circle denoted consecutively

by A1 A2 : : : An (n 3). Initially 1 is written at A1 and 0 at all remaining points. The following operation is allowed: choose a point Ai where a 1 is written and replace the numbers a, b and c written at the points Ai;1 Ai and Ai+1 by 1 ; a, 1 ; b and 1 ; c, respectively. (Here A0 means An and An+1 means A1.) a) If n = 1999, is it possible to have a 0 in all points after performing the described operation a nite number of times? 6

b) Find all values of n such that it is not possible to have a 0 in all points after nite number of operations. Kiril Bankov

Solution: a) After performing the transformation from the con(1)

ditions of the problem consecutively for the points A1 A2 A3 : : : An;2, we have the following distribution of 0s and 1s:

A1 A2 A3 : : : An;3 An;2 An;1 An 1 1 1 ::: 1 0 1 1

If n = 1999, then arrange the obtained 1998 ones in 666 groups of three 1s and then perform the operation on each of the groups. We obtain a zero in every point. b) If n = 3k + 1 we can repeat the steps from a) and we get only zeroes in the points. If n = 3k +2, then starting from (1) and performing the operation with An;1 one obtains:

and again to obtain anly zeroes. We shall prove now that if n = 3k, it is not possible to have only zeroes after a nite number of operations. Assume the opposite, i. e., that after a nite number of operations we have only zeroes. Denote the number of operations performed with the point Ai by ai. Since each operation changes the number of ones by an odd number (1 or 3), it follows that the sum S = a1 + a2 + : : : + an of all operations 7

A1 A2 A3 : : : An;2 An;1 An 1 1 1 ::: 1 0 0 There are 3k ones which can be arranged in k groups of 3 ones each

is an odd number. On the other hand S = (a1 + a2 + a3) + (a4 + a5 + a6) + : : : + (a3k;2 + a3k;1 + a3k ). Note that a1 + a2 + a3 is equal to the number of changes (from 0 to 1 or vice versa ) of the number written at A2. Since at the beginning there is a 0 written at A2 and a 0 again at the end, it follows that a1 + a2 + a3 is an even number. The same applies for a4 + a5 + a6 and so on. Therefore S is a sum of even numbers, a contradiction to the fact that S is odd. Answer to b): all numbers divisible by 3.

Problem 9.1. It is known that if the real parameter a equals any of the numbers p < q < r, then at least one of the remaining two is a root of the equation
x2 ; (2 ; a)x + a2 ; 2a ; 7 = 0:
8 b) p < ;1. Prove that a) p > ; 3
Sava Grozdev

Solution: a) It follows from the conditions of the problem that D = (2 ; a)2 ; 4(a2 ; 2a ; 7) 0, so ; 8 3 a 4 and therefore 8 . When a = ; 8 , then the equation has an unique root p ;3 3 7 7 . Thus in 7 and x = x = 3 , and when a = 3 , the roots are x = ; 8 3 3 8 the case of p = ; 3 three distinct numbers p, q and r satisfying the condition do not exist. Therefore p > ; 8 3.
b) Suppose that when a = p, then x = q is a root. The case x = r is treated in the same fashion. Since a and x are symmetric in 8

the equality x2 ; (2 ; a)x + a2 ; 2a ; 7 = 0, we get that when a = q, then x = p is a root. When a = r, at least one of the numbers p and q is a root. Let that be p. Now when a = p, the equation has x = r as a root (because of the symmetry). We obtain that when a = p, the roots are q and r. We conclude now that in all cases p + q + r = 2. It is clear that the roots of x2 ; (2 ; p)x + p2 ; 2p ; 7 = 0 are greater than p and therefore p2 ; (2 ; p)p + p2 ; 2p ; 7 > 0, which gives 2 . Therefore . But 3 p < p + q + r = 2 and p < p < ;1 or p > 7 3 3 p < ;1.

Problem 9.2. Through an interior point K of the non-equilateral 4A1A2A3 lines Q2P3 k A2A3, Q3P1 k A3A1 and Q1P2 k A1A2 are
a) 4P1P2 P3 = 4Q1Q2Q3 4A1A2A3 b) point K , the centre of k and the circumcentre 4A1A2A3 lie on a line. Rumen Kozarev the arcs P1^ Q1 P2^ Q2 and P3^ Q3 are equal. Therefore 6 P3P1P2 = ^P2 P1^ ^P2 P3Q3P2 = P3^ Q3 + Q3 Q1 + Q3 = 2 2 2 2 2 6 . Since the6 quadrilateral A1P1KQ1 is a parallelogram we get A3A1A2 = Q1KP1 = Q1^P1 + Q3^P2 , so 6 P P P = 6 A A A . Similarly, one can show 3 1 2 3 1 2 2 2 9

drawn. (Q1, Q2, Q3 lie on A3A1, A1A2 and A2A3, respectively). Points P1 P2 P3 Q1 Q2 Q3 lie on a circle k. Prove that:

Solution: a) It follows from the conditions of the problem that

that 6 Q3Q1Q2 = 6 A3A1A2, and the same equalities for the remaining pairs of angles. Thus 4P1P2P3 4Q1Q2Q3 4A1A2A3 and since the rst two triangles have the same circumcircle, they are identical. b) Since A1P1KQ1 is a parallelogram, we obtain 6 A1P1Q1 = ^ ^ 6 KQ1 P1 = P2 Q2 + P1 Q2 = 6 P1 P2 P3 = 6 A1A2 A3. Similarly 6 A1Q1P1 = 2 2 6 A1 A2A3. Let O be the circumcentre of 4A1A2A3. It is easy to see that OA1 ? P1Q1,OA2 ? P2Q2 and OA3 ? P3Q3. Denote the midpoint of OK by S . If R1 R2 and R3 are the midpoints of P1Q1 P2Q2 and P3Q3 then SR1 k OA1 SR2 k OA2 SR3 k OA3 (SR1 is a middle segment in 4OKA1 =) SR1 k OA1). Therefore S lies on the axes of symmetry of P1Q1 P2Q2 and P3 Q3, so S is the centre of the circle through the points P1 P2 P3 Q1 Q2 Q3.

Problem 9.3. Find all polynomials f (x) = xn + an;1xn;1 + + a1x + a0 a0 = 6 0 with integer coe cients such that f (ai) = 0 i = 0 : : : n ; 1. Sava Grozdev Solution: It is clear that n > 1. Since f (x) = (x ; a0)(x ; a1) : : : (x ; an;1 ), it follows from f (0) = a0 = (;1)n a0 : : : an;1 that jaij = 1 for i = 1 2 : : : n ; 1. First case: ja0j = 1. Now f (x) = (x ; 1)p(x + 1)q p + q = n > 1. We know that (x ; 1)p(x + 1)q = (xp ; pxp;1 + : : :)(xq + qxq;1 + : : :)
and comparing the coe cients in front of xn;1 and xn;2 we obtain q ; p = an;1 = 1 : q(q;1) + p(p;1) ; pq = a n;2 = 1 2 2 10

It is easy to nd now that p + q = 3 and so p = 1 q = 2 or p = 2 q = 1. In the rst case we get that f (x) = x3 + x2 ; x ; 1, which is a solution and in the second one f (x) = x3 ; x2 ; x + 1, which is not. n;1 + Second case: ja0j 2. Now 0 = f (a0) = jan + 0 + an;1 a0 n;1 ;1) j a n n ; 1 2 0 j(ja0 j;2)(ja0 j a1a0+a0j ja0j ;ja0j ; ja0j ;ja0j;ja0j = ja0 j;1 ;1 : : : a a a have the same 0. Therefore ja0j = 2. Moreover an;1an 1 0 0 0 negativity|the opposite to those of an . 0 We conclude that a0 = ;2 n is an even number and ai = (;1)i+1 for i = 1 2 : : : n ; 1. If n > 2, then a2 = ;1 and 0 = f (;1) = (;1)n + (;1)n (;1)n;1 + + (;1)2(;1) ; 2 = ;n ; 2 6= 0, which is impossible. Therefore n = 2 and f (x) = x2 + x ; 2, which is a solution. Answer: f (x) = x2 + x ; 2 and f (x) = x3 + x2 ; x ; 1:

Problem 10.1. Prove that the inequality


holds for any real number x.

x2 + x + 2 ) 0 x(2 3x ; 4x 2+x+1

Rumen Kozarev

2. We shall show that 4x + x + 2 2. The last inequality is equivalent to 4x2 + x + 2 x2 + x + 1 1 +1 and 2x2 + 2x + 2 () x(2x ; 1) 0 () x 2 (;1 0] 2 therefore it holds for x 0. 11

Solution: 1) Let x 0. Then 2 3x 2

2 4 x + x + 2 ) < 2 3x. Assume 2) Let x > 0. We prove that x2 + x+12 2 x + x + 2 > 2 30 = 4 x + x + 2 x the opposite, i. e., x2 + x + 1 2 3 =) 4x 2+x+1 1 2 () x(2x ; 1) > 0 () x 2 (;1 0) ( 2 +1). Since x > 0, 1 +1 . Therefore 4x2 + x + 2 2 3x > we obtain that x 2 2 x2 + x + 12 1 2 2 2 3 2 > 3 () 4x +p x + 2 > 3x 2x ; 1 > p + 3x + 2 () x ;p 0 () x 2 (;1 1 ; 2) (1 + 2 +1) =) x 2 (1 + 2 +1), 4x2 + x + 2 2 3x > 2 31+p2 > 2 32 = 18. because x 1 . Thus 2 2 x2 + x + 1 2 x + x + 2 < 4x + 4x + 4 = 4 for any x > 0, we Since obviously 4x 2 +x+1 x2 + x + 1 get a contradiction.

Problem 10.2. Let M be an interior point in the square ABCD.

Denote the second points of intersection of the lines AM , BM , CM , DM with the circumcircle of ABCD by A1, B1, C1, D1, respectively. Prove that A1B1 C1D1 = A1D1 B1C1: Emil Kolev

Solution: Since 4ABM 4A1B1M 4BCM 4B1C1M 4CDM 4C1D1M 4DAM 4D1A1M , we obtain
AB = BM BC = BM CD = DM DA = DM : A1B1 A1M B1C1 C1M C1D1 C1M D1 A1 A1M
12

It is easy to see now that

AB CD = BM DM = BC DA : A1B1 C1D1 A1M C1M B1C1 D1 A1 Using that AB = BC = CD = DA it follows from the above that A1B1 C1D1 = A1D1 B1C1, Q. E. D.

Problem 10.3. Consider n points in the plane such that no three


lie on a line. What is the least number of segments having their ends in the given points such that for any two points A and B there exists a point C connected to both A and B ? Emil Kolev connecting A1 with all remaining points. Also, draw segments A2A3, A4A5, . . . , An;1 An when n is odd and A2A3, A4A5, . . . , An;2An;1, A2An when n is even. It is easy to see that the condition of the ; 3 segments (dM e denotes problem is met and that there are 3n 2 the least natural number which is greater or equal to M ). ; 3 segments and Suppose it is possible to draw less than 3n 2 to meet the condition of the problem. Obviously each point is connected by a segment with another one. If each point is connected to at least three others, we will have that the number of segments is n , which is greater than 3n ; 3 . Therefore there exists at least 32 2 a point (let that be A1) connected with at most two others. If A1 is connected to exactly one point (let that be A2), a point connected to both A1 and A2 does not exist. Therefore A1 is connected to exactly two points (let them be A2 and A3). It is easy to see that A2 and A3 13

Solution: Denote the points by A1 A2 : : : An. Draw segments

are connected by a segment. Consider the pairs A1 and Ai for any i > 3. It is clear that the point connected to both A1 and Ai could be either A2 or A3. In both cases Ai is connected to A2 or A3. Since there are at least two segments from each point Ai i > 3 then the number of segments from Ai i > 3 is at least 2(n ; 3). Further, since at least n ; 3 from these points connect some point of Ai i > 3 with A2 or A3 (and therefore they are counted once) the total number of 3 = 3n ; 3 . drawn segments is at least 3 + n ; 3 + n ; 2 2 This is a contradiction with the number of drawn segments. ;3 . Therefore the answer is 3n 2

Problem 11.1. Given a function f (x) de ned for any real x and f (tgx) = sin 2x for x 2 (; 2 2 ). Find the minimum and the maxi3
mum of the function f (sin x) f (cos3 x). Oleg Mushkarov, Nikolai Nikolov 2t and it follows from Solution: Let t = tgx. Then sin 2x = 1 + t2
3 3x 4 sin x cos f (sin x) f (cos x) = (1 + sin6 x)(1 + cos6 x) = 3 3

2t for any t. Therefore the conditions of the problem that f (t) = 1 + t2

4(sin x cos x)3 2 ; 3(sin x cos x)2 + (sin x cos x)6

14

u2 ; 2

1 sin 2x. Then u 2 ; 1 1 ] and we have to nd Let u = sin x cos x = 2 2 2 u3 the minimum and the maximum of the function g(u) = 2 ; 34u 3 + u6 1 1 in the interval ; 2 2 ]. We obtain 2 ; u2)(u4 + u2 + 1) > 0 g0(u) = 12u (1 (2 ; 3u2 + u6)2 1 ] and so g(u) is an increasing function in the inwhen u 2 ; 1 2 2 1 1 1 ) = 32 and terval ; 2 2 ]. It follows now that max g ( u ) = g ( 1 1] 2 81 u2 ; 2 2 32 . g(u) = g(; 1 min ) = ; 1 1 2 81
2]

Problem 11.2.;! A circle is tangent to the circumcircle of 4ABC ;!

and to the rays AB and AC at points M and N , respectively. Prove that the excentre to side BC of 4ABC lies on the segment MN . Oleg Mushkarov, Nikolai Nikolov

Solution: Let O be the circumcentre of 4ABC and L be the centre of the circle tangent to the circumcircle of 4ABC . First we shall nd the radius of this circle. For 4OAL we get AL = A , sin 2 j B ; C j AO = R, OL = R + , 6 OAL = . From the Cosine Law we
obtain (R + )2 = R2 +
2

sin2 A 2

2 C 2R cos B; ; sin A 2 . After simpli cation 2 15

the above becomes


s

A (sin A + cos B ; C ) = 2R sin A cos B cos C : cos2 A = 2 R sin 2 2 2 2 2 2 2


s s

(p ; b)(p ; c) , cos B = p(p ; b) , cos C = p(p ; c) Since sin A = 2 bc 2 ac 2 ab and abc = 4RS , it follows from the previous equality that = ra , where r is the exradius to side BC of 4ABC . Let I =

AL \ MN and let T be the projection of I on the line AB . Since IM = AI AM LM = AL ? MN , it follows that IT = AI AM AM AL AI LM = LM cos2 A = r . Since I lies on the bisector of 6 A, we AL 2 a conclude that I is the excentre to side BC of 4ABC .

cos2 A 2

gin O in the plane. Distinct real numbers are written at the points with integer coordinates. Let A be a nonempty nite set of integer points which is central-symmetric regarding O and O 2 = A. Prove that there exists an integer point X such that if AX is the image of A ~ , then at least half of the numbers under translation de ned by OX written at the points of AX are greater than the number written at X . Avgustin Marinov ously even, by 2s. Connect all integer points X with the points from AX by arrows so that the arrow points to the smaller number. Suppose no point X with the required property exists. Then there are at least s + 1 arrows pointing out of any point X . For every natural 16

Problem 11.3. Given an orthogonal coordinate system with ori-

Solution: Let us denote the number of points in A, which is obvi-

number n denote the square with vertices (n n) (;n n) (;n ;n) and (n ;n) by Kn . Since A is a nite set, there exists a natural number d such that A Kd . For every n denote the number of arrows within the square Kn by Sn . Since there are at least s + 1 arrows pointing out of every integer point of Kn (and these arrows are within the square Kn+d ), it follows that (2n + 1)2(s + 1) Sn+d . On the other hand, since AX is a central-symmetric set, there are at most s ; 1 arrows pointing to every integer point of Kn+d . Therefore Sn+d (2n + 2d + 1)2(s ; 1). Thus (2n + 1)2(s + 1) (2n + 2d + 1)2(s ; 1), so s + 1 (1 + 2d )2(s ; 1) for any n. When n ! 1 one obtains s + 1 s ; 1, 2n + 1 a contradiction. Therefore a point X with the required property does exist.

17

WINTER MATHEMATICAL COMPETITION


1995

Grade 8 | First Group.

\The number 7 is a divisor of 3n + n3 if and only if 7 is a divisor of 3n :n3 + 1." Solution. If 7 is a divisor of n, then 7 is neither a divisor of 3n + n3 nor a divisor of 3n :n3 +1. Let 7 be not a divisor of n. In this case 7 divides n6 ; 1 = (n3 ; 1)(n3 + 1) and since 7 is a prime number, then 7 divides either n3 ; 1 or n3 + 1. Now the above proposition follows from the equalities: 3n :n3 + 1 = (n3 ; 1)(3n ; 1) + (n3 + 3n ) and 3n :n3 + 1 = (n3 + 1)(3n + 1) ; (n3 + 3n ): Problem 2. Let ABCDE be a convex pentagon and let M , P , N , Q be the midpoints of the segments AB , BC , CD, DE respectively. If K and L are the midpoints of the segments MN and PQ respectively and the segment AE is of length a, nd the length of the segment KL. Figure 1. Figure 2.

Problem 1. Prove that for every positive integer n the following proposition holds:

Solution. Let F be the midpoint of the segment AD (Figure 1). Then the quadrilateral MPNF is a parallelogram. Hence K is midpoint of the segment FP . It follows from here that 1 :FQ. On the other hand FQ = 1 :a (because F and Q are midpoints of ED and AD KL = 2 2 respectively). Therefore KL = 1 4 :a.

there exists a right angled triangle with hypotenuse of length 2 and an acute angle of 60 , which vertices are colored in one and the same colour. Solution. First we shall show that there exist two points which are colored in one and the same colour and the distance between them is 2. Indeed let ABC be a equilateral triangle of side 2. Obviously two of its vertices (say A and B ) are colored in one and the same colour (e.g. white). Let AXY BZT be a regular hexagon with a big diagonal AB (Figure 2). If one of the vertices X , Y , Z , T (e.g. X ) is white, then the vertices of the triangle ABX (6 AXB = 90 , 6 XAB = 60 ) are colored in one and the same colour. Otherwise the vertices of the triangle XY T (6 Y XT = 90 , 6 XY T = 60 ) are colored in one and the same colour.
Grade 8.

Problem 3. Every point in the plane is colored either in black or in white. Prove that

for which the number C = 2A + B is an integer. 3 Solution. We have 2(x ; 1) = 2(x ; 1) p A = p 1 2 = j2x 1 B = + 1j (2x + 1) (x ; 1)2 jx ; 1j and 1 + x;1 C=2 3 j2x + 1j jx ; 1j 1. Let x > 1. Then

1 and B = p 22x ; 2 . Find all integer values of x, Problem 1. Let A = p4x2 + 4x + 1 x ; 2x + 1

and

C=2 3

1 4(x + 1) 2x + 1 + 1 = 3(2x + 1) > 0

Hence 0 < C < 1, i.e. C is not an integer for any x > 1. 1 < x 1. Then x = 0 (because x is an integer) and C = 0. Thus x = 0 is a 2. Let ; 2 solution of the problem. 3. Let x < ; 1 2 . Then x ;1 (because x is an integer). It is clear that 2 ; 1 ; 1 = ; 4 (x + 1) 0 C=3 2x + 1 3(2x + 1) and

4(x + 1) ; 1 = 1 ; 2x < 0: C ; 1 = 3(2 x + 1) 3(2x + 1)

1 ; 1 + 1 = 1 ; 4(x + 1) = 2x ; 1 > 0: C +1= 2 ; 3 2x + 1 3(2x + 1) 2 (2x + 1) Hence ;1 < C 0, i.e. C = 0 and x = ;1. Finally only x = 0 and x = ;1 are solutions of the given problem. Problem 2. Let M and N be the midpoints of the sides BC and AC of the triangle ABC , (AB = 6 AC , AB = 6 BC ) and G be the intersection point of the lines AM and BN . The angle bisectors of 6 BAC and 6 ABC intersect BC and AC in the points D and E respectively. Prove 2

that the quadrilateral DEMN is inscribed in a circle if and only if there exists a circle, inscribed in the quadrilateral CMGN . Figure 3. Figure 4.

Solution. Without loss of generality we can suppose that N is between A and E . There are two possibilities for the points D and E which are shown in the Figure 3 and Figure 4. The quadrilateral DEMN is inscribed in a circle if and only if 6 CNM = 6 CDE , i.e. 6 BAN + 6 BDE = 1800 (because MN kAB and 6 CNM = 6 BAN ). Thus the quadrilateral DEMN is inscribed in a circle i the quadrilateral ABDE is inscribed in a circle. This is equivalent to 6 DAE = 6 DBE , i.e. to AC = BC . Therefore we should prove that there exists a circle, inscribed in the quadrilateral CMGN if and only if AC = BC . Let AC = BC . Then CM = CN and since G is the cenFigure 5. 1 ter of gravity of the triangle ABC , we have GM = 3 AM = 1 BN = GN . Hence CM + GN = CN + GM , i.e. there ex3 ists a circle, inscribed in the quadrilateral CMGN (Figure 5). Conversely if there exists a circle, inscribed in the quadrilateral CMGN , then CM + GN = CN + GM and 1 2 BC + 1 BN = 1 AC + 1 AM . Hence AM ; BN = 3 (BC ; AC ). 3 2 3 2 Let K and L be the points of contact of the circle and the sides CM and CN respectively. Obviously CK = CL. On the other hand this circle is inscribed in the triangles 1 (AC + CM ; AM ), CL = ACM and BCN . Hence CK = 2 1 (BC + CN ; BN ). Thus AM ; BN = 1 (AC ; BC ). It follows from here that 3 (BC ; AC ) = 2 2 2 1 (AC ; BC ), i.e. AC = BC . 2 Problem 3. Thirty points are given in the plane. Some of them are connected with segments as it is shown in the Figure 6. The points are labeled with di erent positive integers. If a is a segment and p and q are the numbers, corresponding to its endpoints, we denote (a) = jp ; q j. a) Construct an example of labeling of the points with the integers 1 2 : : : 30, in which there

exists exactly one segment a with (a) = 5 b) Prove that for every labeling there exists at least one segment a with (a) 5. Figure 6. Figure 7.

Solution. a) A possible example is shown in the Figure 7. b) Let the points be labeled with the positive integers m = m1 < m2 < : : : < m30 = M . It is clear that M m + 29. Let A and B be the points labeled with m and M respectively and let a1 = AC1 , a2 = C1C2, : : :, ak;1 = Ck;2Ck;1, ak = Ck;1B be the shortest path of segments, connecting A and B . It is not di cult to see that k 7. If we assume that (ai ) < 5 for i = 1 2 : : :k, then m + 29 M m + 4k m + 28, which is a contradiction.
Grade 9.

Problem 1. Let m be a real number, such that the roots x1 and x2 of the equation f (x) = x2 + (m ; 4)x + m2 ; 3m + 3 = 0
are real numbers. 2 a) Find all values of m for which x2 1 + x2 = 6. b) Prove that 2 2 1 < mx1 + mx2 + 8 1 ; x1 1 ; x2 Solution. a) Since x1 and x2 are real numbers, then Hence ; 2 3 121 : 9

D (f ) = (m ; 4)2 ; 4 m2 ; 3m + 3 = ;3m2 + 4m + 4 0: m 2. On the other hand


2 2 2 6 = x2 1 + x2 = (x1 + x2 ) ; 2x1x2 = ;m ; 2m + 10: p We obtain from here that m = ;1 5. But p 2 < ;1 + p5 < 2 ;1 ; 5 < ; 3

and therefore only m = 5 ; 1 is a solution of the given problem. b) We have


2 mx2 mx2 m x2 1 2 1(1 ; x2 ) + x2 (1 ; x1 )] + = 1 ; x1 1 ; x2 f (1) 2 2 = x1 + x2 ; x1x2 (x1 + x2 ) m;2 3 2 = m ; 8m + 13m ; 2 = m2 ; 6m + 1 m;2

mx2 + 8, then F = (m ; 3)2 and 1 Thus if F = 1mx + ; x1 1 ; x2 121 = ; 2 ; 3 2 F > (2 ; 3)2 = 1: 9 3 Problem 2. The point D lies inside the acute triangle ABC . Three of the circumscribed circles of the triangles ABC , ABD, BCD and CAD have equal radii. Prove that the fourth circle has the same radius. Solution. There are two cases: 1. The radii of the circumscribed circles of the Figure 8. triangles ABD, BCD, CAD are equal. Let O1 , O2 and O3 be the centers of these circles (Figure 8). Obviously the quadrilaterals O2 CO3D, O3 AO1D and O1BO2 D are rhombuses. Hence O2C kO3DkAO1 and O2 C = AO1 . Thus the quadrilateral CAO1O2 is a parallelogram. It follows from here that AC kO1O2 and since O1 O2 ? DB , then BD ? AC . Analogously CD ? AB and AD ? BC . Therefore D is the altitude center of 4ABC . Now it is easy to see that 6 BDC = 1800 ; 6 BAC , which implies that the circumscribed circles of 4BCD and 4ABC are symmetric according to the line BC . Therefore their radii are equal. 2. The radii of the circumscribed circles of the triangles ABC , ACD and BCD are equal. In this case the circumscribed circles of 4ABC and 4BCD are symmetric according the line BC . Hence 6 BDC = 1800 ; 6 BAC . Analogously 6 ADC = 1800 ; 6 ABC . Therefore 6 ADB = 3600 ; 6 BDC ; 6 ADC = 1800 ; 6 ACB and the circumscribed circles of 4ABD and 4ABC are symmetric according to the line AB. Thus they have equal radii. Problem 3. Let A be a set with 8 elements. Find the maximal number of 3-element subsets of A, such that the intersection of any two of them is not a 2-element set. Solution. Let B1 B2 : : : Bn be subsets of A such that jBi j = 3, jBi \ Bj j 6= 2 (i j = 1 : : : n). Assume that there exists an element a 2 A, which belongs to four of the subsets B1 B2 : : : Bn (e.g. a 2 B1 B2 B3 B4). Then jBi \ Bj j 1 (i j = 1 : : : 4). But Bi = 6 Bj if i 6= j , i.e. jBi \ Bj j 6= 3. Thus jBi \ Bj j = 1 (i j = 1 2 3 4). It follows from here that jAj 1 + 4:2 = 9,
2 2

which is a contradiction. Therefore every element of A belongs to at most three of the subsets B1 B2 : : : Bn . Then 3n 8:3, i.e. n 8. If A = fa1 a2 : : : a8g, then the subsets

B1 = fa1 a2 a3g B2 = fa1 a4 a5g B3 = fa1 a6 a7g B4 = fa8 a3 a4g B5 = fa8 a2 a6g B6 = fa8 a5 a7g B7 = fa3 a5 a6g B8 = fa2 a4 a7g
provide an example of exactly eight 3-element subsets of A, such that jBi \ Bj j 6= 2. Therefore the searched number is n = 8.
Grade 10.

Problem 1. Find all positive roots of the equation


where a > 1 is a real number. Solution. It is clear that if a > 1 and x > 0, then x + a ; 1 > 0 and 4 > 0. Hence in x+1 4 this case log(x+a;1) x + 1 is well de ned. Since 4 log a x+1 log(x+a;1) 4 = x + 1 loga (x + a ; 1) then the given equation is equivalent to 4 = 2loga(x+a;1) : (1) x+1 4 is strictly decreasing in the interval (0 +1). The function log (x + a ; 1) The function x + a 1 is strictly increasing in the interval (0 +1) and obviously the same is true for the function 2loga (x+a;1) . Therefore the equation (1) has no more than one root in the interval (0 +1). On the other hand it is easy to check that x = 1 is a root of this equation. Problem 2. A circle k with center O and diameter AB is given. The points C and D are _ moving along the arc AB so that C is between B and D and if 6 BOC = 2 and 6 AOD = 2 , then tan = tan + 3 2 . Prove that the lines, which are perpendicular to CD and divide CD in ratio 1 : 4 measured from C , pass through a xed point of the given circle. _ Solution. Let E be such a point on the arc AB , not containing C and D, that if 6 BOE = 2 , then tan = 2 (Figure 9). We shall show that the point E satis es the problem's conditions. It is enough to prove that if F is the foot of the perpendicular from E to CD, then the point CF = 1 . F is between C and D and FD 4 We have tan(6 ECD) = tan(6 ECA + 6 ACD) = tan( 2 ; + ) 6 4 = log 2 log(x+a;1) x + a 1

1 + 2 tan + 3 2 = 4 + 2 tan = 8 + 4 tan = 1 ; tan 3 1 ; 2 tan 2 ; tan + 2 2 + tan = 2 + tan : tan(6 EDC ) = tan(6 EDB + 6 BDC ) = tan( + ) = 1tan ; tan : tan 1 ; 2 tan Thus Figure 9. tan(6 ECD) = 4 tan(6 EDC ): It follows from the last equality that either
6

1 + tan tan 2 ; + tan 2 tan = = 2 tan = 12+ ; tan 1; 2 1 ; tan 2 ; : tan

ECD

or
6

2 and
6 6

EDC

But these angles belong to the triangle EDC . Hence they are acute angles. Therefore the point F is between the points C and D. Since

ECD < 2 and

EDC < 2 :

EF , i.e. FD = 4:FC . then EF = 4 FC FD Problem 3. Find all prime numbers p, for which the number p(2p;1 ; 1) is a k-th power (k > 1) of a positive integer. Solution. Let p(2p;1 ; 1) = xk (x > 0 is an integer). It is clear that p 6= 2, i.e. p = 2q + 1 is an odd number. Since p=x, then x = p:y (y is a positive integer) and (2q ; 1)(2q + 1) = pk;1 y k . At least one of the numbers 2q ; 1 and 2q + 1 is a k-th power of an integer, because they are relatively prime numbers. 1. Let 2q ; 1 = z k , i.e. 2q = z k + 1. If k is even, then z k + 1 is not divisible by 4 . Hence q = 1, p = 3 and p(2p;1 ; 1) = 32. If k = 2l + 1 then 2q = (z + 1)(z 2l ; z 2l;1 + ; z + 1), i.e. z + 1 = 2 , where 0 < q. On the other hand 2q = (2 ; 1)2l+1 + 1 = 22 :A + 2 (2l + 1) (A is an integer). The last equality contradicts with < q . 2. Let 2q + 1 = z k , i.e. 2q = z k ; 1. If k is odd, we obtain a contradiction as in the previous case. If k = 2l, then (z l ; 1)(z l + 1) = 2q and since GCD(z l ; 1 z l + 1) = 2, we have z l ; 1 = 2, i.e. q = 3, p = 7, p(2p;1 ; 1) = 7:63 = 212. The sought numbers are p = 3 and p = 7.
7

EF 6 tan(6 ECD) = EF FC and tan( EDC ) = FD

Grade 11.

Problem 1. Find all the values of the real parameter p, for which the range of the function p) + cos x : f (x) = 2(1p; ; sin2 x
contains the interval 1 2]. Solution. Let y = cos x. The problem is to nd all values of p, such that for every k 2 1 2] the equation 2(1 ; p) + y = k p ; 1 + y2 has at least one root y0 2 ;1 1], i.e. we should nd all values of q = 1 ; p such that for every k 2 1 2] the equation ky 2 ; y ; q (k + 2) = 0 2 6= q , y 2 ;1 1]. has at least one root y0 0 2 = q , then ;y ; 2q = 0, y = ;2q and 4q 2 = q , i.e. q = 0 or q = 1 . If y0 0 0 1 2 4 1 2 If q = 0 the roots of the equation ky ; y = 0 are y1 = 0, y2 = . But y2 = 1 2 ;1 1] and k k 2 6= q = 0 for every k 2 1 2]. Thus q = 0 satis es the problem's conditions. y2 If q = 1 , then the roots of the equation ky 2 ; y ; 1 (k + 2) = 0 are y1 = ; 1 , y2 = 1 + 1 . 4 4 2 2 k 1 But y2 2 ;1 1] only if k = 2. Thus q = 4 doesn't satisfy the problems conditions. 1 . The equation ky 2 ;y ;q (k +2) = 0 has real roots i D = 1+4k(k +2)q Let q 6= 0 and q 6= 4 1 . The vertex of the parabola 0, i.e. i q ; 4k(k1+ 2) for every k 2 1 2]. Hence q ; 32 g(y) = ky2 ; y ; q(k + 2) has as its rst coordinate y 0 = 21k 2 (0 1] ;1 1]. Therefore the equation g (y ) = 0 has at least one root y0 2 ;1 1] i at least one of the inequalities g (;1) 0 k + 1 for every k 2 1 2]. Hence and g (1) 0 holds. It is easy to obtain from here that q k +2 2 1 2 1 q 3 , i.e. q 2 ; 32 3 and q 6= 4 . 1 33 , p 6= 3 . Finally p 2 3 32 4 Problem 2. The point O is on the edge AB of the tetrahedron ABCD. The circumscribed sphere of the tetrahedron AOCD intersects the edges BC and BD in the points M and N (M 6= C , N 6= D) respectively. The circumscribed sphere of the tetrahedron BOCD intersects the edges AC and AD in the points P and Q (P 6= C , Q 6= D) respectively. Prove that the triangles OMN and OQP are similar. Solution. Since the points A, C , M and O are in one and the same plane and are lying on a sphere, then the quadrilateral ACMO is inscribed in a circle (Figure 10). Then 4BOM BM OQ AQ 4BCA, and OM CA = BA (Figure 11). Analogously from 4ABD: BD = AB . Hence

OM = BM AC : OQ AQ BD
8

(1)

BM , and from 4ACD: PQ = AQ . Thus = From 4BCD we have MN CD BD CD AC Figure 10. Figure 11.

MN = BM AC : PQ AQ BD MN ON MN It follows from (1) and (2) that OM OQ = PQ . Similarly OP = PQ . Therefore 4OMN 4OQP .

(2)

Problem 3. Solve in positive integers the equation:

1 + 5x = 2y + 2z :5t:
Solution. If y 2 and z 2, then the right side of the given equation is divisible by 4. But 1 + 5x 2 (mod 4) and hence min(y z ) = 1. On the other hand 2y 1 (mod 5). Thus y is divisible by 4 (4 is the index of 2 modulo 5). It follows from here that y 4, z = min(y z ) = 1 and the equation is 1 + 5x = 24y0 + 2:5t, where y = 4y0 (y0 is a positive integer). If t = 1 then 5x ; 24y0 = 9. Using congruence modulo 3 we obtain that x is even, i.e. x = 2x0. Hence (5x0 ; 22y0 )(5x0 + 22y0 ) = 9, from where x0 = y0 = 1, i.e. x = 2 and y = 4. Let t > 1. Then 16y0 1 (mod 25) and y0 is divisible by 5 (5 is the index of 3 modulo 25). Hence y0 = 5y1 and 1 + 5x = 220y1 + 2:5t. On the other hand 210 1 (mod 11) and 5x 2:5t (mod 11). Obviously x > t. Thus 5x;t 2 (mod 11), which is not true. Therefore the given equation has a unique solution (x y z t) = (2 4 1 1).
Grade 12.

p 2Problem 1. For every real number x we denote by f (x) the maximal value of the function t + 2t + 2 in the interval x ; 2 x]. a) Prove that f (x) is an even function and nd its minimal value b) Prove that the function f (x) is not di erentiable for x = 0. c) Prove that the sequence an = ff (n)g, n = 1 2 : : : is convergent and nd its limit. (For every real number a we denote with fag the unique real number in the interval 0 1) for which the number a ; fag is an integer.)
9

Solution. It is clear that the maximal value of the function t2 + 2t + 2 in the interval x ; 2 x] is reached at the endpoints of this interval. Since the inequality (x ; 2)2 +2(x ; 2)+2 x2 + 2x + 2 is equivalent to x 0, then 8p 2 > if x 0 < x ; 2x + 2 f (x) = > p : x2 + 2x + 2 if x 0:

a) Obviously f (x) = f (;x), i.e. f (x) is an even function and its minimal value is f (0) = 2. b) Since p2 p p f ( x ) ; f (0) x ; 2 x + 2 ; 2 2 = lim = ; lim x!0 x ! 0 x;0 x 2 and
x<0

then f 0(0) doesn't exist. np o p c) It is clear that an = n2 + 2n + 2 . On the other hand n + 1 < n2 + 2n + 2 < n + 2, p i.e. an = n2 + 2n + 2 ; (n + 1). Therefore 1 p = 0: a = nlim nlim !1 n !1 n2 + 2n + 2 + n + 1 Problem 2. The points A, B, C and D lie on Figure 12. a straight line in the given order. A circle k passes through the points B and C and AM , AN , DK and DL are tangents to k. a) Prove that the points P = MN \ BC and Q = KL \ BC don't depend on the circle k. b). If AD = a, BC = b (a > b) and the segment BC is moving along AD, nd the minimal length of the segment PQ. Solution. a) From the Steward's formula for the triangle AMN and the segment AP (Figure 12) we have:

p2 p p f ( x ) ; f (0) x + 2 x + 2 ; 2= 2 lim = lim x!0 x!0 x ; 0 x 2 x>0 x>0

x<0

AP 2 :MN = AM 2:NP + AN 2:MP ; MN:MP:NP = (AM 2 ; MP:NP ):MN


(here AM = AN because they are tangents to a circle). Hence AP 2 = AM 2 ; MP:NP = AB:AC ; BP:CP = AB:AC ; (AC ; AP )(AP ; AB ) = 2:AB:AC ; AP (AB + AC ) + AP 2 10

2AB:AC . Analogously DQ = 2DB:DC . These equalities show that the position i.e. AP = AB + AC DC + DB of the points P and Q doesn't depend on the circle k. b) Let us denote AB = x, BC = y and CD = z . It follows from a) that (x + y ) 2z (y + z ) PQ = AD ; AP ; DQ = x + y + z ; 2x y + 2x ; y + 2z 2x + z 1 ; 2(y + z ) = (x + y ) 1 ; y + 2x y + 2z
2 + y + z) : = (yy+(x 2x)(y + 2z ) Having in mind that y = b, x + y + z = a, we obtain 2 2 PQ = (a + x ; zb)(a = 2 ba 2 a + z ; x) a ; (x ; z)

b2 : a

CD = x = z = a ; b . p:q.
Solution.

2 Therefore the minimal length of the segment PQ is b and this length is reached i AB =

2 Problem 3. Find all prime numbers p and q , such that the number 2p + 2q is divisible by

Proof. Assume that k divides 2k;1 + 1. Obviously k is odd. Let k = p1 1 p2 2 : : :pr r , where p1 p2 : : : pr are odd prime numbers (pi 6= pj if i 6= j ), 1 2 : : : r are positive integers and r 1. Let pi ; 1 = 2mi :ti, where ti are odd integers ( i = 1 2 : : : r). Let m1 be the smallest number in the sequence m1 m2 : : : mr . It follows from pi 1 (mod(pi ; 1)) that pi 1 (mod 2mi ) and pi i 1 (mod 2mi ), i = 1 2 : : : r. Hence k ; 1 = 2m1 :u (u is an integer). If m1 :u k ; 1 2 2 ;1 (mod k), then 2 ;1 (mod k) and 2(p1;1):u ;1 (mod p1), because t1 is p ; 1 odd. But 2 1 1 (mod p1 ) | a contradiction. p q Let 2 + 2 is divisible by p:q . We have three cases: 1. p and q are odd prime numbers. Then 2p + 2q 0 (mod p) and since 2p 2 (mod p), then 2q ;2(modp) and 2pq (;2)p ;2 (mod p). Similarly 2pq ;2 (mod q ). Thus 2pq;1 ;1 (mod pq ), which is a contradiction with the lemma. 2. p = 2, q > 2. Then 4 + 2q 0(modq ) and it follows from 2q 2 (mod q ) that 6 0(mod q ) and q = 3. It is clear that 22 + 23 = 12 0 (mod 2 3). 3. p = q = 2. Then 22 + 22 = 8 0 (mod 2 2). Therefore the sought numbers are: p = q = 2 p = 2, q = 3 p = 3, q = 2.

Lemma. If k > 1, then k doesn't divide 2k;1 + 1.

11

WINTER MATHEMATICAL COMPETITION


1996

Grade 8

Problem 1. For which integer values of the parameter a the equation j2x + 1j + jx ; 2j = a has an integer solutions? Solution. We shall consider the following cases: I. x > 2. Then 2x + 1 > 0, x ; 2 > 0 and the equation is equivalent to 2x + 1 + x ; 2 = a. a 1+a Thus, x = 1 + 3 , which is a solution when 3 > 2, i.e. when a > 5. II. ; 1 2 x 2. Then 2x +1 0, x ; 2 0 and the equation has the form 2x +1 ; (x ; 2) = a. Thus, x = a ; 3, which is a solution when ; 1 a ; 3 2, i.e. 5 a 5. In this interval the 2 2 integers are a = 3 4 5 and respectively we get x = 0 1 2, which are integer solutions. 1 . Then 2x+1 < 0, x;2 < 0 and the equation is equivalent to ;(2x+1);(x;2) = III. x < ; 2 a , which is a solution, when 1 ; a < ; 1 , i.e. when a > 5 . a or ;3x = a ; 1. Thus x = 1 ; 3 3 2 2 5 According to the considered cases the given equation has solution when a > 2 . When a = 3 the only integer solution is x = 0. When a = 4 there are two integer solutions x = 1 and x = ;1. When a = 5, x = 2 is the only integer solution. When a > 5 the equation has two solutions a and x = 1 ; a . For these solutions we have: if a = 3k ; 1, then x is integer only x1 = 1 + 2 1 3 3 if a = 3k + 1, then x2 is integer only if a = 3k, there is no integer solution. As a result, we get integer solutions when a = 3 or a = 3k 1, where k is a positive integer, greater than 1. Problem 2. The bisector AD (D 2 BC ) of the acute isosceles triangle ABC divides it into two isosceles triangles. Let O and I be the incenter and the circumcenter of 4ABC , respectively. AO meets BC in point E , while F is the intersection point of the lines BI and DO. Prove that: a) the quadrilaterals ABEF and ADCF are rhombi with equal side lengths b) If H is the altitude center of 4ABE , then the points A, D, E , F , H are concyclic. Solution. Firstly, let us justify the position of AD. If it is a bisector of the angle between the two equal sides, then it is perpendicular to BC , 4ADB and 4ADC are isosceles. Consequently AD = BD = CD and 6 BAD = 6 CAD = 45 , i.e. 6 BAC = 90 , which contradicts to the condition that 4ABC is acute. It follows that AD is bisector of the angle belonging to the base AB, while 4ABD and 4ACD are isosceles (Figure 1). a) If 6 BAD = 6 CAD = , then 6 ABC = 2 . Since 6 ADB > 6 CAD = 6 BAD , the only possibility is 6 ADB = 6 ABC = 2 and AD = AB . But 6 ACD = 6 ADB ; 6 CAD = 2 ; =
1

, i.e. 6 ACD = 6 CAD and CD = AD. From the equality 2 +2 + = 180 we de ne = 36 . Thus, 6 ACB = 36 , 6 BAC = 6 ABC = 72 . The points O and I lie on the bisector of 6 ACB , which is a segment bisector of AB . We have OA = OC and 6 OAC = 6 OCA = 18 = 6 OAD, while 6 OAB = 36 + 18 = 54 , 6 AEB = 180 ; 72 ; 54 = 54 = 6 BAE . It follows that BE = AB and the bisector BI of 6 ABC is the segment bisector of AE . In the triangle ACD, which is isosceles, we have that AO and CO are bisectors and it follows that DO is the bisector of 6 ADC and the segment bisector of AC . Then F is the intersection point of the segment bisectors of AE and AC , i.e. F is the circumcenter of 4ACE , the radius of the circumcircle is AF = EF = CF . We have 6 AFE = 26 ACE = 72 , while 6 AFB = 6 BFE = 36 = 6 ABF = 6 EBF and consequently AF = AB = BE = EF , i.e. ABEF is a rhombus. In the same way CF = AF = AB = AD = CD and ADCF is a rhombus, which side is AD = AB . b) It is clear that 6 AEF = 6 AEB = 54 and Figure 1. 1 6 ADF = 6 ADC = frac12(180 ; 72 ) = 54 , 2 the segment AF is seen from the points D and E under 54 . We draw a line through A, perpendicular to BC , which intersects BI in the point H | the altitude center of 4ABE , because BI ?AE . But in 4ABD the altitude AH is a bisector of 6 BAD = 36 . Thus, 6 BAH = 18 , 6 AHF = 18 + 36 = 54 = 6 AEF = 6 AOF . Note that the points D, E , H are in one and the same semiplane with respect to AF . Consequently, D, E , H together with A and F are concyclic. Problem 3. Every day a student preparing himself for the Winter competition in mathematics has been solving problems during a period of 5 weeks. He has been solving at least one problem daily but no more than 10 problems weekly. a) Prove that during some consecutive days the student has solved 19 problems exactly. b) If 1 n 34 is a natural number, prove that during some consecutive days the student has solved n problems exactly. Remark. Every week begins on Monday and ends on Sunday. Solution. Since a) is a particular case of b), we shall solve b) only. According to the condition the student has been solving problems during 5 7 = 35 days and has solved at most 5 10 = 50 problems. Let xi be the number of the problems, solved during the i-th day (i = 1 2 : : : 35). Let 1 n 34 be a xed natural number. We want to prove that there exist such k < l, that xk+1 + + xl = n. Denote Xi = x1 + + xi . Obviously, 1 X1 < X2 < : : : < X35 50 and the problem is to prove the existence of such k < l, that Xl ; Xk = n. Case 1. 1 n 19. We consider the numbers

X1 < X2 < : : : < X35 X1 + n < X2 + n < : : : < X35 + n

(1)

which are integers and their number is 70. Obviously they are in the interval 1 50 + n], in which there are 50 + n 50 + 19 < 70 integers. Consequently, among the numbers (1) there 2

are at least two equal. The rst 35 of them as well as the next 35 are di erent from each other. Therefore, there exist such k and l, for which Xl = Xk + n, i.e. for which Xl ; Xk = n. Case 2. 20 n 34. Firstly, we shall prove the following Lemma. If the integers z1 z2 : : : zm belong to the interval 1 2n] and if m > n, then among the numbers z1 z2 : : : zm there are two, the di erence of which is equal to n exactly. Proof. With the numbers from the interval 1 2n] we construct the following pairs: (1 n + 1) (2 n + 2) : : : (n 2n): The number of these pairs is n and the di erence of the numbers in each pair is equal to n. Since m > n, at least two of the numbers z1 z2 : : : zm belong to one and the same pair. Therefore their di erence is equal to n.

Let us nish now the solution of the problem. If n 25, then 2n 50, and thus all the numbers X1 X2 : : : X35 are in the interval 1 2n]. On the other hand n < 35 and according to the lemma there are two numbers among X1 X2 : : : X35 which di erence is n. If 20 n 24, we represent the interval 1 50] as an union of the intervals 1 2n] 2n +1 50]. In the second one there are 50 ; (2n + 1) + 1 = 50 ; 2n integers. Then, the number of the integers among X1 X2 : : : X35, which belong to the interval 1 2n], is at least 35 ; (50 ; 2n) = 2n ; 15 40 ; 15 = 25 > n. Consequently, we can apply the lemma again. Remark. The case 1 can be solved by the lemma proved above.
Grade 9

parameter. a) Prove that f (3 ; p) = 0. b) Find all values of p, for which two of the roots of the equation p f (x) = 0 are lengths of the cathetuses of a rectangle triangle which hypotenuse is equal to 4 2. Solution. a) We have f (x) = (x + p ; 3)(x2 ; 2(p p + 1)x + 4(p ; 3)). b) The roots of f (x) = 0 are x1 2 = p + 1 p2 ; 2p + 13 and x3 = 3 ; p. If p > 3, p 2 2 then x1 > 0, x2 > 0 and x3 < 0. The equation x2 gives p = 1, which is 1 + x2 = 32 = 4 2 impossible. If p = 3, then two of the roots are equal to 0 and this casep gives no solution. If p < 3, 2 2 2 2 then x1 > 0, x3 > 0 and x2 < 0. Therefore, 32 = x2 1 + x3 = (p + 1 + p ; 2p + 13) + (3 ; p) . This equation is equivalent to (p + 1)(3(p ; 3) + 2 p2 ; 2p + 13) = 0: If p = 6 p ;1, we get 2 p2p ; 2p + 13 = 3(3 ; 0 and pp) > 0, from where 5p2 ; 46p + 29 = p 23 8 6 23 + 8 6 23 ; 8 6 23 ; 8 6. p = 5 . Since > 3 and < 3, we nd p 1 = ;1 and p2 = 5 5 5 Problem 2. The incenter of the quadrilateral ABCD is O. The lines lA ? OA, lB ? OB , lC ? OC and lD ? OD are drawn through the points A, B, C and D respectively. The lines lA and lB meet each other in the point K , lB and lC | in L, lC and lD | in M , lD and lA | in N. a) Prove that the lines KM and LN meet each other in the point O. 3
p q

Problem 1. Let f (x) = x3 ; (p + 5)x2 ; 2(p ; 3)(p ; 1)x + 4p2 ; 24p + 36, where p is a real

b) If the lengths of the segments OK , OL and OM are p, q and r respectively, nd the length of the segment ON . Solution. a) We shall prove that the points N , O and L are colinear. Denote 6 ABC = 6 B , 6 BCD = 6 C , 6 CDA = 6 D and 6 DAC = 6 A. Since O is incenter, the segments OA,OB ,OC and OD are the bisectors of the corresponding angles of the quadrilateral ABCD. Note that each of the quadrilaterals AKBO, BLCO, CMDO and DNAO is inscribed. Consequently: 6 NOK + 6 KOL = ; Figure 2. 6 ONA ; 6 OKA + ; 6 OKB ; 6 OLB = 6 6 ; ADO ; 6 ABO6 + ;6 6 BAO ; 6 BCO = 6 D B + A + C) = 2 ; = . 2 ;( 2 + 2 2 2 It follows from here that the points N ,O and L are colinear. Analogously, the points K ,O and M are colinear. Therefore, O is the intersection point of the diagonals of the quadrilateral KLMN . b) Firstly, we shall prove that the quadrilateral KLMN is inscribed. Indeed, 6 NKL + 6 NML = =
6 6

Thus, OK:OM = OL:ON , from where

AKO + 6 OKB + 6 DMO + 6 OMC B +6 A+6 C +6 D =2 : 2 2 2 2

lines. Let A be the set of the vertexes of the unit squares which are not on the sides of the given square. How many points from A can be chosen at most in a way that no three of them are vertexes of isosceles rectangle triangle? Solution. We shall prove that the maximal number is 6. Let us enumerate the points in the way, shown on the table 1. It is easy to be seen that no 3 of the points 1,2,3,8,12 and 16 are vertexes of a isosceles rectangle triangle. Assume that there exists a set of 7 points with the desired property. Note that if 4 points form a square, then at most 2 of them can be among the already chosen ones. The points 1,4,16 and 13 2,8,15 and 9 3,12,14 and 5 form squares. Consequently, at most 6 of the chosen points lie on the contour. It 13 14 15 16 9 10 11 12 follows from here that at least one of the points 6,7,10 and 11 is from the 5 6 7 8 chosen ones. Due to the symmetry we may assume that this is the point 7. 1 2 3 4 Since the points 7,16 and 9 1,7 and 14 form isosceles rectangle triangles, then at most two of the points 1,9,14 and 16 are from the chosen ones. The points 5,7,13 and 15 form a square and therefore at most one of the Table 1: points 5,13 and 15 is from the chosen ones. It follows from here that at least 3 points are chosen from 2,3,4,6,8,10,11 and 12. By the pigeonhole principle we deduce that at least two points are chosen in one of the sets 3,6,11,8 and 2,4,10,12. It is easy to see that if the two points are in the rst set, then we have two possibilities | 3 and 11 or 6 and 8 (in both 4

Problem 3. A square with side length 5 is divided into unit squares by parallel to its sides

p:r ON = OK:OM OL = q :

cases it is not possible to chose more points on the square which encounters 7). Analogously, if the two points are in the second set, then the possibilities are two again | 2 and 12 or 4 and 10 (in both cases it is not possible to chose more points on the square which encounters 7). The contradiction shows that the maximal number of points which can be chosen is equal to 6.
Grade 10

Solution. There are two possibilities for the order of the numbers in the geometric progression: x1 1 x2 and 1 x1 x2. In the rst case we get q = x1x2 = 12 , i.e. q = 1 is a perfect 2 cube. Let now x2 = x2 1 . We have ;p = x1 + x2 = x1 + x1 , i.e. x1 satis es the equation 2 x2 1 + x1 + p = 0. On the other hand x1 satis es also x1 + px1 + q = 0. From these two equations 3 we get (p ; 1)x1 + (q ; p) = 0. If p 6= 1, then x1 is rational and q = x1 x2 = x1 x2 1 = x1 is the cube of a rational number. Since q is integer, then it is a perfect cube. If p = 1, then q = p and the quadratic equation becomes x2 + x + 1 = 0. The last equation has no real root. Problem 2. A triangle ABC with a radius R of Figure 3. the circumcircle is given. Let R1 and R2 be the radii of the circles k1 and k2 , respectively, which pass through C and are tangent to the line AB in A and B , respectively. a) Prove that the numbers R1, R and R2 form a geometric progression. b) Find the angles of 4ABC , if the radius of the circle which is tangent to k1 , k2 and the line AB , is equal to R 4. Solution. a) Let AB = c, BC = a and CA = b. Denote by O1 and O2 the centers of the circles k1 and k2 (Figure 3). Then O1 is the intersection point of the perpendicular from A to AB and the segment bisector of AC . Since 6 MAO1 = AM AC and by the j90 ; 6 Aj (M is the midpoint of AC ), then R1 = AO1 = cos j90 = ; 6 Aj 2 sin 6 A sine theorem it follows that b: R1 = R a (1) Analogously, R2 = R a (2) b:

x + px + q = 0 are real numbers. Prove that if the numbers 1, x1 , x2 (in some order) form a geometric progression, then the number q is a perfect cube.
2

Problem 1. Let p and q be such integers that the roots x1 and x2 of the quadratic equation

From here R1R2 = R2 and the proposition is proved. b) Let O be the center of the circle which touches k1 , k2 and the line AB (Figure 4). Denote by T the tangent point of this circle , and by r its radius. From the rectangle trapezoid p with AB p p 2 2 ATOO1 it follows that AT = (R1 + r) ; (R1 ; r) = 2 rR1. Analogously, BT = 2 rR2. 2 p p Then c = AT + TB = 2 rR1 + 2 rR2 and from here we nd r = p c p 2 . Using (1) 4( R1 + R2 ) and (2), we get 2 (3) r = 4R(abc a + b)2 : 5

ab 1 R 2 2 Since (a + b)2 4 and c 4R , it follows from (3) that r 4 . The equation is reached when a = b and c = 2R. Now, it follows that 6 A = 6 B = 45 and 6 C = 90 . Problem 3. A positive integer n and a Figure 4. real number ' are given in a way that cos ' = 1 n . Find all positive integers k, for which the number cos k' is an integer. Solution. Case 1. n = 1. Then cos ' = 1 and ' = 2m (m = 0 1 : : :). For all k 2 N the number cos k' is an integer. 1 and Case 2. n = 2. Then cos ' = 2 ' = 3 + 2m (m = 0 1 : : :). It is clear that for all k 2 N, which are divisible by 3, the number cos k' is an integer. Case 3. n 3. We shall prove that for 1 , cos 2' = all k 2 N the number cos k' is not an integer. Let n be odd. We have cos ' = n 2 2 cos2 ' ; 1 = 2 ; n and (2 ; n2 n) = 1. We shall prove by induction that cos k' = a , where n2 nk

(a n) = 1. Assume that the assertion is true for all integers from 1 to k. We shall check it for k + 1. From cos(k + 1)' + cos(k ; 1)' = 2 cos k' cos ' 2 it follows that cos(k + 1)' = 2 a ; b = 2a ; bn , where cos k' = a and cos(k ; 1)' =

n nk nk;1 nk+1 nk b . We have (a n) = 1 and (b n) = 1 according to the inductive assumption. It is clear that nk;1 (2a ; bn2 n) = 1 and this ends the proof. The case when n is even is analogous. Now cos k' is expressed by a fraction which denominator is equal to 2pk with n = 2p, while the nominator has no common divisor with p. Answer: if n = 1 8k 2 N if n = 2 k = 3q , where q 2 N if n 3 there is no solution.
Grade 11

Problem 1. Find the values of the real parameter a, for which the function f (x) = x2 ; 2x ; jx ; 1 ; aj ; jx ; 2j + 4
has nonnegative values for all real x. Solution. Firstly let 1 + a 2, i.e. a 1. Then
8 > < f (x) = > :

x2 + 1 ; a x 1+a x2 ; 2x + 3 + a 1 + a x 2 x 2 ; 4 x + 7 + a x 2:
6

If 1 + a 2, i.e. a 1, we nd that 8 2 > x 2 < x +1;a f (x) = > x2 ; 2x + 5 ; a 2 x 1 + a : x2 ; 4x + 7 + a x 1 + a: Hence the smallest value of f (x) is reached in one of the points 0, 1, 2, 1 + a. We have f (0) = 2 ; ja + 1j, f (1) = 2 ; jaj, f (2) = 4 ; j1 ; aj, f (1 + a) = a2 ; ja ; 1j + 3. These four numbers must be nonnegative. We nd that a 2 ;2 1]. Then ja ; 1j = 1 ; a and f (1 + a) = a2 + a + 2 > 0 for all a. Let now a 2 = ;2 1]. If a < ;2, then x2 ; 2x + 3 + a < 0 when x = 1. Analogously, if a > 1, 2 then x + 1 ; a < 0 when x = 0. Finally, a 2 ;2 1]. Problem 2. The point O is circumcenter of the acute triangle ABC . The points P and Q lie on the sides AB and AC respectively. Prove that O lies on the line PQ if and only if

QC sin 2 sin 2 + sin 2 = PB PA QA where = 6 BAC , = 6 ABC , = 6 ACB . Solution. Firstly, let O be on PQ. Denote Figure 5. x = 6 AOP (Figure 5). By the sine theorem for the triangles AOP , BOP , AOQ and COQ we nd that PA sin x PB sin(2 ; x) QA sin(180 ; x) QC sin(2 ; 180 + x) PO = sin(90 ; ) PO = sin(90 ; ) QO = sin(90 ; ) QO : = sin(90 ; )

PB sin 2 + QC sin 2 = sin(2 ; x) sin 2 ; sin(2 + x) sin 2 = PA QA sin x sin x 1 ((sin 2 cos x ; sin x cos 2 ) sin2 ; (sin 2 cos x + sin x cos 2 ) sin 2 ) sin x = ; sin 2 cos 2 ; sin 2 cos 2 = ; sin 2( + ) = sin 2 : Conversely, let the given equality be satis ed. Denote by Q0 0the intersection point of OP 0C PB Q C Q and AC . It follows from the above that sin 2 = PA sin 2 + Q0 A sin 2 . Then Q0A = QC . QA Since the points Q0 and Q lie on the segment AC , then Q0 = Q. Problem 3. Find all functions f (x) with integer values and de ned in the set of the integers,
such that 3f (x) ; 2f (f (x)) = x 7

Then

for all integers x. Solution. The function f (x) = x satis es the condition of the problem. Let f (x) be a function which satis es the condition. Let g (x) = f (x) ; x. The condition can be written in the form 2f (f (x)) ; 2f (x) = f (x) ; x which is equivalent to g(x) = 2g (f (x)): From here we obtain

g(x) = 2g (f (x)) = 22 g (f (f (x))) = 23g (f (f (f (x)))) = 24g (f (f (f (f (x))))) = : : ::


Since the numbers g (f (f : : : f (x)) : : :) are integer, then g (x) is divisible by 2n for all integers x and all natural numbers n. This is possible only if g (x) = 0. Thus, f (x) = x is the only solution of the problem.

Winter mathematics competition|Burgas, 1997


Problem 8.1. Let F be the set of points with coordinates (x y) such that jjxj ; jyjj + jxj + jyj = 2.
(a) Draw F . (b) Find the number of points in F such that 2y = j2x ; 1j ; 3.

Solution: (a) If jxj jyj, then jjxj ; jyjj + jxj + jyj = jxj ; jyj + jxj+jyj = 2jxj = 2, thus jxj = 1 and therefore 1 jyj, so ;1 y 1. We conclude that the segments ;1 y 1 on the lines x = 1 and x = ;1 belong to F . If jxj jyj, then jjxj ; jyjj + jxj + jyj = ;jxj + jyj + jxj + jyj = 2jyj = 2, thus jyj = 1 and therefore 1 jxj, so ;1 x 1. We conclude that the segments ;1 x 1 on the lines y = 1 and y = ;1 also belong to F .
Thus we have determined that F consists of the sides of a square with vertices A(;1 ;1) B (1 ;1) C (1 1) D(;1 1). 1

(b) We nd the number of solutions of 2y = j2x ; 1j; 3 on each of the segments AB BC CD DA. The segment CD consists of all points (x y) such that ;1 x 1, y = 1. The equation 2 = j2x ; 1j ; 3 has no solution x when ;1 x 1. Therefore 2y = j2x ; 1j ; 3 has no solution on CD. The segment AB consists of all points (x y) such that ;1 x 1, y = ;1. The equation ;2 = j2x ; 1j ; 3 has two solutions: x = 0 and x = 1. Therefore 2y = j2x ; 1j ; 3 has two solutions on AB . As above we get that 2y = j2x ; 1j ; 3 has a unique solution on AD: (x y) = (;1 0) and a unique solution on BC : (x y) = (1 ;1). Note that the last one has already been obtained as a point on AB . Thus there are three solutions of 2y = j2x ; 1j ; 3 in F : (x y) = (;1 0) (0 ;1) (1 ;1).

Problem 8.2. Let H be the orthocentre of an acute triangle ABC .


Prove that the midpoints of AB and CH and the intersecting point of the internal bisectors of 6 CAH and 6 CBH lie on a line. 2

and the intersecting point of the internal bisectors of 6 CAH and 6 CBH ( g. 1). Let AA1 (A1 2 BC ) and BB1 (B1 2 AC ) be altitudes in 4ABC . We show rst that M , N and P lie on the axis of symmetry l of A1B1. From 6 CA1H = 6 CB1H = 90 we get PA1 = PB1 = CH 2 . Similarly, from 6 AA1B = 6 AB1B = 90 we get MA1 = MB1 = AB 2 . Therefore M 2 l and P 2 l. We prove now that 4NMA1 = 4NMB1. Now 6 BAA1 = 90 ; and 6 ABB1 = 90 ; , thus 6 NBB1 = 6 NBA1 = 1 6 A1 BB1 = 45 ; . By analogy 6 NAA1 = 45 ; , 2 2 2 so 6 ANB = 180 ; 6 NAB ; 6 NBA = 90 . Therefore 4ABN is a right triangle and MN = MA1 = MB1 = AB 2 .

Solution: Denote by M , P and N the midpoints of AB and CH

Further 6 NMA1 = 6 NMB ; 6 A1MB = 90 ; . By analogy NMB1 = 90 ; and therefore the considered triangles are identical. It follows now that NA1 = NB1, so N 2 l.
6

Problem 8.3. The n points A0 A1 : : : An;1 lie a circle in this


order and divide it into equal arcs. Find an ordering B0 B1 : : : Bn;1 3

of the same points such that the length of B0B1 : : :Bn;1 is maximal.

Solution: Let rst n = 2k +1. Clearly a chord AiAj is of maximal length if ji ; j j = k or k + 1. Cosider the following points:
A0 Ak A2k Ak;1 A2k;1 Ak;2 A2k;2 : : : A1 Ak+1:
Since each segment is of maximal length, it follows that the length of A0Ak A2kAk;1A2k;1Ak;2A2k;2 : : :A1Ak+1 is maximal. Let now n = 2k. A chord AiAj is of maximal length if ji ; j j = k. There are k such segments: A0Ak A1Ak+1 : : : Ak;1A2k;1. The second longest chord AiAj is obtained when ji ; j j = k ; 1 or k + 1. Cosider the following points:

A0 Ak A2k;1 Ak;1 A2k;2 Ak;2 : : : Ak+1 A1:


It is easy to see that there are k segments of maximal length and k ; 1 segments of the second greatest length. Trivially, this is the required ordering.

Problem 9.1. Let 6= be the roots of the equation x2 + px + q =


0. For any natural number n denote:

an =

n; n

; :

(a) Find p and q such that for any natural number n the following equality holds:

an+1 an+2 ; anan+3 = (;1)n :


4

(b) Prove that for these p and q it is true that an + an+1 = an+2 for any natural number n. (c) Prove that for any natural number n, an is integer and if 3 divides n, then an is even.

Solution: (a) Since and are the roots of x2 + px + q = 0, we know that + = ;p, = q and therefore: n+1 ; n+1 n+2 ; n+2 n; n n+3 ; n+3 (;1)n = ; ; ; ; ; 1 ;( + )( )n+1 + ( 3 + 3)( )n] = ( ; )2
1 (pqn+1 ; p(p2 ; 3q)qn) = p2 ; 4q n q = p2 ; 4q (;p3 + 4pq) = ;pqn:

Thus (1) pqn = (;1)n+1 : It follows from (1) for n = 1 and n = 2 that pq = 1 and pq2 = ;1 and so p = ;1, q = ;1. Direct veri cation shows that p = ;1 and q = ;1 satisfy (1) for any n. Also 6= . (b) Since and are the roots of x2 ; x ; 1 = 0, we know that 2 = + 1 and 2 = + 1. Therefore n; n n+1 ; n+1 n n a +a = + = (1 + ) ; (1 + )
n n+1

;
5

n:

= 1 and a2 = ; = + = 1, it (c) Since a1 = ; ; ; follows by induction from (2) an + an+1 = an+2 that an is integer for any n. From (2) we obtain: an+3 = an+2 + an+1 = an+1 + an + an+1 = 2an+1 + an: Observe that a3 = a1 + a2 = 2 is an even number. It is easy to see now (again by induction) that an is even for n = 3k.
2 2

which completes the proof of (b).

; n: 2 = ;

n+2 ; n+2

= an+2

be the intersecting point of AC and BD, and let Q be the intersecting point of AD and CE . Prove that if the triangles ABP , AEQ, CDP , CDQ and APQ have the same area, then ABCDE is a regular pentagon.

Problem 9.2. A pentagon ABCDE is inscribed in a circle. Let P

Solution: It su ces to prove that the sides of ABCDE are equal. Since 4ABP and 4CDP have equal areas, so do the triangles ACD

and ADB . Therefore the quadrilateral ABCD ( g. 1) is a trapezoid inscribed in a circle, so AB = CD. By analogy from SAEQ = _ SCDQ we get AC kDE and AE = CD. Now, 6 PCQ = 6 ACE = AE 2 = _ AB = 6 ADB = 6 PDQ. Therefore CDQP is inscribed in a circle. 2 6

On the other hand it follows from SCDP =_ SCDQ that CDQP is a _ trapezoid. Thus 6 CDP = 6 DCQ )BC =DE and therefore BC = DE . It remains to show that AB = BC . Consider the triangles ABP and APQ, whose areas are equal. They have AP as a common side and 6 AQP = 6 APQ = 6 ACD = 6 ABD = 6 ABP . It is easy to see now that 4ABP = 4APQ. Since 6 APB = 6 ADP + 6 PAQ > 6 PAQ,

6 APB = 6 APQ = 6 AQP = 6 ABP . Therefore it follows that _ _ _ _ _ _ AB + CD=AE + DE , so AB=DE =BC or AB = BC , which completes the proof.

Problem 9.3. Given a rectangular table of 100 rows and 1997

columns. The table is lled with zeroes and ones in such a way that there are at least 75 ones in any column. Prove that it is possible to remove 95 rows in such a way that there is at most one 7

column consisting of zeroes in the remaining table (5 rows and 1997 columns).

Solution: We show rst that there is a row with at least 1498

ones. Assume the contrary. Denote by ai the number of ones in the i-th row (i = 1 2 : : : 100) and by bi the number of ones in the i-th 1997 100 X X column (i = 1 2 : : : 1997). Now bi = ai. Note that the sum i=1 i=1 on the left-hand side is at least 1997 75, whereas the sum on the right-hand side is at most 1497 100, a contradiction. Without loss of generality assume that the rst row begins by 1498 ones. Consider the table formed by the last 499 columns. As above we prove that there is a row of this table (not necessarily distinct from the rst one) with at least 375 ones. Let that be the second row (if it is not the rst one) and let it begin with 375 ones in the new table. Now consider the table formed by the last 124 columns. Analogously, there exists a row having at least 93 ones. Let that be the third row (if it is not the rst or the second one) and let it begin with 94 ones in the new table. Consider next the table formed by the last 31 columns and note that there exists a row that originally has 24 ones. Finally consider the table formed by the last 7 rows and note that there exists a row that originally has 6 ones. We now have 5 rows (if there are fewer of them, we add arbitrary rows). Remove the remaining 95 rows of the original table. Since 1498+375+93+24+6 = 1996, there is at most one column consisting of zeroes.

tan 12 and tan 12 + x form (in some order) a geometric progression. to consider:

Problem 10.1. Find all real numbers x such that tan 12 ; x ,

Solution: Denote a = tan 12 and y = tan x. There are three cases


;y a+y = 1. tan 12 ; x tan 12 + x = tan2 12 . Now 1a+ ay 1 ; ay a2. Therefore a2 ; y2 = a2(1 ; a2y2), and so (a4 ; 1)y2 = 0. Since a 6= 1 we get that y = 0, so tan x = 0. Obviously all numbers x of the kind x = k , k 2 Zare solutions to the problem.
2. tan 12 tan 12 + x = tan2 12 ; x . We obtain !2 a + y a ; y a 1 ; ay = 1 + ay =) (a2 + 1)y ay2 + (a2 ; 1)y + 3a] = 0: The case of y = 0 is settled in 1. Let y1 and y2 be the roots of the equation ay2 + (a2 ; 1) y + 3a = 0. Since a = p tan 15 deg = p tan(45 deg ;30 deg) = 2 ; 3 we get y1 = y2 = 3, so tan x = p 3. Obviously all x of the kind x = 3 + k , k 2 Zare solutions of the problem. 3. tan 12 tan 12 ; x = tan2 12 + x . The substitution z = ;x transforms this case to the previous one. Therefore x = . ;3 + k , k 2 Z 9

The required numbers are x = k and x = 3 + k , k 2 Z .

Problem 10.2. Two points C and M are given in the plane. Let H be the orthocentre of 4ABC such that M is a midpoint of AB .
(a) Prove that CH CD = jAM 2 ; CM 2j where D 2 AB and CD ? AB: (b) Find the locus of points H when AB is of given length c.

Solution: (a) It is easy to see that the equality holds if 4ABC is a right triangle. If 4ABC is not a right triangle, then 4BDH BD = and 4ADC exist and 4BDH 4ADC . Therefore DH AD CD
and so (1)

CD DH = AD BD

There are three cases: Case 1: 4ABC is an acute triangle ( g. 1). Case 2: 4ABC is obtuse triangle and 6 C > 90 ( g. 2). Case 3: 4ABC is obtuse triangle and 6 A > 90 or 6 B > 90 ( g. 3). In cases 1 and 2 it follows from (1) that

CD DH = (AM DM )(AM DM ) = AM 2;DM 2 = AM 2;(CM 2;CD2)

10

so (2)

AM 2 ; CM 2 = CD(DH ; CD):
In case 1 we have CD = CH + DH and from (2) we get

AM 2 ; CM 2 = CD(DH ; CH ; DH ) = ;CH CD:


In case 2 we have CD = DH ; CH and from (2) we get

AM 2 ; CM 2 = CD(DH ; DH + CH ) = CH CD:
In case 3 it follows from (1) that CD DH = (DM AM )(DM AM ) = DM 2 ; AM 2 = CM 2 ; CD2 ; AM 2 and therefore CM 2 ; AM 2 = CD(DH + CD). Now DH = CH ; CD and so CM 2 ; AM 2 = CD(CH ; CD + CD) = CH CD. Summarising all three cases considered (and the case of a right triangle 4ABC ) we get

CH CD = jAM 2 ; CM 2j:
11

(b) It follows from the conditions of the problem that A and B are diametrically opposite in the circle (k) of centre M and radius c . When the diameter AB varies in (k) we obtain all triangles ABC 2 with the xed vertex C . There are three cases:

c = jCM j ( g. 4) Case 2: c > jCM j ( g. 5) Case 3: Case 1: 2 2 c < jCM j ( g. 6). 2 In case 1 C lies on (k) and triangles ABC are right triangles and their orthocentres coincide with C . In this case the locus consists of point C .

12

Consider now case 2. Let AB be a diameter in (k) perpendicular to CM . The orthocentre H of 4ABC lies on line CM . Moreover H is an external point for (k) because 4ABH is acute. The last follows from 6 AHB = 180 deg ;6 ACB and 6 ACB > 90 deg. Let A1B1 be a arbitrary diameter of (k) such that C 2 = A1B1: Denote by H1 the orthocentre of A1B1C and by D1 the foot of the altitude from the vertex C . Since 4A1B1C is obtuse, we get that H1 6= C . Consider 4MD1C and 4CHH1 and use a). We are in 2 case 2 of (a). For 4ABC we obtain that CH CM = c4 ; CM 2 and 2 CH = CD1 , for 4A1B1C that CH1 CD1 = c4 ; CM 2. Therefore CH CM 1 so 4MD1C 4CHH1. Since 4MD1C is a right triangle, we know that H1H ?CM and so H1 lies on a line l through H which is perpendicular to CM . Let H1 6= H be an arbitrary chosen point of l and let A1B1 be a diameter of (k) such that AB ? CH1. Since H is an external point for (k) and l?CM through H , it follows that all points of l are external for (k). In particular H1 6= C and therefore A1B1 exists. The orthocentre of 4A1B1C lies on CH1 13

and it follows from the above that it lies on l. Since H1 is the intersecting point of these two lines it is obvious that H1 is the orthocentre of 4A1B1C . Therefore the locus is a line l perpendicular to CM through a c2 ;jCM j point H on the ray opposite to CM~ and of distance 4jCM j from point C . Consider case 3. Let AB be a diameter of (k) perpendicular to CM . The orthocentre H of 4ABC lies on CM but now H is an internal point for (k). Consider an arbitrary diameter A1B1 of (k) such that H 2 A1B1. Let H1 be the orthocentre of 4A1B1C and CD1{ the altitude. Consider 4MD1C and 4CHH1. It is essential to show that 4CHH1 is uniquely determined (for 4MD1C it is obvious). It su ces to prove that H1 6= C . If the contrary is true, then 4A1B1C is a right triangle with its right angle at C and since A1B1 is a diameter, C must lie on (k), a contradiction. Since 4ABC 2 is acute, we apply case 1 of (a), so CH CM = CM 2 ; c4 . There are three cases for 4A1B1C : 1. if it is an acute triangle, we apply case 1 of (a) and therefore 2 CH1 CD1 = CM 2 ; c4 2. if it is an obtuse triangle, we apply case 3 of (a) and we get the same equality (note that 4A1B1C is not obtuse at C because C is an external point for (k)) 3. if it is a right triangle (say at B1) then D1 B1 H1 and 2 2 therefore CH1 CD1 = CB1 = CM 2 ; c4 . 14

The further considerations follow those from case 2. We conclude that the locus is a line perpendicular to CM and passing through c2 from C . H 2 CM~ of distance jCM j ; 4jCM j In particular it follows that PQ?CM , a well-known property.

that:

Problem 10.3. How many natural numbers a1a2 : : : a2n exist such
(a) none of the digits ai is zero (b) the sum a1a2 + a3a4 + : : : + a2n;1a2n is an even number?

is even if at least one of the digits a2i;1 and a2i is even. Therefore there are 5 4 + 4 5 + 4 4 = 56 choices for a2i;1 and a2i such that a2i;1a2i is even. Similarly, a2i;1a2i is odd when both a2i;1 and a2i are odd. There are 5 5 = 25 choices for a2i;1 and a2i such that a2i;1a2i is odd. The number of a1a2 : : :a2n such that items P inof the n i n ; i 2 i a2i;1a2i are odd is i 25 56 : Therefore An = i 2i 25 56n;2i . Let Bn = Pi 2in 252i+1 56n;2i;1 . Obviously +1

Solution: Denote the required number by An. The product a2i;1a2i

An + Bn =

n n X i=0

i n;i n n i 25 56 = (56 + 25) = 81

n 25i 56n;i = (56 ; 25)n = 31n : An ; Bn = i i=0 Thus 2An = 81n + 31n and An = (81n + 31n )=2.
(;1)i 15

n X

Problem 11.1. The sequence fxng1 n=1 is de ned as: x1 = 3 xn+1 = x2 n ; 3xn + 4 n = 1 2 3 : : : (a) Prove that fxng1 n=1 is monotone increasing and unbounded. 1 (b) Prove that the sequence fyn g1 n=1 de ned as yn = x ; 1 +
x2 ; 1 + its limit.
1
1

+x 1 , n = 1 2 3 : : :, is convergent and nd n;1

0 and so fxng1 n=1 is a monotone increasing function. We now prove by induction that xn n + 2. Obviously this equality holds for n = 1. Suppose it is true for n = k 1. Then xk+1 = xk (xk ; 3) + 4 (k + 2)(k ; 1) + 4 k + 3: Therefore xn n + 2 when n = 1 2 3 : : : and so the sequence is unbounded. (b) It follows from the recursive de nition of our sequence that xk+1 ; 2 = (xk ; 1)(xk ; 2). Hence 1 = 1 1 ; 1 = xk+1 ; 2 (xk ; 1)(xk ; 2) xk ; 2 xk ; 1 1 = 1 ; 1 : so xk ; 1 xk ; 2 xk+1 ; 2 By adding the above equalities for k = 1 2 : : : n we get 1 =1; 1 : yn = x 1 ; xn+1 ; 2 xn+1 ; 2 1;2 16

Solution: (a) xn+1 ; xn = (xn ; 2)2

Since 0 x 1 ; 2 n+1 lim y = 1. n n!1

1 it follows that lim 1 n!1 xn+1 ; 2 = 0, so n

Problem 11.2. Given 4ABC such that 6 ABC 60 deg and 6 BAC 60 deg. Let BL (L 2 AC ) be the internal bisector of 6 ABC and AH H 2 BC be the altitude from A. Find 6 AHL if
6

BLC = 36 AHL.

Solution: Denote = 6 AHL, = 6 BAC , = 6 ABC and = sin = 6 ACB . From the Sine Law for 4AHL and 4CHL we get AL sin 6 ALH and sin(90 deg ; ) = sin 6 CLH . Since CL = BC =

AH CL CH AL BA sin and AH = tan (from 6 ACB < 90 deg it follows that H lies sin CH on a ray CB~), we get cos = sin : (1) sin cos

It follows from the conditions of the problem that 3 = + 2 , so 6 ; = + . Thus cos = ; cos(6 ; ) and (1) is equivalent to cos cos(6 ; ) + sin sin = 0 () cos(7 ; ) + cos(5 ; ) + cos( ; ) ; cos( + ) = 0 () ; sin 4 sin(3 ; ) + cos(3 ; ) cos 2 = 0 () cos 2 (2 sin 2 sin(3 ; ) ; cos(3 ; )) = 0. There are two cases to consider: 1. cos 2 = 0. From 0 < < 90 deg it follows that = 45 deg. (Working backwards we get that 135 deg = 6 BLC = 36 AHL 17

for any 4ABC such that + 2 = 135 deg). 2. cos 2 6= 0. It follows from 3 ; = 2 that (2) But 60 deg 2 sin 2 = cotg 2 : < 180 deg and so

(3) cotg 2 < cotg30 deg = 3: On the other hand, also from the conditions of the problem we get 180 deg > 3 = + 2 90 deg, so 30 deg < 60 deg. Therefore (4) 2 sin 2 2 sin 60 deg = 3: The inequalities (3) and (4) show that (2) holds only if = 30 deg. In this case = = = 60 deg. (It is obviously true that 90 deg = 6 BLC = 36 AHL for any equilateral 4ABC ).

Problem 11.3. Find all integer numbers m n 2 such that


is integer. 1 + m3n + m2:3n n

18

Solution: Let m and n satisfy the conditions of the problem. Since

n is an odd integer number, we get (m n) = 1 and n > 2. When n = 3, all m 4 such that m n 1(mod 3) are solutions, because 3 2 3n if m ;1(mod 3), then 1 + m + m 1 ; 1 + 1 1(mod 3). 3n Let now n > 3. It follows that m 6 1(mod n), because otherwise 1 + m3n + m2 3n n+1 3(mod n), i. e., n=3. On the other hand 1 + 3 ; 1 and therefore m3n+1 1(mod n). Let k m3n + m2 3n = m n 3 m ;1 be the least natural number such that mk 1(mod n). Further, k=3n+1 and k ; = 3n , so k = 3n+1 . Let '(n) be Euler's function. From (m n) = 1 it follows that m'(n) 1(mod n), so k '(n). Therefore 3n+1 '(n) n ; 1, which is impossible. The required numbers are: n = 3 and all m 4 such that m 1(mod 3).

19

Winter mathematics competition|Pleven, 6{8 February 1998


Dedicated to the One Hundredth Anniversary of the UBM

Problem 8.1. Let three numbers a, b and c be chosen so that


a = b = c. b c a
1

y = 6x ; 15y and the b.) Find the sum x + y if 3x = y 2x ; 5y x expression ;4x2 + 36y ; 8 has its maximum value.

a.) Prove that a = b = c.

Solution:
a.) It is obvious that a 6= 0, b 6= 0, c 6= 0. The rst equality gives b2 = ac, whence by multiplying both sides by b we get b3 = abc. Similarly a3 = abc and c3 = abc. Hence a3 = b3 = c3 and therefore a = b = c. b.) By multiplying both the numerator and the denominator of the second fraction by 3 and using the result of a.) we obtain x = 3y. Thus ;4x2 + 36y ; 8 = ;9(4y2 ; 4y + 1) + 1 = ;9(2y ; 1)2 + 1, and its maximum value is 1 when 2y ; 1 = 0. 1 3 Therefore y = 2 and x = 2 , i. e., x + y = 2.

Problem 8.2. In the acute triangle 4ABC with 6 BAC = 45 deg, BE (E 2 AC ) and CF (F 2 AB ) are altitudes. Let H , M and

K be the orthocentre of ABC and the midpoints of BC and AH , respectively.


a.) Prove that the quadrangle MEKF is a square. b.) Prove that the diagonals of the quadrangle MEKF intersect at the midpoint of OH , where O is the circumcentre of 4ABC . c.) Find the length of EF when the circumradius of 4ABC is 1. 2

Solution:
a.) The segments EM and FM are medians to the hypotenuses 1 BC . of 4BCE and 4BCF and therefore EM = FM = 2 Similarly, for 4AHE and 4AHF we get EK = FK = 1 AH . 2 6 Since BAC = 45 deg, we nd that 4AEB and 4CEH are isosceles. Hence AE = BE and EC = EH , i. e., 4AHE = 4BCE . Therefore EK = EM . Thus MEKF is a rhombus. Furthermore, 6 MEK = 6 MEB + 6 HEK = 6 CBE + 6 HEK = 6 EAH + 6 HEK = 6 EAH + 6 AHE = 90 deg i. e., the quadrangle is a square.

b.) It follows from a.) that the intersecting point S of the diagonals of the quadrangle MEKF is the midpoint of both diagonals. Since 4AEB is isosceles, E lies on the axis of symmetry of the segment AB and therefore EO?AB , i. e., EOkHF . Similarly FOkEH . Thus the quadrangle EOFH is a parallelogram. From the above we conclude that S is the midpoint of OH . 3

c.) a.) implies that in the acute triangle 4ABC with orthocentre H and 6 BAC = 45 deg it is true that AH = BC . 4AFE is of the same type and therefore EF = AO = 1. (It follows from b.) that O is orthocentre of this triangle.)

Problem 8.3. Let 1998 points be chosen on the plane so that out of

any 17 it is possible to choose 11 that lie inside a circle of diameter 1. Find the smallest number of circles of diameter 2 su cient to cover all 1998 points. (We say that a circle covers a certain number of points if all points lie inside the circle or on its outline.) Choose 1998 points as follows: the 6 vertices of the hexagon and 1992 points inside a circle of diameter 1 centred at the centre of the hexagon. It is clear that the above 1998 points satisfy the condition of the problem. Moreover any circle of radius 1 covers at most one of the vertices of the hexagon. Therefore the required number is no less than 7 (in our case: 6 circles for each vertex and a single circle for the remaining points). Now we shall prove that the required number is no greater than 7. Arbitrarily choose 8 points and add other 9, for a total of 17. It is clear that there is a circle of diameter 1 covering at least 11 of these 17 points. At most 6 points lie outside the circle and therefore at least 2 of the initially chosen 8 points lie inside the circle. The distance between these two points is no greater than 1. We have proved that among any 8 points there always exist 2 such that the distance between them is no greater than 1. 4

Solution: Consider a regular hexagon with a side of length 3.

Now choose a circle of radius 1 centred in one of the points. If the remaining points lie inside the circle, the required number is 1 and thus no greater than 7. If this is not the case, take another point outside the rst circle. If all points lie in the two circles, then the required number is 2 and thus no greater than 7. Continuing in this way we either obtain no more than 7 circles covering all points or have 7 circles and a point that lies outside all circles. Consider this point and the centres of the chosen circles. There exist 2 points among these 8 such that the distance between them is no greater than 1. But this is impossible because of the way we chose our points. Together the two parts of the proof demonstrate that the required number is 7.

Problem 9.1. Find all quadratic functions f (x) = x2 ; ax + b with integer coe cients such that there exist distinct integer numbers m, n, p in the interval 1 9] for which jf (m)j = jf (n)j = jf (p)j = 7. Solution: Let f (x) be a function satisfying the conditions of the
problem. Such a function cannot take one and the same value for three di erent arguments (otherwise we would have a quadratic equation having three distinct roots). Therefore two of the numbers f (m), f (n) and f (p) equal 7 (or ;7) and the third one equals ;7 (or 7). Case 1. f (m) = f (n) = 7, f (p) = ;7. Without loss of generality we may assume that m > n. Since m, n are roots of x2 ; ax + b ; 7 = 0, we obtain that a = m + n, b = mn + 7. 5

Subtracting the two equalities m2 ; am + b = 7 p2 ; ap + b = ;7 we nd 14 = m2 ; p2 ; a(m ; p) = (m ; p)(m + p ; a) = (m ; p)(p ; n): Thus the numbers m ; p and p ; n are either both positive or both negative and since m > n, they are positive. Moreover they are integer and therefore are equal to 1 and 14 or to 2 and 7. But since m n p 2 1 9] it follows that neither m ; p nor p ; n is 14. There are two cases to consider: m ; p = 2, p ; n = 7 and m ; p = 7, p ; n = 2, i. e., either m = p + 2, n = p ; 7 or m = p + 7, n = p ; 2. It is obvious that in both cases at least one of m n p lies outside the interval 1 9]. Case 2. f (m) = f (n) = ;7, f (p) = 7. As in Case 1 we get a = m + n, b = mn ; 7 and (m ; p)(p ; n) = ;14. Using similar arguments we obtain that either m ; p = 2, p ; n = ;7 or m ; p = ;2, p ; n = 7. (Without loss of generality we suppose that jm ; pj < jp ; nj.) Therefore the two options are m = p + 2, n = p + 7 and m = p ; 2, n = p ; 7. Simple calculations show that all triples (m n p) satisfying the conditions are (3 8 1), (4 9 2), (6 1 8) and (7 2 9). So the functions are f (x) = x2 ; 11x + 17, f (x) = x2 ; 13x + 29, f (x) = x2 ; 7x ; 1 and f (x) = x2 ; 9x + 7.

Problem 9.2. Three points A1, B1 and C1 lie on the sides BC , CA and AB of 4ABC so that AB1 = C1B1 and BA1 = C1A1. Let D be the re exion of C1 in A1B1 (D 6= C ). Prove that the line CD is perpendicular to the straight line through the circumcentres of 4ABC and 4A1B1C .
6

Solution: It su ces to prove that D is the second intersecting


point of the two circumcircles. We know that 6 A1DB1 = 6 A1C1 B1 = 180 deg ;6 BC1A1 ; 6 AC1B1 = 180 deg ;6 C1BA1 ; 6 C1AB1 = 6 A1CB1:

On the other hand, A1D = A1C1 = A1B and B1D = B1C1 = B1A, which shows that A1 and B1 are the circumcentres of 4BC1D 6 and 4AC1D. Therefore 6 ADB = 6 ADC1 + 6 BDC1 = 1 2 AB1C1 + 16 6 6 6 2 BA1C1 = 90 deg ; C1AB1 + 90 deg ; C1BA1 = ACB . Since C and D lie in one and the same semiplane in regard to both A1B1 and AB , it follows from 6 A1DB1 = 6 A1CB1 and 6 ADB = 6 ACB that D is the second intersecting point of the circumcircles of 4A1B1C and 4ABC . This completes the proof.

Problem 9.3. All natural numbers from 1 to 1998 inclusive are


written 9 times (so that there are 9 ones, 9 twos and so on) in the 7

cells of a rectangular table with 9 rows and 1998 collumns, so that the di erence between any two elements lying in one and the same column is no greater than 3. Find the maximum possible value of the smallest sum amongst all 1998 sums of the elements lying in one and the same column.

Solution: Consider the placement of the ones in the columns. If

they are all in a single column, then the minimum sum of elements lying in one column is 9. Let all ones lie in exactly 2 columns. Therefore there are at least 5 ones in a single column and thus the minimal sum is no greater that 5 1 + 4 4 = 21. If all ones are placed in exactly 3 columns, then the sum of all numbers in these three columns is at most 9 1 + 9 4 + 9 3 = 72. Hence the minimal sum is at most 72 : 3 = 24. If all ones are placed in exactly 4 columns, then the sum of all numbers in these columns is at most 9 1 + 9 4 + 9 3 + 9 2 = 90 and therefore the minimal sum is at most 90 : 4, i. e., 22. It is impossible to have ones in more than 4 columns, because in that case the total number of 2s, 3s and 4s does not su ce to ll the remaining cells. Therefore the required sum is at most 24. 1 1 1 2 2 2 7 8 . . . 1998 1 1 1 2 2 2 7 8 . . . 1998 1 1 1 2 2 2 7 8 . . . 1998 The following example 3 3 3 5 5 5 7 8 . . . 1998 shows that this sum can 3 3 3 5 5 5 7 8 . . . 1998 be 24, consequently the 3 3 3 5 5 5 7 8 . . . 1998 answer is 24: 4 4 4 6 6 6 7 8 . . . 1998 4 4 4 6 6 6 7 8 . . . 1998 4 4 4 6 6 6 7 8 . . . 1998 8

x2 and x3 such that sin 23 x1 , sin 23 x2 and sin 23 x3 form (in some order) an aritmetic progression.

Problem 10.1. Find all values of the real parameter a for which the equation x3 ; 3x2 +(a2 +2)x ; a2 = 0 has three distinct roots x1,

Solution: Since x3 ; 3x2 + (a2 + 2)x ; a2 = (x ; 1)(x2 ; 2x + a2), in order for there to be three distinct real roots it that pis necessary 2 2 2 D = 1 ; a > 0. Therefore a < 1 and thus p 1 1;a > p0. The roots of our equation are x1 = 1, x2 = 1+ 1 ; a2, x3 = 1 ; 1 ; a2.
It follows now that x2 + x3 = 2 and 2 x2 > 1 and 1 > x3 0. There are two cases to consider: 1. The second term of the progression is sin 23 x1 . Then sin 23 x2 + sin 23 x3 x3 cos 2 x2 ; x3 2 sin 23 x2 + 2 3 2 cos 3 (x2 ; x3) = 2 sin 23 = 2 sin 23 = 1:

p But 3 jx2 ; x3j = 23 1 ; a2 23 , and hence 3 (x2 ; x3) 2 ; 23 23 ]. Therefore cos 3 (x2 ; x3) = 1 when x2 = x3, which is impossible, since the roots are distinct. 2. The rst or the third term of the progression is sin 23 x1 . Then sin 23 + sin 23 xi = 2 sin 23 (2 ; xi) 9

for i = 2 or 3. Hence sin 23 + sin 23 xi = 2 sin 43 cos 23 xi ; 2 cos 43 sin 23 xi : 1 . From the reAfter simple calculations we get cos 23 xi = ; 2 strictions for x2 and x3 we obtain xi = 1 or xi = 2. In the rst case a2 = 1, which is impossible, and in the second case x2 = 2 x3 = 0 and a2 = 0. Thus a has a unique value and it is a = 0.

Problem 10.2. A point C lies on the periphery of a circle. Two points


A and B are chosen anticlockwise away from C such that if 6 CAB = and 6 CBA = , the following equality holds:
2 cos 2 + = sin 2 ; : Prove that the bisectors of 6 CAB pass through a xed point.

Solution: It is easy to see that = 90 deg, = 45 deg and tan 2 =

1 , = 90 deg satisfy the condition for and . Therefore the 2 required point is the midpoint of OE {F ( g. 1). From the premises of the problem we obtain 2 cos 2 cos ; 2 sin 2 sin = sin 2 cos ; cos 2 sin 10

and after dividing by cos 2 cos (cos 2 6= 0 (why?), and if cos = 0, i. e., = 90 , we have one of the two cases already considered) we obtain tan 2 (1 + 2 tan ) = 2 + tan It follows in particular that if is xed, then is uniquely determined. Suppose that tan 2 > 2. Thus 2 > 45 and therefore > 90 . If tan < 0, we get > 90 , which is impossible, since + < 180 . If tan > 0, we get 2 + tan > (1 + 2 tan ) 2 i. e., tan < 0, which is a contradiction. Therefore tan 2 2 and B lies on CED where 6 GCF = 6 FCD and tan 6 GCF = 2 ( g. 1). Fix the point B such that < 90 . Let T be the midpoint of the arc CB and let A1 be the intersecting point of TF and the circle. We shall show that A1 A. We obtain 6 OA1F = 2 ; (90 ; ) = + 2 ; 90 and 6 A1FO = 45 ; 2 + ; 45 = ; 2 . It follows from 11

the Sine Theorem for 4A1FO that

equivalent to 2 cos 2 + The case of > 90 can be dealt with by analogy. The condition for B to lie on CED shows that A1 lies between C and B ( g. 3).

;2 = 2, which is sin + 2 ; 90 = sin 2 ; . Therefore A1 A.


sin

Problem 10.3. Let n be a natural number. Find the number of sequences a1a2 : : : a2n, where ai = +1 or ai = ;1 for i = 1 2 : : : 2n,
such that
2l X

for all k and l for which 1 k l n. 1 k n satis P2l es the condition of the problem,nbecause any sum of the form i=2k;1 ai equals zero. There are 2 such sequences. Let us determine the number of sequences such that there exists a k for which a2k;1 + a2k 6= 0. Let k1 k2 : : : ks be all k with the above property. It is easily seen that if a2k ;1 + a2k = 2 (;2), then a2k +1;1 + a2k +1 = ;2 (2). Therefore all sums a2k ;1 + a2k (and so also a2k ;1a2k ) are uniquely determined by a2k1;1 + a2k1 (there are two possibilities for a2k1;1 a2k1 ). There are two possibilities for any of the remaining n ; s pairs (for which a2t;1 + a2t = 0). Therefore there are ! ! ! ! ! n n n n ;1 n n ;2 n n ;k n 2 +2 2 1 +2 2 2 + +2 2 k +: : :+2 2 n ; 1 +2 n :
i i i i i i i i

i=2k;1

ai

Solution: It is clear that a sequence having a2k;1 + a2k = 0 for

12

sequences with the required property. By adding and subtracting 2n to and from the above expression we get: ! ! ! ! !! n n n n n ;1 n n ;2 n 2: 2 0 +2 1 +2 2 + +2 n ; 1 + n ;2n =2 3n ;2n Thus there are 2 3n ; 2n sequences.

px + px ; 4 ; Problem 11.1. Consider the function f ( x ) = p p x ; 1 ; x ; 3, x 4.


b.) Prove that f (x) is an increasing function. c.) Find the number of real roots of the equation f (x) = a where a is a real parameter. a.) Find xlim f (x). !1

q x;3
x

Solution: a.) By grouping the rst and third radicals and the

second and fourth radicals and rationalising we get that when x > 4, 1 1p p f (x) = px+p f (x) = 0. x;1 ; x;4+ x;3 . Therefore xlim !1 b.) When x > 4, 1 ; p1 ; p1 + p1 f 0(x) = 2p x 2 x;1 2 x;3 2 x;4 1 p p = p p + 2 x;3 x;4( x;3 + x;4) ; 2pxpx;1(1 px+px;1) > 0: 13

Therefore f (x) is an increasing function if x 4. c.) It follows from a.) and b.) that f (x) < 0 when x 4, i. e., the equation could have a q solution only if a < 0. Let a < 0. The qx ;3 3 is decreasing and continuous and function g(x) = a x = a 1 ; x g(x) = a < 0. Since f (x) is an increasing and continuous funcxlim !1 tion and xlim f (x) = 0, in accordance with the Bolzano{Weierstra !1 Theorem the equation f (x) = g(x) has a solution p (and it is a unique one) exactly when f (4) g(4), i. e., if 2(1 ; 3) a < 0.

Problem 11.2. The convex quadrangle ABCD is inscribed in a

circle with centre O. Let E be the intersecting point of AC and BD. Prove that if the midpoints of AD, BC and OE lie on a straight line, then AB = CD or 6 AEB = 90 deg.

Solution: It su ces to prove that if 6 AEB 6= 90 deg, then AB = 6 90 deg. If O E , then ABCD is a rectangle CD. Let 6 AEB = and therefore AB = CD. Suppose O 6 E . Let M , N , P , Q be the

midpoints of AD, BC , AC , BD, respectively, and R be the intersecting point of the straight lines through P and Q perpendicular to BD and AC , respectively. It is clear that MPNQ and OPRQ are parallelograms. Therefore the midpoints of MN and OR coincide with the midpoint of PQ, and since the midpoint of OE lie on MN , we get that RE kMN . On the other hand R is the orthocentre of 4PQE and therefore RE ?PQ. Hence MN ?PQ, i. e., the parallelogram MPNQ is a rhombus. It is easy to see now that AB = 2PN = 2NQ = CD, which solves the problem.

14

point of the axes of symmetry of AC and BD, then the assertion of the problem and its opposite are true for a quadrangle that is not inscribed in a circle. This could be demonstrated by using complex numbers or trigonometry.

Note: The above solution shows that if O is the intersecting

Problem 11.3. Let famg1 m=1 be a sequence of integer numbers


such that their decimal representations consist of even digits (a1 = 2, a2 = 4, a3 = 6,. . . ). Find all integer numbers m such that am = 12m.

15

Solution: Let m be an integer number such that m = b0 + b1 5 +

+ bn 5n . Denote f (m) = 2b0 + 2b1 10 + + 2bn 10n . It is clear that ff (m) j m 2 Ng is the set of integer numbers with only even digits in their decimal representation. Since f (m1) < f (m2) () m1 < m2, it follows that am = f (m) for any m. Therefore it su ces to nd all m such that 12(b0 + b1 5 + + bn 5n ) = 2b0 + 2b1 10 + + bn 5n ) = b0 + b1 10 + + 2bn 10n + bn 10n :

i. e., (1) 6(b0 + b1 5 + Since b0+b1 5+ +bn 5n 5n+1 ;1 and b0+b1 10+ +bn 10n 10n , it follows from (1) that 6(5n+1 ; 1) 10n , i. e., 6 5n+1 > 10n . Thus 2n < 30 and therefore n 4. If n = 4, we get from (1) that b0 + 4b1 + 10b2 = 50b3 + 1250b4 1250, which is imposible. In the same way it is easy to show that n 3, i. e., n = 3. In this case b0 +4b1 +10b2 = 50b3. Obviously b3 = 1 and b0 = b1, because b0 ;b1 is divisible by 5. As a result we have the equation b0 +2b2 = 10, and its solutions are b0 = 2 b2 = 4 and b0 = 4 b2 = 3. Therefore all integer numbers m with the required property are m = 2+2 5+4 52 +53 = 237 and m = 4 + 4 5 + 3 52 + 53 = 224.

16

Winter mathematics competition|Varna, 1999


Problem 8.1. Find all natural numbers x and y such that:
1;1=1 a) x y 3 1+1=1+ 1. b) x y 3 xy

y = 6. Hence there is a unique solution x = 2 y = 6. 1=1+1; b) Let x y be a solution of the problem. Now 3 x y 1 2 ; 1 = 2y ; 1 < 2y = 2 , giving x < 6. When x = 1, x = 2 xy y xy xy xy x or x = 3, no solution exists. When x = 4, it follows that y = 9, and x = 5 implies y = 6. If y x, we apply the same reasoning. The
1

Solution: a) The equation is equivalent to 3y ; 3x = xy, so y = 3y + 9 ; 9 = 3 ; 9 . Therefore y + 3 = 9 and thus x = y3 +3 y+3 y+3

problem has four solutions:

x = 4 y = 9 x = 5 y = 6 x = 6 y = 5 x = 9 y = 4:

the following property: A segment connecting a vertex of a triangle with a point on the opposite side divides the triangle into two triangles such that the ratio of their areas equals the ratio of the parts into which the point divides the side. Let G 2 MN and G1 , G2, G3 be the projections of G on BC , AC and AB , respectively. Further, denote the projections of N on BC and AB by N1 and N3 and those of M on AC and AB by M2 and M3. We shall prove that GG3 = GG1 + GG2 and from the above property it will follow straightforwardly that the altitude from C is equal to the sum of the remaining two altitudes. We obtain GG1 = CM GG1 = SGMC = GM : NN1 CM NN1 SNMC NM GG2 = GN , implying that GG1 + GG2 = 1, so By analogy MM NM NN1 MM2 2 (1) GG1 MM2 + GG2 NN1 = MM2 NN1: 2

Problem 8.2. Given an acute 4ABC with centroid G and bisectors AM (M 2 BC ) BN (N 2 AC ) CK (K 2 AB ). Prove that one of the altitudes of 4ABC equals the sum of the remaining two if and only if G lies on one of the sides of 4MNK . Solution. We shall repeatedly use

Let M4 and G4 be the projections of M and G on NN3 . It easily follows now that NN3 ; GG3 = GN = GG2 : NN3 ; MM3 NM MM2 Further, using that NN3 = NN1 and MM3 = MM2, we obtain GG3 MM2 = GG1 MM2 + GG2 MM2 and therefore GG3 = GG1 + GG2. Conversely, let the altitude through C be the sum of the remaining two. Now GG3 = GG1 + GG2. If G? = GG3 \ MN , then it follows straightforwardly that the sum of the distances from G? to AC and BC equals to G?G3. It is easy to check now that G? = G. of m such that k = 2m;2 is integer and A = 1999k + 6 is a sum of the squares of n integers (not necessarily distinct and di erent from zero). 1) n = 1 and k = m is integer only if m = 4p and m = 4p + 2. 2 If m = 4p, then A = (2 1000 ; 1)2p + 1 and it follows by induction that A is of the form A = 4a + 7, so A is congruent to 3 modulo 4. We conclude that A is not a perfect square. If m = 4p + 2, then A = (25 80 ; 1)2p+1 + 6 and it follows by induction that A is of the form A = 25a + 5. Therefore A is not a perfect square, because 5 divides A, but 25 does not. 2) n = 2 and k = m is integer for any m. Now A = 1999m + 6. When m = 0, we get A = 7, which is not a sum of two squares. 3

Problem 8.3. Let n be a natural number. Find all integer values

Solution: It is su cient to consider only nonnegative values of m.

When m = 1, we obtain A = 2005 = 412 + 182 . If m 2, then A = (2 999 + 1)m + 1 and as above A is congruent to 3 modulo 4. On the other hand the sum of two perfect squares is congruent to 0, 1 or 2 modulo 4 and so no solution exists in this case. 3) n = 2 and k = m is integer for any m. Now A = (8 250 ; 2 m 1) + 6, which can be written in the form A = 8a + 7. Therefore A is congruent to 7 modulo 8, whereas a sum of three perfect squares is congruent to 0, 1, 2, 3, 4, 5 or 6 modulo 8. Thus no solution exists in this case. 4) n 4 and k is integer for any m. Now A = (1999m2n;3 )2 + n ; 3 22 + 12 + 12 and if a1 = 1999m2 a2 = 2 a3 = a4 = 1 a5 = a6 = 2 = an = 0, then A = a2 + a2 1 + a2 + n. Answer: if n = 1, no solution exists if n = 2, there is an unique solution m = 1 if n = 3, no solution exists if n 4, any m 0 is a solution.

Problem 9.1. Let p be a real parameter such that the equation x2 ; 3px ; p = 0 has real and distinct roots x1 and x2.
a) Prove that 3px1 + x2 2 ; p > 0. b) Find the least possible value of
2 2 + 3p 1 A = 3px +px2 + 3p + 3px2 +px : 2 1 2 When does equality obtain?

Solution: a) It follows from the equation that x2 2 = 3px2 + p and 2 2 so 3px1 + x2 ; p = 3p(x1 + x2) = 9p > 0. The inequality is strict
because otherwise x1 = x2 = 0. 2 b) As in a), we obtain 3px1 + x2 2 + 3p = 3px2 + x1 + 3p = 9p2 + 4p > 0 (the last inequality follows from the conditions of the problem x1 and x2 to be distinct and real, giving p 6= 0). Therefore
2 2 4p A = 9p2p+ 4p + 9p p+ 2

(from the Arithmetic{Geometric Mean Inequality) and equality obtains when 9p2 + 4p = p2, i. e., when p = ;1=2.

Problem 9.2. Given an acute 4ABC such that AC > BC , let M

be the midpoint of AB and let CD, AP and BQ be the altitudes. Denote the circumcircle of 4PQC by k1 and the circumcircle of 4DRP by k2, where R is the point of intersection of AB and PQ. Prove that: a) MP is tangent to both k1 and k2. b) RH ? CM , where H is the orthocentre of 4ABC .

Solution: a) Note that H 2 k1. Since 6 APM = 6 PAM = 90 ; 6 ABC = 6 BCD, we obtain that MP is a tangent to k1. On the other hand 6 MPD = 6 MPB ; 6 DPB = 6 MPB ; 6 DPB = 6 MBP ;6 BPR (4ABC 4DBP ), so 6 ARP = 6 MBP ;6 BPR = 6 MBP ; 6 QPC = 6 MBP ; 6 BAC (4ABC 4PQC ). Therefore
6

MPD = 6 MBP and thus MP is a tangent to k2.


5

b) Let L = CM \ k1. It follows from a) that ML MC = MP 2 = MD MR. We conclude that L lies on the circumcicrle of 4DRC and therefore RL ? CM . Further HL ? CM , since HC is a diameter of k1. Hence RH ? CM .

Problem 9.3. A square table lled with nonnegative (not neces-

sarily distinct) integer numbers is said to be a magic square with sum m if the sum of the numbers in each row and each column equals m. Prove that the number of magic squares 3 3 of sum m such that the minimal element among ! the elements on the main diagonal lies m + 4 in the centre is 4 . and the element in the cell (1 2) (see g. 1) one can determine all elements in the table. Indeed, there is a unique choice for all remaining cells (see g. 2). Therefore it su ces to see when all elements are nonnegative and b is the minimal element among the elements on the main diagonal. a d a d m;a;d b m+c;a;b;d b a+d;c c b+d;c m;b;d c Fig. 1 Fig. 2 It is clear from g. 2 that the following inequalities hold: (1) a + d m (2) b + d m 6

Solution: It is evident that knowing the elements of main diagonal

(3) c a + d (4) c b + d (5) a + b + d ; c m. The conditions of the problem imply b a and b c. It is clear now that (3) follows from (4) and (2) and (5) follow from (1). Therefore we can consider only (1) and (4). Consider the following chain of inequalities

b 2b + d ; c a + b + d ; c a + d m
(the rst follows from (4), the second from b a, the third from b c, and the fourth is equivalent to (1)). It is easy to see that knowing the quadruple (b 2b+d;c a+b+d;c a+d) we can uniquely determine a b c and d and so nd a magic square. Therefore the +4 required number equals the number of quadruples, which is m4 .

Problem 10.1. Find all values of the real positive parameter a


such that the inequality acos 2x + a2 sin2 x 2 holds for any real x.
2 2 2

Solution: We know that acos 2x + a2 sin x = a1;2 sin x + a2 sin x = a 2 sin x 2


2 sin2 x

+ a 2 . Substitute t = a2 sin2 x. Since 0 sin x 1, a we obtain that t is between 1 and a2. Our inequality now becomes a + t 2 () t2 ; 2t + a 0. Since it holds true for any x t (i. e., for any t between 1 and a2), it follows that the roots of f (t) = t2 ; 2t + a = 0 lie outside the open interval determined by 1 and a2. Therefore f (1) 0 and f (a2) 0. The rst inequality gives a 1 and the second one implies a4 ; 2a2 + a 0 () a3 ; 2a + 1 0 () (a ; 1)(a2 + a ; 1) 0. Since a 1, we obtain a2 + a ; 1 0. 7

p p# ; 1 ; 5 ; 1 + 5 . So we The solution of this inequality is a 2 2 2 p# " obtain a 2 1 ;1 + 5 .


"

Problem 10.2. Let CH and CM be an altitude and a median in a non-obtuse 4ABC . Let the bisector of angle BAC meet CH and
a) 4ABC is a right triangle b) BP = 2CH .

CM at points P and Q, respectively. If 6 ABP = 6 PBQ = 6 QBC , prove that:

BQ. Denote AB = c, BC = a, CA = b. It is easy to see that P lies between A and Q (otherwise 6 ABP > 6 PBQ). It follows from Ceva's Theorem for point P that:
(1)

Solution: a) Let R = BC \ AP , T = AC \ BP and S = AC \

AH BR CT = 1 () b cos c SBTC = 1 HB RC TA a cos b SABT 2 2 BT a sin sin b cos c cos 3 () a cos b BT c sin = 1 () cos sin 3 = 1 3 3 beta 2 sin 3 cos 3 = 1 () cos = 1 : () cos cos cos sin 3 2 cos 3
8

It follows from Ceva's Theorem for point Q that: (2) AM BR CS = 1 () c SBSC = 1 () c BS a sin 3 = 1 MB RC SA b SABS b BS c sin 23

a sin 3 = 1 () a sin 3 = 1 () a = 2 cos : () c b c sin 23 b 3 2b sin 3 cos 3 b . From the Sine Law we obtain Now (1) and (2) imply cos = cos a sin = b , so cos = sin () sin 2 = sin 2 . If = , then sin a cos sin the triangle is isosceles and therefore P Q, implying that 6 ABP = 6 PBQ = 6 QBC = 0 , which is impossible. Thus + = 90 and therefore 6 ACB = 90 . b) It follows from 4BCS that cos = a . Combining the 3 BS above with (2) gives 2b = BS . Note that 4ABC 4BHC , which BP = BS = 2. Therefore BP = 2CH . implies CH AC

Problem 10.3. Let A be a set of natural numbers with no zeroes in their decimal representation. It is known that if a = a1a2 : : :ak 2 A,
then b = b1b2 : : :bk , where bj 1 j k is the remainder of 3aj modulo 10, belongs to A and the sum of the digits of b equals the sum of the digits of a.

a) Prove that the sum of the digits of a k-digit number in A equals 5k. 9

b) Find the smallest k-digit number which could be an element of A. sum of whose digits is S . Consider the following numbers: b = b1b2 : : : bk c = c1c2 : : : ck and d = d1d2 : : :dk , where bj 1 j k is the remainder of 3aj modulo 10, cj 1 j k is the remainder of 3bj modulo 10 and dj 1 j k is the remainder of 3cj modulo 10. By the conditions of the problem all b, c and d belong to A. Further (1)

Solution: a) Let a = a1a2 : : : ak be a k-digit number from A, the

S=

k X

j =1

aj =

k X

j =1

bj =

k X

j =1

cj =

k X

j =1

dj :

Direct veri cation shows that for xed j the sum aj + bj + cj + dj is equal to 20 (e. g., if aj = 3, then bj = 9 cj = 7 dj = 1 and therefore a 20). It follows j + bj + c j + dj = P Pk P P Pk now from (1) that 4S = k k k j =1 aj + j =1 bj + j =1 cj + j =1 dj = j =1 (aj + bj + cj + dj ) = 20k . Therefore S = 5k, Q. E. D. b) We shall prove that the required number is a = a1a2 : : :a2t, where a1 = 1 a2 = 1 : : : at = 1 at+1 = 9 at+2 = 9 : : : a2t = 9 if k = 2t and b = b1b2 : : : b2t+1, where b1 = 1 b2 = 1 : : : bt = 1 bt+1 = 5 bt+2 = 9 : : : b2t+1 = 9 if k = 2t + 1. It is easy to see that a and b could be elements of a set having the required property. Let k = 2t and suppose there exists c = c1c2 : : :c2t 2 A such that c < a. Since there are no zeroes among the digits of c, we obtain c1 = c2 = : : : = ct = 1. But it follows from a) that the sum of the digits of c is 5k = 10t. The last is possible only if 10

ct+1 = ct+2 = : : : = c2t = 9. Hence c = a, a contradiction with the choice of c. Similarly, suppose k = 2t + 1 and there exists c = c1c2 : : :c2t+1 2 A such that c < b. Since there are no zeroes among the digits of c we obtain c1 = c2 = : : : = ct = 1. But it follows from a) that the sum of the digits of c is 5k = 10t + 5. The latter is possible only if ct+1 5 and since c < b, it follows that ct+1 = 5. It is easy to see now that ct+1 = ct+2 = : : : = c2t = 9. Hence c = b, a contradiction with the choice of c.
where a is a real number:

p Problem 11.1. Given the sequence an = n+a n2 + 1 n = 1 2 : : :,


a) Find the values of a such that the sequence fang1 n=1 is convergent. b) Find the values of a such that the sequence fang1 n=1 is monotone increasing.

Solution: a) If a = ;1, the sequence fang1 n=1 is convergent bep2 1 1 ! 0 when ; 1 p 2 = ;n cause an = n ; n + 1 = 1 + n1 n+ n +1 n ! 1. Conversely, n=1 be convergent. Since p 2 fang1 p 2 let the sequence an = p n ; n + 1 + (a + 1) n + 1,pwe get that the sequence (a+1) n2 + 1 is also convergent. Since n2 + 1 ! 1 when n ! 1, it follows that a + 1 = 6 0, so a = ;1. b) Let fang1 n=1 be a monotone increasing sequence, i. e., an+1
2

11

an for any n. This inequality is equivalent to a(2n + 1) q (?) ;1: p (n + 1)2 + 1 + n2 + 1


Since
q nlim !1
1 2 + 2n + 1 n q q =1 = lim p 1 2 ) + n12 + 1 + n12 (n + 1)2 + 1 + n2 + 1 n!1 (1 + n

it follows from (?) that a ;1.

2n + 1 Conversely, let a ;1. It follows from q < p (n + 1)2 + 1 + n2 + 1 2n + 1 = 1 that (?) holds true so the sequence fa g1 is increasn n=1 n+1+n ing. The required values of a are a 2 ;1 +1).

AB BC and CA in the points C1 A1 and B1. Let the lines determined by the midpoints of the segments AB1 and AC1, BA1 and BC1, CA1 and CB1 meet at points C2 A2 and B2. Prove that the r circumcircle of 4A2B2C2 is of centre O and radius R + 2 .

Problem 11.2. Given a 4ABC with circumcentre O and circumradius R. The incircle of 4ABC is of radius r and touches the sides

Solution: We show rst that the projection B3 of B2 on AC is the

midpoint of AC . Let Ba and Bc be the midpoints of AB1 and CB1. We shall use the standard notation for the elements of 4ABC . 12

r a B3 We obtain B = tg = B2B3 2 p;a B B r , so and Bc B3 = tg 2 = p ; c 2 3 BaB3 = p ; c . Since B B = b , a c BcB3 p ; a 2 p ; c it follows that BaB3 = 2 = CBc and B B = p ; a = ABa , c 3 2 2 2 which gives AB3 = CB3. Therefore B2O = B2B3 + B3O = (p ; c)(p ; a) + R cos . 2r r and by analogy We shall show that the above expression equals R + 2 r , which will complete the proof. A2O = C2O = R + 2 )(p ; a) +R cos = r +R () S ; We obtain that (p ; c2 r 2 2(p ; b) S = R(1 ; cos ) () Sb = 2R sin2 () r = 2p 2p(p ; b) 2 p;b 2 4R sin2 () tg = 2 sin 2 () sin = 2 sin cos , which is b 2 2 sin 2 2 a true equality.

Problem 11.3. Find the smallest natural number n such that the Solution: It is clear that n has at least three divisors and let 1 <
d1 < d2 <
13

sum of the squares of its divisors (including 1 and n) equals (n +3)2.

< dk < n be those di erent from 1 and n. The

conditions of the problem imply 2 (?) d2 + d2 1 + d2 + k = 6n + 8: Let n = p , where p is a prime number. It follows now from (?) that p2 + p4 + + p2 ;2 = 6p + 8, so pn8 and therefore p = 2. The above equality implies 1 + p2 + p4 + + p2 ;4 = 6p ;2 + 2, which is impossible. Therefore k 6= 1 3 5, because otherwise the number of divisors of n equals 3, 5, 7, i. e., n = p2 , n = p4 or n = p6, where p is a prime number. Suppose that k 6. Since didk;i = n, it follows from (?) that (dk;1 ; d1)2 + (dk;2 ; d2)2 + (dk;3 ; d3)2 8. The last inequality is impossible, since the numbers dk;1 ; d1 dk;2 ; d2 and dk;3 ; d3 are distinct (if for example dk;1 ; d1 = dk;2 ; d2 = A, then d1(A + d1) = d2(A + d2), so d1 = d2). We conclude now that k = 2 or k = 4. Assume k = 4. Then n has 6 divisors and thus n is of the form n = p q2, where p and q are distinct prime numbers. (n is not of the form n = p5). It follows from (?) that (??) p2 + q2 + q4 + p2 q2 = 6pq2 + 8: If q 5, then q4 + p2q2 2pq3 10pq2 > 6pq + 8 and therefore q = 2 or q = 3. Direct veri cation shows that inequality (??) is impossible. Thus k = 2 and hence n = pq, where p and q are distinct prime numbers such that p2 + q2 = 6pq + 8: Since qnp2 ; 8, it is easy to see that if p 17 then p = 7, q = 41 and n = 287. Since 172 = 289 > 287, we conclude that the smallest n with the required property is n = 287. 14

You might also like